Está en la página 1de 690

Introduccin al Pensamiento

Cientfico
UNIVERSIDAD DE BUENOS AIRES

Rector Ruben Hallu


Alberto
Vicerrector Edgardo
Barbieri
Mara
Secretaria de Asuntos
Catalina
Acadmicos
Nosiglia
Subsecretara de
Marilina
innovacin y Calidad
Lipsman
Acadmica
PROGRAMA UBA XXI

Claudia
Lombardo
Vicedirectora Diana Mazza
Directora

Mara
Coordinacin Desarrollo
Alejandra
Pedaggico
Codazzi
Coordinacin Produccin Liliana
Multimedial Castillo
Mara
Alejandra
Batista
Ariadna Pou
Patricia
Bucich
Ariel
Guglielmo
Coordinacin Marcela
Comunicacin Integral Gamberini

GUA DE ESTUDIO

Equipo Docente
Cristina
Ambrosini
Especialistas en Cristina
contenidos Ambrosini
Gastn
Beraldi
Introduccin al pensamiento cientfico :
gua de estudio . - 1a ed. - Buenos Aires
: Eudeba, 2012.
- (UBA XXI; 0)
E-Book.
Profesora titular de IPC

ISBN 978-950-23-2026-7
1. Epistemologa.

CDD 121

Eudeba
Universidad de Buenos Aires
1 edicin: agosto de 2012
2012, Editorial Universitaria de
Buenos Aires
Sociedad de Economa Mixta
Av. Rivadavia 1571/73 (1033) Ciudad
de Buenos Aires
Tel.: 4383-8025 / Fax: 4383-2202
www.eudeba.com.ar

UBA XXI es el Programa de Educacin


a Distancia de la Universidad de Buenos
Aires, dirigido a alumnos interesados en
adoptar esta modalidad de enseanza
para cursar algunas materias del Ciclo
Bsico Comn.
Es una propuesta multimedial que brinda
distintos materiales y recursos para
organizar el estudio de modo autnomo,
sin la obligacin de asistir a clases.
En la pgina web institucional
http://www.uba.ar/academicos/uba21
se encuentra la informacin sobre esta
propuesta de estudio y las primeras
orientaciones para incorporarse al

Programa.
Adems, UBA XXI cuenta con un
Campus virtual,
http://www.ubaxxivirtual.uba.ar, un
entorno en el cual los alumnos
interactan con docentes y compaeros,
as como tambin acceden a actividades,
materiales didcticos y recursos
multimediales para acompaar el
cursado de las asignaturas elegidas.
Introduccin al Pensamiento
Cientfico en UBA XXI
Para abordar los contenidos de la
materia IPC en esta modalidad, usted
cuenta con los siguientes materiales y
recursos didcticos:
- Textos impresos de lectura
obligatoria: Gua de Estudio de IPC

y Bibliografa.
- Un espacio en el Campus vir tual
desde el cual podr acceder al correo
electrnico para consultas sobre la
materia, actividades de aprendizaje
como foros de discusin y de
orientacin pedaggica y a otros
recursos didcticos que lo
acompaarn en su estudio. Se
recomienda escuchar los programas
grabados de radio, disponibles en la
Mediateca.
- Tutoras presenciales que son
encuentros optativos para abordar los
temas del programa, aclarar dudas
sobre los contenidos y sobre las
actividades de aprendizaje.
- Programas de radio sobre diversos

temas de la asignatura, que se emiten


por: FM Radio UBA 90.5
- Sala de lectura en la que se puede
consultar la bibliografa obligatoria
para el estudio de la materia, en la
Sede Central de Pte. J. E. Uriburu
950, 1 piso, correo electrnico:
bibliotecaubaxxi@rec.uba.ar.
La Gua de Estudio de IPC es una
herramienta didctica que acompaa la
lectura de la bibliografa obligatoria de
la asignatura. El objetivo principal es
guiar a los alumnos en la comprensin
de la materia. Est organizada siguiendo
el desarrollo del Programa.
En cada unidad se anticipan los temas y
la bibliografa, se sugieren preguntas y

actividades de aprendizaje y se incluyen


los "Documentos de Ctedra", que son
textos elaborados por los profesores
sobre algunos contenidos del programa.
Este material didctico es de lectura
obligatoria.
Las ilustraciones correspondientes a
Alicia en el pas de las maravillas han
sido basadas en los originales de Sir
John Tenniel. Se agradece a Ariel F.
Guglielmo por los dibujos realizados.
Recomendamos ingresar con frecuencia
al espacio que la materia tiene en el
Campus virtual para participar de los
foros y otras actividades que all se

proponen.

Objetivos generales
QUE LOS ALUMNOS:
- comprendan las caractersticas
generales del conocimiento cientfico;
- conozcan los fundamentos del
lenguaje lgico y del lenguaje
cientfico;
- comprendan las principales
orientaciones de la epistemologa
contempornea;
- desarrollen un lenguaje riguroso y
preciso;
- desarrollen la capacidad de anlisis
y el pensamiento crtico;
- valoren al pensamiento y a la
actividad cientfica como

dimensiones fundamentales de la
cultura y la sociedad;
- valoren el papel y el impacto social
de la ciencia y la tecnologa;
- valoren el compromiso social de los
cientficos y del resto de los
ciudadanos.
Objetivos especficos
QUE LOS ALUMNOS:
- reconozcan las estructuras del
lenguaje como paso indispensable
para la comprensin de temas
metodolgicos;
- establezcan diferencias entre el
plano formal o emprico del
conocimiento cientfico;
- evalen las diferentes
comprensiones de las estructuras

cientficas: enfoque sincrnico


(esttico: las teoras vistas como unas
estructuras lgicas) o diacrnico
(dinmico: el proceso histrico de
gnesis, mejoramiento y cambio de
teoras);
- comprendan los aspectos salientes
de los debates contemporneos en el
terreno de la epistemologa y el
impacto social de la ciencia y la
tecnologa en el mundo
contemporneo;
- identifiquen las diferencias tericas
y metodolgicas entre las ciencias
formales y las ciencias fcticas.
Unidad 1. Consideraciones sobre el
lenguaje
1.1. Lenguaje y teoras cientficas

1.2. Lenguaje y realidad


1.3. Uso y mencin del lenguaje
1.4. La Semitica
1.5. Nombrar y clasificar. Vaguedad y
ambigedad
1.6. La definicin
Unidad 2. Argumentacin: el
escenario formal y el escenario
informal
2.1. Las leyes lgicas
2.2. Tautologas, contradicciones,
contingencias
2.3. Los razonamientos. Verdad y
validez
2.4. Argumento y consecuencia lgica.
Reglas lgicas
2.5. La induccin y la analoga
2.6. Es formal la induccin? El intento

de Carnap
2.7. Lgica informal y falacias
materiales
2.8. Toulmin, sobre los usos
argumentativos
Unidad 3. Las ciencias formales
3.1. La matemtica: constructos formales
y realidad
3.2. Sistemas axiomticos
3.3. Propiedades de los sistemas
axiomticos
3.4. Interpretacin y modelos de los
sistemas axiomticos
Unidad 4. La cuestin del mtodo en
las ciencias fcticas
4.1. Estructura de las teoras empricas
4.2. El problema del mtodo cientfico
4.3. Estrategias metodolgicas bsicas

de las ciencias fcticas


4.4. Mtodo inductivo: inductivismo
"estrecho" e inductivismo "sofisticado"
4.5. Mtodo hipottico-deductivo
Unidad 5. Explicacin y prediccin en
ciencias fcticas
5.1. Qu significa "explicar"?
5.2. Concepto de explicacin cientfica
5.3. Modelos de explicacin cientfica
5.4. La especificidad de las ciencias
sociales: intencionalidad y explicacin
por mecanismos
5.5. Dimensin explicativa y dimensin
predictiva
5.6. El enfoque comprensivista en
ciencias sociales
Unidad 6. Corrientes epistemolgicas
contemporneas

6.1. La epistemologa del siglo XX:


problemas fundamentales
6.2. La concepcin epistemolgica del
Positivismo lgico
6.3. Las perspectivas falsacionistas:
Popper y Lakatos
6.4. La ruptura: del racionalismo de
justificacin al anlisis de la historia de
la ciencia
6.5. El pensamiento de Kuhn en
perspectiva
Unidad 7. Ciencia y sociedad
7.1. Poltica cientfica
7.1.1. Caractersticas generales
7.1.2. La planificacin de la poltica
cientfica. Debates contemporneos
7.1.3. La formacin del investigador
7.1.4. Etapas de la poltica cientfica

argentina
7.1.5. Institutos de investigacin y
Universidad
7.2. Ciencia, tecnologa y sociedad
7.2.1. Ciencia, tecnologa y tica: la
responsabilidad social del cientfico y
el tecnlogo
7.2.2. Ciencia y tica: debates en torno a
la neutralidad valorativa de la ciencia
7.2.3. La tica de la investigacin
cientfica
7.2.4. Ciencia bsica, ciencia aplicada y
tecnologa. Cientificismo y
anticientificismo

Unidad 1. Consideraciones sobre el


lenguaje
BIBLIOGRAFA OBLIGATORIA:
Unidad 1 en la Gua de Estudio de IPC,
producida por UBA XXI y editada por
Eudeba, a partir de la edicin de 2010.
Asti Vera, C. y Ambrosini, C. (2009).
Captulo 1. En Argumentos y teoras.
Aproximacin a la Epistemologa.
Buenos Aires: Educando.
BIBLIOGRAFA COMPLEMENTARIA:
Copi, I. (1984). Captulos 2 y 4. En
Introduccin a la lgica. Buenos Aires:
Eudeba.
Unidad 2. Argumentacin: el
escenario formal y el escenario

informal
BIBLIOGRAFA OBLIGATORIA:
Unidad 2 en la Gua de Estudio de IPC,
producida por UBA XXI y editada por
Eudeba, a partir de la edicin de 2010.
Asti Vera, C. y Ambrosini, C. (2009).
Captulos 2 y 3. En Argumentos y
teoras. Aproximacin a la
Epistemologa. Buenos Aires:
Educando.
BIBLIOGRAFA COMPLEMENTARIA:
Copi, I. (1984). Captulos 1 y 3. En
Introduccin a la lgica. Buenos Aires:
Eudeba.
Dez, J. A. y Moulines, C. U. (1999).
Captulo 2. En Fundamentos de
filosofa de la ciencia. Barcelona:
Ariel. Bunge, M. (1997). Captulo 3. En

Epistemologa. Mxico: Siglo XXI.


Klimovsky, G. (1994). Captulo 18. En
Las desventuras del conocimiento
cientfico. Buenos Aires: AZ.
Unidad 3. Las ciencias formales
BIBLIOGRAFA OBLIGATORIA:
Unidad 3 en la Gua de Estudio de IPC,
producida por UBA XXI y editada por
Eudeba, a partir de la edicin de 2010.
Asti Vera, C. y Ambrosini, C. (2009).
Captulo 4. En Argumentos y teoras.
Aproximacin a la Epistemologa.
Buenos Aires: Educando.
BIBLIOGRAFA COMPLEMENTARIA:
Dez, J. A. y Moulines, C. U. (1999).
Captulos 4 y 5. En Fundamentos de
filosofa de la ciencia. Barcelona:
Ariel.

Klimovsky, G. (1994). Captulos 2, 3, 4


y 5. En Las desventuras del
conocimiento cientfico. Buenos Aires:
AZ.
Unidad 4. La cuestin del mtodo en
las ciencias fcticas
BIBLIOGRAFA OBLIGATORIA:
Unidad 4 en la Gua de Estudio de IPC,
producida por UBA XXI y editada por
Eudeba, a partir de la edicin de 2010.
Asti Vera, C. y Ambrosini, C. (2009).
Captulo 5. En Argumentos y teoras.
Aproximacin a la Epistemologa.
Buenos Aires: Educando.
BIBLIOGRAFA OBLIGATORIA:
Cohen, I. y Nagel, E. (1980). Captulos
X, XI, XIV y XVI. En introduccin a la
lgica y al mtodo cientfico (vol. II).

Buenos Aires: Amorrortu.


Hempel, C. G. (1985). Captulos 2, 3 y
4. En Filosofa de la ciencia natural.
Madrid: Alianza.
Klimovsky, G. (1994). Captulos 9, 13 y
14. En Las desventuras del
conocimiento cientfico. Buenos Aires:
AZ.
Unidad 5. Explicacin y prediccin en
ciencias fcticas
BIBLIOGRAFA OBLIGATORIA:
Unidad 5 en la Gua de Estudio de IPC,
producida por UBA XXI y editada por
Eudeba, a partir de la edicin de 2010.
Asti Vera, C. y Ambrosini, C. (2009).
Captulo 6. En Argumentos y teoras.
Aproximacin a la Epistemologa.
Buenos Aires: Educando.

Beraldi, G. (2010). Documento de


Ctedra: La tensin entre explicacin y
comprensin. El problema de la
explicacin en las ciencias sociales.
En esta Gua de Estudio de IPC, Buenos
Aires: Eudeba; y en el Campus virtual
de UBA XXI.
BIBLIOGRAFA COMPLEMENTARIA:
Dez, J. A y Moulines, C. U. (1999).
Captulo 7. En Fundamentos de
filosofa de la ciencia. Barcelona:
Ariel.
Schuster, G. (1986). Captulos 2, 3, 4 y
5. En Explicacin y prediccin. Buenos
Aires: Clacso.
Wright, G. H. von (1979). Explicacin y
comprensin. Madrid: Alianza.
Unidad 6. Corrientes epistemolgicas

contemporneas
BIBLIOGRAFA OBLIGATORIA:
Unidad 6 en la Gua de Estudio de IPC,
producida por UBA XXI y editada por
Eudeba, a partir de la edicin de 2010.
Asti Vera, C. y Ambrosini, C. (2009).
Captulo 7. En Argumentos y teoras.
Aproximacin a la Epistemologa.
Buenos Aires: Educando.
BIBLIOGRAFA COMPLEMENTARIA:
Echeverra, J. (1999). Captulos 1, 2, 3,
4, 5 y 6. En introduccin a la
metodologa de la ciencia. La filosofa
de la Ciencia en el siglo XX. Madrid:
Ctedra.
Klimovsky, G. (1994). Captulos 21, 22
y 23. En Las desventuras del
conocimiento cientfico. Buenos Aires:

AZ.
Unidad 7. Ciencia y sociedad
BIBLIOGRAFA OBLIGATORIA:
Unidad 7 en la Gua de Estudio de IPC,
producida por UBA XXI y editada po
Eudeba, a partir de la edicin de 2010.
Contratti, M. B. (2010). Documento de
Ctedra: Poltica cientfica: problemas
y perspectivas.
En esta Gua de Estudio de IPC, Buenos
Aires: Eudeba; y en el Campus virtual
de UBA XXI.
Contratti, M. B. (2010). Documento de
Ctedra: tica y ciencia.
En esta Gua de Estudio de IPC, Buenos
Aires: Eudeba; y en el Campus virtual
de UBA XXI.

A travs del curso a distancia de esta


materia, introduccin al Pensamiento
Cientfico, se busca poner al alcance de
los estudiantes de la Universidad el
conocimiento de la epistemologa,
aquella regin de la filosofa que
problematiza los mtodos y prcticas de
la ciencia as como evala sus
resultados. Esta propuesta est animada
por la idea de establecer una estrecha
relacin entre "construccin de la
ciudadana" y "enseanza de
epistemologa".
Por qu, desde la perspectiva que
sostenemos en esta asignatura, ensear
epistemologa para posibilitar la
constitucin de la ciudadana plena?

Porque, por un lado, el aprendizaje de la


epistemologa permite, entre otras
cuestiones, distinguir formas de
argumentacin racional de otras que
solamente parecen serlo, y analizar y
debatir en torno de los criterios que
permiten esa distincin. Por otro lado,
favorece el perfeccionamiento de
habilidades de pensamiento necesarias
para interactuar en la vida social. Mas
para que esta posibilidad se concrete, es
imprescindible el acceso a determinados
bienes simblicos: el uso adecuado del
lenguaje; la competencia discursiva en
diversas esferas de la vida social; una
disponibilidad de informacin acorde,
no slo con las demandas profesionales,
sino que posibilite participar de

distintas interacciones en las que las


personas se constituyen como
ciudadanos porque estn en condiciones
de tomar la palabra y sostenerla,
evitando as ser vctimas de
manipulaciones. Desarrollar las
habilidades para pensar y actuar con
flexibilidad a partir de lo que ya se sabe
es apenas el comienzo de esta tarea.
Implementar estrategias para la
construccin del conocimiento implica
establecer un puente entre el producto y
el productor, porque un conocimiento
slo es un producto acabado cuando el
alumno logra pensar en l como un
producto propio. La generacin de este
conocimiento autocrtico y consciente,
que conforma un instrumento importante

para la transformacin de la realidad,


resulta prioritaria.
En la actualidad observamos dos tipos
de situaciones: el prejuicio contra la
teora, que a menudo es presentada ante
la opinin pblica como un intil
pasatiempo para intelectuales, y una
cultura grafa, basada en estmulos
intensos y pasajeros (al estilo de los
videoclips), en la que la capacidad
crtica es cuidadosamente desactivada
para ponerla al servicio de algunos
prejuicios, sintetizados en unas pocas
frases hechas. Arriesgarse a tomar la
palabra es un gesto propio de una
ciudadana activa; estar impedido de
hacerlo revela al lenguaje como barrera
social. Por ello creemos que, en el caso

de esta propuesta, es esencial armonizar


la enseanza de habilidades de
pensamiento y contenidos, donde
podemos apreciar la centralidad de un
uso reflexivo del lenguaje para
propiciar, a su vez, la conformacin de
un conocimiento pluralista, capaz de
estimular en los estudiantes, desde el
inicio de su formacin, el desarrollo de
un pensamiento crtico y autnomo.
Creemos firmemente que la enseaza de
esta materia, de raigambre filosfica, no
es simple transmisin de conocimientos
(en este caso de lgica, nociones de
metodologa e historia de la ciencia),
sino que es produccin y, en tanto
produccin filosfica, es produccin
autocrtica y reflexiva de conocimientos.

Actualmente se menciona la "sociedad


del conocimiento" para aludir al tipo de
sociedades donde el conocimiento es un
factor principal en la produccin de
riquezas, de all que sea cada vez ms
alta la demanda social de educacin
superior en sociedades como la nuestra.
Esta propuesta didctica, entonces,
busca introducir a los alumnos en la
revisin de las estructuras del lenguaje y
los distintos escenarios argumentativos
para que, con estas herramientas, puedan
luego evaluar las distintas posiciones
epistemolgicas y el mundo de debates
que se genera alrededor de estas
particulares y poderosas
manifestaciones de la creatividad y de la
inteligencia humana que llamamos

"ciencias".
Por qu Introduccin al Pensamiento
Cientfico en el inicio de los estudios
superiores?
En este curso se trata de tomar en cuenta
las particularidades del conocimiento
cientfico tal como los problematiza la
epistemologa. Para ello, y antes de
iniciar el estudio de esta disciplina,
conviene recordar que nuestro concepto
de ciencia es un producto histrico de
ascendencia griega. Los babilonios, los
egipcios, los aztecas y los incas
reunieron y registraron mucha
informacin sobre los fenmenos
naturales, con estos conocimientos
pudieron alcanzar grandes logros
culturales y, sin embargo, es discutible

si este conocimiento puede considerarse


"cientfico". Por otra parte, podemos
admitir que la ciencia es un producto
social bastante reciente si lo
comparamos con otros logros culturales
ms antiguos como el arte o la literatura.
Dentro de esta tradicin, para que haya
ciencia, es necesario que estos
conocimientos formen un sistema
ordenado segn criterios lgicos. En la
conformacin de la idea de ciencia, el
epistemlogo argentino Juan Samaja[1]
reconoce dos momentos:
a) Un primer momento que
correspondera al nacimiento de la
ciencia en el sentido ms amplio de la
episteme tal como la concibe Platn
(427-347 a.C.), como un conocimiento

acorde a la razn o logos, como un


poder que permite controlar sus
fundamentos. De all el uso del trmino
logos para designar disciplinas
cientficas como antropologa,
psicologa, etc. En este perodo se
puede ubicar el pasaje de la concepcin
mitolgica del mundo a la concepcin
epistemolgica y los hroes de esta
conquista fueron los filsofos griegos
Tales de Mileto (639-547 a.C.),
Parmnides de Elea (nacido entre el 530
y el 515 a.C.), Pitgoras de Samos
(aproximadamente 582-507 a.C.) y,
especialmente, Aristteles (384-322
a.C.), entre otros.
b) Un segundo momento que
corresponde al nacimiento de las

ciencias positivas segn se concibe a


partir del desarrollo de las ciencias
experimentales con Galileo Galilei
(1564-1642), Isaac Newton (16431727), Blaise Pascal (1623-1662), Ren
Descartes (1596-1650), para nombrar
slo a los primeros.
Desde este punto de vista, el
conocimiento cientfico es el resultado
de una prctica que consiste
bsicamente en "teorizar" acerca de
distintas entidades, empricas o
formales. "Argumentar" y "teorizar" no
son sinnimos, sin embargo con estos
trminos sealamos dos actividades
ntimamente conectadas ya que la
construccin de una teora supone
utilizar argumentos que la fundamenten o

justifiquen. Realizar una actividad no


implica necesariamente estar capacitado
para formular las reglas de estas
prcticas ni para criticar algunas de las
creencias vigentes y menos para conocer
su historia y las distintas
interpretaciones que puedan hacerse de
ellas. Est claro que se puede ser un
excelente director de cine y un psimo
crtico de arte o un destacado jugador de
tenis y un mal comentarista deportivo.
Salvando las distancias, de un modo
parecido teorizar, como hablar o
argumentar, es una actividad que puede
realizarse correctamente sin realizar
explcitamente una revisin filosfica de
sus presupuestos. Los cientficos y los
estudiantes de las distintas ciencias, a

menudo, suelen mirar con desconfianza a


los epistemlogos y esta desconfianza en
parte se justifica por lo abstracto de sus
formulaciones que, a veces, parecen
conducir a discusiones bizantinas
completamente alejadas de las prcticas
cientficas concretas y de los acuciantes
dilemas morales a los que se enfrentan.
Por otra parte, algunos epistemlogos
sostienen que para hacer epistemologa
es necesario ser cientfico o estar
familiarizado con la produccin de
teoras en el interior de alguna
disciplina, mientras que otros defienden
la autonoma y especificidad del
discurso filosfico y el cientfico.
Volviendo a la analoga anterior,
equivale a la discusin acerca de si,

para ser crtico literario, es necesario


haber escrito una novela o para discutir
sobre cine, ser cineasta. Ms all de
estas polmicas, podemos admitir que es
deseable que un buen cientfico sea
capaz de poder reflexionar crticamente
acerca de su objeto de estudio y de
evaluar los resultados sociales de su
prctica cientfica. La epistemologa se
ocupa de formular teoras filosficas
sobre teoras cientficas buscando as
evitar la aceptacin dogmtica y acrtica
de los logros de las distintas ciencias.
Desde esta concepcin, renunciamos a
asentar una nica definicin de
"ciencia". Por el contrario, de lo que se
tratar es de presentar distintas
posiciones epistemolgicas que en el

desarrollo del conocimiento occidental


dieron lugar a distintas concepciones de
las ciencias.
Un curso de epistemologa en el inicio
de la formacin universitaria de los
futuros cientficos o profesionales
involucrados en el tratamiento de teoras
cientficas tiene el sentido de hacer
explcitos los problemas que afectan a la
produccin de teoras y a las
particularidades de esta actividad de tan
alto y controvertido impacto social. A
menudo se asocia el conocimiento
cientfico al progreso de la humanidad y
al logro de desarrollos tecnolgicos que
permiten aumentar nuestra calidad de
vida, como son la elaboracin de
vacunas o drogas para paliar

enfermedades antes incurables, pero


tambin es cierto que puede presentarse
a la ciencia como responsable de la
contaminacin del planeta y de las
distintas amenazas blicas. En este curso
no pretendemos tomar partido por
ninguna concepcin pesimista u
optimista acerca de la ciencia y sus
promesas o amenazas sino, por el
contrario, ya antes de adoptar una
posicin, esperamos que el alumno
refuerce las competencias
argumentativas para fundamentar alguna.
En nuestro tiempo, constatamos que el
viejo ideal griego del conocimiento del
mundo como parte de una empresa
desinteresada no da cuenta del concepto
actual de ciencia, ya que de ella se

espera alguna "utilidad" para ordenar


los fenmenos de un modo "eficaz".
Desde este punto de vista, la
aceptabilidad de las teoras se sustenta
en la obtencin de xitos tecnolgicos y
la verdad cientfica queda asimilada a la
utilidad. La ciencia adquiere el alto
grado de prestigio a partir de su utilidad
social, en especial a partir de su
interdependencia con el sistema
productivo. Desde la concepcin
positivista (que recorreremos en varios
captulos de este curso) se concibe a la
ciencia como un producto de la razn,
que suministra el conocimiento
necesario para dominar la naturaleza y
controlar el orden social. Segn esta
idea, la ciencia es la principal

herramienta del progreso humano. No se


trata de negar este supuesto, sino de
constatar su presencia y llevarlo al
campo reflexivo.
Bsicamente, en nuestras concepciones
acerca de la ciencia creemos que el
conocimiento cientfico no nos fue
otorgado como un don de los dioses,
sino que es el resultado de las
actividades humanas que, al igual que
otras adquisiciones, son productos
falibles y perfectibles, que en ningn
caso sus resultados son inocuos, por el
contrario, que deben ser cuestionados y
revisados tanto por la comunidad
cientfica como por el resto de la
comunidad. Esperamos que este curso
brinde herramientas conceptuales a las

nuevas generaciones de cientficos y


profesionales en esta tarea central para
la construccin de una ciudadana
activa.
Un curso, una aventura llevados de la
mano de Alicia
Recurrir a la figura de Alicia en el pas
de las maravillas[2] en un curso de
epistemologa no es una originalidad.
Por el contrario, los personajes
presentes en el cuento son conos de
nuestra cultura y su autor es objeto de
culto en el mundo de los filsofos y
cientficos desde la aparicin del
cuento, en Inglaterra, en 1864. Podemos
advertir que en esa poca haba una
fuerte revisin de los conceptos
centrales de la fsica y la biologa y que

esta obra es contempornea a la


publicacin de El origen de las
especies por medio de la seleccin
natural, o la preservacin de las razas
preferidas en la lucha por la vida,
publicada en 1859 por otro cientfico
revolucionario ingls, Charles Darwin.
Para el gran pblico, especialmente el
infantil, el xito editorial de Alicia fue
inmediato. Tanto en el mundo de la
literatura como en el de la ciencia,
distintos autores reconocieron en el uso
y la creacin de nuevos lenguajes la
puerta de entrada a un experimento
revolucionario. Si optamos por definir
al lenguaje como un conjunto de
smbolos regidos por reglas, la lgica
que usamos es la crcel del lenguaje ya

que nos confina al uso de algunas,


consagradas como las reglas
"correctas". Lewis Carroll nos abre las
puertas de la celda para que salgamos a
"jugar", a usar otras reglas, otras leyes,
otros principios, mostrando as todo tipo
de situaciones paradjicas y
demenciales. A juicio del escritor
argentino Jorge Luis Borges (18991986), las aventuras de Alicia parecen
arbitrarias e irresponsables, luego
advertimos que encierran "el secreto
rigor del ajedrez y de la baraja", es
decir, denuncian la naturaleza
convencional y arbitraria de estas
reglas, lo que permite experimentar
mentalmente, imaginativamente, la
posibilidad de transitar distintos

rdenes.
La inocente mirada de Alicia, perpleja
pero irreverente, consigue un efecto
desestructurador sobre nuestras
convicciones profundas al poner en
descubierto el absurdo de las
situaciones que no encuadran
perfectamente en las reglas conocidas.
Bajo un disfraz de locura, el relato
disimula, detrs de las actitudes
candorosas de una nia, la agudeza de
una inteligencia crtica capaz de
ridiculizar y mostrar en toda su
insignificancia y precariedad algunas de
las categoras ms respetadas por las
ciencias y la lgica. Acompaando a
Alicia en los pasajes cruciales del
cuento, encontramos la figura tutelar del

gato de Cheshire que, al modo de un


alter ego, aparece y desaparece a
voluntad (hasta quedar solamente su
sonrisa) y es el nico que toma en broma
todo lo que ocurre.
En este curso aparecern varias veces
alusiones a pasajes de la obra de Lewis
Carroll as como fragmentos de textos
de otro admirador de Alicia, Borges.
Ambos consagraron gran parte de su
genio creativo a la imaginacin de
situaciones donde aparecen fuertemente
cuestionadas las reglas y presupuestos
del lenguaje. Conmover las creencias
profundas acerca de nuestras
habilidades lingsticas provoca
perplejidad y una cierta sensacin de
inseguridad, ya que con ello ponemos en

tela de juicio nuestras certezas y


seguridades acerca del mundo. Revisar
los mecanismos ntimos de la
construccin de teoras implica superar
las concepciones intuitivas para acceder
(haciendo un esfuerzo contraintuitivo) a
la comprensin de concepciones
alternativas acerca de la relacin entre
el lenguaje y la realidad. Poner una cua
entre estos dos planos es uno de los
logros de la epistemologa
contempornea. Este pasaje es necesario
para acceder al territorio de la
epistemologa, especialmente para
deslindar entre el plano del lenguaje
(formal) y el plano de la realidad
(emprico) al momento de diferenciar
entre las ciencias formales y fcticas y

advertir que, a lo largo de todo el curso,


estamos haciendo afirmaciones acerca
de teoras y que el conocimiento de la
realidad est mediado por el
conocimiento de teoras cientficas, que
no se da de un modo inmediato ni natural
y que, por el contrario, implica la
"desnaturalizacin" de nuestras certezas
para transformarlas en hiptesis
conjeturales y provisorias.
Alicia en el pas de las maravillas: [3]
resea de la obra
Quin escribi un libro de cuentos
dedicado a su amiga predilecta, Alice
Liddell? Podemos afirmar que fue el
Reverendo Charles Lutwidge Dodgson

(1832-1898), profesor de Lgica y


Matemtica de la Universidad de
Oxford, autor de libros tcnicos, y/o
podemos inclinarnos a pensar que el
seudnimo "Lewis Carroll"
[4]

esconde a otro personaje, no


yuxtapuesto sino distinto y dividido del
anterior. En este caso decidir por la "y"
o la "o" no es un detalle. Bajo el
seudnimo de Lewis Carroll
encontramos, en las aventuras de Alicia,
plasmadas las pesadillas de su autor, el
lgico-matemtico ingls Charles
Dodgson, tambin autor de tratados
acadmicos de geometra y de lgica.

Las aventuras de Alicia comienzan


cuando, en una soleada tarde de verano,
la nia ve pasar a un conejo -con
chaleco, reloj y guantes- y movida por
la curiosidad lo sigue hasta una
madriguera en la que desciende
lentamente hasta caer en un pozo muy
profundo sobre un montn de palos y
hojas secas. Carroll, como matemtico,
manifestaba enorme inters por las
teoras de la gravitacin universal, que
estaban siendo objeto de gran desarrollo
en su tiempo. Se ha sealado la
similitud entre la cada de Alicia en la
madriguera y un ejemplo empleado por
Einstein para ilustrar algunos aspectos
de su teora. Podemos decir que las
aventuras se inician a partir de lo que

podramos denominar un "resorte


moral", un impulso que sobrepasa la
capacidad reflexiva y voluntaria del
personaje. Alicia se ve superada por la
rigidez mecnica de la gravedad que la
coloca al margen de una accin
deliberada y libre puesto que queda
reducida a la condicin de "cosa que
cae", aunque esta cada parece
responder a otras leyes distintas a las
conocidas por la fsica de la poca ya
que cae lentamente en una situacin que
llamaramos "antigravitatoria", hasta
llegar al fondo de la madriguera.
Alicia comienza, entonces, una serie de
metamorfosis a partir de lo que bebe o
come: se achica hasta casi desaparecer

o se agranda y pierde de vista sus


propios pies. Tantos cambios
imprevistos provocan el llanto de la
nia, la que comienza a dudar de si
seguir siendo la que era antes de caer
en la madriguera. La angustia de Alicia
se produce frente a la posibilidad de la
prdida de identidad o de unidad ante
los sorpresivos cambios, que ella
identifica con la prdida del nombre.
Este tema es retomado, en la
conversacin entre Alicia y la Oruga, la
que le da consejos para sobrellevar los
cambios corporales radicales, ya que
ella es experta en metamorfosis
profundas.
El efecto "bola de nieve", de

acumulacin de situaciones, presente en


toda la obra, se produce a partir de
varias metamorfosis que trastocan la
condicin inicial y que consigue
neutralizar el asombro y la perplejidad
del lector, el que se ve obligado a
dejarse llevar por una dinmica
vertiginosa.
El encuentro de Alicia con la Duquesa y
con el gato de Cheshire es uno de los
captulos ms conocidos de la obra. El
gato de Cheshire puede ser visto como
el animal emblemtico de la pequea
herona de esta historia, ya que la
acompaa desde un trasfondo fantasmal
donde aparece y desaparece; entre todos
los animales es el nico capaz de

enfrentar a la Reina con su sonrisa


sardnica. Alicia se encuentra por
segunda vez con el sonriente gato de
Cheshire posado sobre la rama de un
rbol -la primera vez lo ve en la cocina
de la Duquesa- y ste le aconseja
conocer al Sombrerero loco y a la
Liebre de marzo. En este episodio
aparece cuestionada la idea del tiempo
(recordemos que al inicio aparece
cuestionado el espacio) y se presentan
curiosos relojes que miden el ao o los
das del mes en lugar de las horas.
Luego de una conversacin disparatada,
Alicia da por terminada "la merienda de
locos", en la que el Tiempo se ha
detenido a las seis en punto de la tarde,
lo que condena a los personajes a

repetir eternamente la ceremonia del t


sin darles tiempo siquiera a lavar las
tazas.
Nuevamente, luego de esta extraa
merienda, la nia se ve arrastrada por
un impulso o resorte que la lleva al
Juego de Croquet de la Reina. En este
captulo, Carroll ridiculiza los aspectos
esenciales del espritu ingls de su
poca y presenta a una Reina desptica
(recordemos que estaba en el trono la
Reina Victoria y que su personalidad
impregn la poca dando lugar al
adjetivo "victoriano" para referirse a
cosas que sucedieron en su reinado),
dispuesta a cortar la cabeza de todo el
mundo. En el campo de Croquet de la

Reina de Corazones, las pelotas son


erizos vivos y los palos unos incmodos
flamencos que intentan escapar, los
arcos los forman los soldados de la
Reina curvando sus cuerpos de naipes.
Todos juegan al mismo tiempo sin
esperar su turno, peleando con los
erizos que se mueven caprichosamente
por todo el campo haciendo desaparecer
el resultado de la jugada. Para alegra
de Alicia se hace visible la cabeza del
gato de Cheshire y, cuando terminan de
aparecer las orejas, le cuenta entre
asombrada y divertida: "No estn
jugando limpiamente", empez
diciendo Alicia con tono quejumbroso,
"y se estn peleando todo el tiempo, de
forma que no hay quin oiga nada..., y

adems, nadie hace demasiado caso a


las reglas del juego; parece como si no
tuviera ninguna, o, en todo caso, si las
hay, nadie parece que las est
siguiendo...[...] es tan seguro que va a
ganar la Reina que no vale la pena que
los dems sigan jugando."
Un juego de esta ndole, sin reglas,
donde se sabe de antemano quin va a
ganarlo, donde no se distingue entre la
destreza y la pura casualidad, no puede
ser puesto como modelo sino como
contrafigura del juego social
identificado entre los ingleses como
fairplay. En los tres captulos que
siguen ("Historia de la Tortuga
Artificial", "La cuadrilla de la

Langosta" y "Quin rob las tartas?")


se multiplican los juegos de palabras, la
ridiculizacin de los cuentos con
moraleja y el sistema escolar ingls.
Carroll explota al mximo los efectos
cmicos que crea el lenguaje a travs de
distintos mecanismos que, en muchos de
los casos, son intraducibles. La crtica
social se ve reforzada por las
caracterizaciones zoomrficas de los
personajes. Estos animales (reales o
fantsticos) que hablan con la nia,
cuestionan los parmetros de la
racionalidad en un territorio donde se
diluye el lmite entre la razn y la
locura. En este sentido puede asociarse
a Carroll con la tradicin iniciada por
Esopo y continuada por La Fontaine,

hasta el mismo Walt Disney. Los


animales que rodean al personaje son
puestos como contrafiguras, unas veces,
o como prolongacin o alter ego del
protagonista en otros casos. En el ltimo
episodio, Alicia se ve involucrada en un
proceso judicial disparatado que
presenta algunas notas en comn con el
Juego de Croquet de la Reina. La
acusacin recae sobre la Sota de
Corazones por robar unas tartas de la
Reina. Nuevamente la ausencia de
reglas preexistentes junto al arbitrario y
caprichoso ejercicio del poder por parte
de la Reina posibilita la aparicin del
absurdo:
"En este momento, el Rey, que haba

estado muy ocupado escribiendo


apretadamente en su libreta de notas,
exclam: "Silencio!", y ley a
continuacin lo que acababa de
anotar: "Regla Nmero Cuarenta y
Dos: Todas las personas que midan
ms de una milla de altura habrn de
abandonar la Sala". Todos miraron a
Alicia.
"pero si yo no mido una milla de
altura!", dijo Alicia.
"Ciertamente que s!", declar el Rey.
"Casi dos millas", aadi la Reina.
"pues lo que es yo, no me marchar en
ningn caso", anunci Alicia;
"adems, esa regla no vale porque se
la acaba de inventar usted".
"Es la regla ms antigua de todo el

libro", asegur el Rey.


"Entonces sera la primera y no la
cuarenta y dos", acus Alicia.
Las actitudes contradictorias del Rey y
la Reina, junto a la sorpresa de la
protagonista frente a un nuevo cambio
de tamao, precipitan el final cuando
Alicia asume una decisin vital,
transformadora, que revela la
precariedad y fragilidad de las
convenciones admitidas:
"A callar!", vocifer la Reina
ponindose morada de rabia. "pues no
me callo", respondi Alicia.
"Que le corten la cabeza!", chill la
Reina con toda la fuerza de sus
pulmones; pero nadie hizo el menor

movimiento.
"Quin les va a hacer caso?", dijo
Alicia (que para entonces ya haba
recobrado su estatura de todos los
das). "Si no son ms que un mazo de
cartas!"
Alicia despierta del sueo y vuelve al
punto de partida. El mundo del absurdo,
en el que estuvo sumergida, queda
abolido. La seguridad amenazada, la
tranquilidad perdida por la irrupcin de
un "otro mundo" incomprensible se
diluyen en una sonrisa cuando logra
restablecerse el orden conocido y todo
se encarrila nuevamente en la
normalidad. Podemos ver, en el final de
la obra, que el personaje recupera la

condicin de persona a partir de un acto


de voluntad, de rebelda: la pesadilla
termina en el momento en que una
decisin de no-sumisin, de
sublevacin aniquila la coaccin de un
mundo donde estaba reducida a la
condicin de ttere y le permite
recuperar su libertad, su autonoma.
Propuesta de un plan de estudio para
los alumnos
La lectura del material de estudio
requiere la puesta en prctica de
distintas habilidades por parte de los
alumnos. Una competencia necesaria es
la de deslindar los temas centrales de
aquellos que los sustentan o justifican.
Es decir, es necesario identificar dentro

de la bibliografa, los conceptos y


nociones que vertebran los contenidos
del curso y aquellos que son
comentarios sobre otras posiciones,
polmicas, distintos criterios de
clasificacin, enfoques, etc. En este
sentido, el abordaje del material de
estudio supone una "estrategia de
lectura", una jerarquizacin de los
conceptos. En virtud de un mejor modo
de organizacin para el estudio,
sugerimos que:
- Tome contacto con el programa de
la materia, en el que encontrar los
contenidos y la bibliografa,
identificados por unidad. Lea la
Introduccin a la materia y las
presentaciones de cada unidad y luego

la bibliografa obligatoria.
- Realice las actividades propuestas
en esta gua de estudio, buscando la
informacin que se requiera. En cada
unidad, adems del libro de lectura
obligatoria, se encuentra sealada una
bibliografa de consulta. Tome en
cuenta que en el nivel de estudios
superiores es necesario recurrir a
fuentes bibliogrficas reconocidas.
Internet es un recurso muy accesible
para encontrar informacin pero,
como toda herramienta, debe ser
usada con cierta destreza, en este
caso, para poder determinar aquellas
fuentes bien documentadas de otras
poco autorizadas. La bsqueda de
informacin requiere un

entrenamiento y ste puede ser el


momento de adquirir esta
competencia, necesaria para continuar
estudios universitarios. Cuando dude
acerca de la confiabilidad de la fuente
encontrada puede consultar a los
docentes en las distintas instancias de
tutoras presenciales y virtuales.
- A partir de las situaciones
problemticas, propuestas por las
actividades, recurra al material de
estudio y realice esquemas, mapas
conceptuales, resmenes y todo tipo
de actividades que permitan organizar
estos contenidos, unidad por unidad.
En esa tnica, conviene localizar en
espacio y tiempo a los autores
principales que aparezcan

mencionados. Los ideales cientficos


corresponden a perodos histricos
determinados y se relacionan
estrechamente con ideales religiosos,
estticos, polticos y sociales.
Conviene tomar en cuenta el contexto
histrico de los autores y de las
concepciones epistemolgicas que
sostienen para poder comprender
mejor sus planteos y el mundo de
ideas que les da sentido. Para ello, se
propone realizar una lnea histrica
que podr ir completndose con el
orden cronolgico de los autores, a
medida que vaya avanzando en el
curso. Advierta que la primera vez
que se los nombra aparecen entre
parntesis sus datos de nacimiento y

muerte.
- Prepare con anticipacin las
actividades y tngalas a mano para
despejar sus dudas a travs de los
distintos medios de comunicacin
ofrecidos por UBA XXI (correo
electrnico, tutoras presenciales,
foros, etc.). Recuerde que el mejor
resumen o esquema es el que realiza
usted mismo y que el modo de
aprender a hacerlo es intentndolo
hasta adquirir esta destreza. Si se nos
permite una analoga con la
adquisicin de habilidades fsicas
como "andar en bicicleta" o "nadar",
la adquisicin de las competencias
para estudiar es tambin cuestin de
"entrenamiento".

La construccin del conocimiento es un


logro social que se alcanza en el
contacto con los materiales de estudio y
en el dilogo con otras personas.
Realizar un curso a distancia no es
equivalente a estudiar en solitario. Las
tutoras presenciales y el campus virtual
permiten el encuentro con otros alumnos,
con los docentes a cargo del dictado de
la materia y con los pedagogos
encargados de orientar la resolucin de
problemas de distinta ndole.
Recomendamos enfticamente usar estos
recursos previstos para la realizacin de
este curso y, en la medida de lo posible,
conformar una comunidad de estudio con
otros compaeros del curso.
Los invitamos, adems, a escuchar los

programas de radio y los recursos


sonoros disponibles en la Mediateca del
Campus, en los que se abordan diversos
temas de la materia.
Respecto a las Actividades de
aprendizaje
Las actividades propuestas en cada
unidad estn orientadas a la
problematizacin de los contenidos del
curso y no son indicativas del tipo de
ejercitacin que se tomar en los
parciales. Por el contrario, tienen el
sentido de presentar problemticamente
distintos "casos" para ilustrar diversos
aspectos de las teoras abordadas y
profundizar en la comprensin de los
temas. Lo importante, en estos
ejercicios, no es "dar con la respuesta

correcta", sino pensar y poder justificar.


"Justificar", en este caso, tiene el
sentido de hacer explcitos los aspectos
tericos de los distintos temas
(definiciones, reglas o criterios de
clasificacin vinculados a los
conceptos). En ningn caso esta
aplicacin ser meramente mecnica ya
que se tratar de interpretar estos
conceptos en base a algunos criterios y
cabr la posibilidad de que se
privilegien algunos por encima de otros.
Por ejemplo: en base a una clasificacin
de distintas funciones del lenguaje
preguntamos en dnde ubicara distintos
casos, se debe tratar de poner en juego
esta clasificacin y despus dar cuenta
del criterio que se adopt en cada

eleccin. Aqu lo importante es


constatar si se consider el criterio de la
clasificacin. Sabemos que hay una
brecha entre la definicin de un
concepto y su aplicacin a casos
individuales. Para usar un ejemplo
conocido, digamos que, por
estrictamente definida que est la
definicin de "gol" en un reglamento de
ftbol, es bastante comn que se generen
fuertes polmicas al momento de decidir
si una jugada fue un gol o no, es decir,
cul es la interpretacin del reglamento.
Ms que acertar con la respuesta
correcta, lo que buscan estos ejercicios
es problematizar los conceptos
presentados en el curso y mostrar la
posibilidad de aplicacin sobre casos

donde, ms all de lo anecdtico y


particular de la situacin presentada,
pueda llegarse a una comprensin ms
profunda de la teora y no la mera
asimilacin de datos. Conviene recordar
que los temas presentados son de ndole
filosfica y motivo de disputas tericas
en las que pueden sostenerse posiciones
distintas, o abordarse los problemas
desde ngulos diferentes segn distintas
decisiones metodolgicas.

Temas de la Unidad
1.1. Lenguaje y teoras cientficas
1.2. Lenguaje y realidad
1.3. Uso y mencin del lenguaje
1.4. La Semitica
1.5. Nombrar y clasificar. Vaguedad y
ambigedad
1.6. La definicin
Bibliografa obligatoria
Unidad 1 en la Gua de Estudio de IPC,
producida por UBA XXI y editada por
Eudeba, a partir de la edicin de 2010.
Asti Vera, C. y Ambrosini, C. (2009).
Captulo 1. En Argumentos y teoras.
Aproximacin a la Epistemologa.

Buenos Aires: Educando.


BIBLIOGRAFA COMPLEMENTARIA
Copi, I. (1984). Captulos 2 y 4. En
Introduccin a la lgica. Buenos Aires:
Eudeba.
Presentacin de la Unidad

Los doce jurados estaban muy


atareados escribiendo en sus pizarras.
"Questn haciendo?" susurr Alicia
a Grifo; "no pueden estar
nada, puesto que el juicio no ha
empezado todava ".
"Estn escribiendo sus nombres ",
cuchiche Grifo, "no vaya a ser que los
olviden antes de que acabe el juicio ".
"Pero es que son imbciles?", empez a

decir Alicia, con una voz muy


indignada pero se call a tiempo al ver
que el Conejo Blanco gritaba
"Silencio en la Sala!", y al ver que el
rey se calaba las gafas y miraba
severamente de un lado para otro
intentando descubrir quin era el que
estaba hablando.
(Carroll, L.,
Alicia en el pas de las maravillas)

Por qu comenzamos este curso


abordando el tema del lenguaje? En
principio, aceptamos que toda teora

cientfica es una construccin


lingstica. El lenguaje es el instrumento
bsico para la construccin de teoras ya
que es impensable una teora "inefable",
es decir, una teora que no pudiera
expresarse en algn lenguaje. La
revisin de las estructuras lingsticas,
presentes en las teoras cientficas, se
har desde dos disciplinas
emparentadas: la Semitica (estudio de
los signos) y la Lgica (estudio de las
estructuras del lenguaje). Estas dos
disciplinas actualmente tienen
desarrollos tericos autnomos aunque
remiten a un origen comn en las teoras
griegas. Comenzamos este recorrido con
la concepcin mgica que identifica al
nombre con el alma o espritu de la cosa

nombrada, antes de mencionar las ideas


de Platn presentes en la obra Cratilo
para caracterizar la teora esencialista.
A continuacin estudiaremos la posicin
revolucionaria del filsofo y telogo
franciscano ingls Guillermo de Ockham
(1298-1349), quien introduce, en el
siglo XIV, una novedosa teora acerca
del signo lingstico, en oposicin a la
teora esencialista de Platn. La
posicin de Ockham, caratulada como
"nominalismo", nos introduce en las
nociones de la Semitica al proponer un
enfoque rival a la concepcin del
lenguaje del esencialismo utilizando una
nueva teora acerca de los signos.
Advertimos en este punto que, en el
texto de la bibliografa, aparecen

distintas referencias al semilogo


italiano contemporneo Umberto Eco (n.
1932) quien rinde homenaje a Ockham
en su novela El nombre de la rosa y
problematiza all y en otros tratados de
semiologa algunos de los problemas
ligados al campo semntico y
pragmtico del lenguaje.
Es importante comprender la diferencia
entre uso y mencin del lenguaje para
destacar el nivel metalingstico de la
Epistemologa, ya que no advertir la
distincin entre las teoras, consideradas
como entidades lingsticas, y la
realidad, da lugar a todo tipo de
confusiones y paradojas.
En el terreno de la Semitica veremos,
en primer lugar, la distincin del

filsofo y cientfico estadounidense


Charles Peirce (1839-1914) entre signo
natural, cono y smbolo a los efectos de
llegar a comprender el sentido de la
definicin del lenguaje como "conjunto
reglado de smbolos".
En segundo lugar, nos aproximaremos a
los distintos temas que aluden a la
divisin del estudio de los signos: la
dimensin sintctica (acerca de las
relaciones entre signos y las reglas que
los ordenan), la semntica (acerca de las
relaciones entre el signo y sus
significados) y la pragmtica (acerca de
la relacin entre el signo y sus
intrpretes o usuarios).
Al estudiar la dimensin semntica, nos
detendremos en la nocin de "trmino"

usado para nombrar a una de las


estructuras lgicas y con ello entramos
ya en el territorio de la Lgica (para la
cual los trminos son unidades de
significacin). En el terreno de los
trminos (signos lingsticos, nombres,
smbolos) se distingue entre
designacin, extensin y denotacin,
considerados como partes del
significado.
Con la dimensin pragmtica y la
distincin entre las funciones del
lenguaje, se introduce el concepto de
"proposicin", otra de las estructuras
lgicas que permite predicar el concepto
de "verdadero" o "falso" a partir de
tener alguna teora acerca de la verdad
proposicional. Para ello, tomaremos la

definicin de proposicin del filsofo


austraco Ludwig Wittgenstein
(18891951) citado en la bibliografa
obligatoria:
Lo que engrana con el concepto de
verdad (como una rueda dentada) eso es
una proposicin [...] Y lo que es una
proposicin est en un sentido
determinado por las reglas de formacin
oracional (de la lengua castellana, por
ejemplo) y en otro sentido por el uso del
signo en el juego del lenguaje. (Asti
Vera y Ambrosini, 2009. 26-27)
El concepto semntico de verdad del
lgico polaco Alfred Tarski (19011983)
es presentado para destacar uno de los
intentos, en la semntica contempornea,
de definir el concepto de verdad

proposicional evitando la incursin en


paradojas.
Adems de los temas mencionados,
tendr que estudiar las distintas
operaciones lingsticas, fundamentales
en el uso de un lenguaje, en especial en
el caso de los lenguajes cientficos:
nombrar, clasificar, definir. Es muy
importante reconocer sus
particularidades antes de entrar en el
anlisis de la estructura de las teoras
cientficas.
La clasificacin de las ciencias que se
encuentra al inicio del captulo 1 de la
bibliografa, servir de parmetro o
mapa conceptual til para anticipar los
temas que se trabajarn en las diferentes
unidades del programa. Como sostiene

Borges, toda clasificacin es arbitraria y


conjetural. Esta clasificacin de las
ciencias, entre formales y fcticas, no se
la presenta con la intencin de
legitimarla como la clasificacin
correcta, sino como la que,
metodolgicamente, utilizamos en este
curso para ordenar los distintos temas
que se presentan.

Actividades de
aprendizaje
ACTIVIDAD 1. LENGUAJE Y
REALIDAD
En el punto 1.2. del captulo 1, se

presentan dos posiciones antagnicas


acerca del modo de justificar la relacin
entre el lenguaje y la realidad: Platn y
Guillermo de Ockham. Estas actividades
estn orientadas a destacar la diferencia
entre estas dos posiciones.

1.1. La suppositio materialis


en El nombre de la rosa de
Umberto Eco[5]
Lea los siguientes prrafos y luego
responda las preguntas que aparecen a
continuacin.
En la novela histrica El nombre de la rosa, el semilogo
italiano Umberto Eco (1932) rinde homenaje a la figura de
Guillermo de Ockham a travs del personaje de un monje

franciscano ingls, Guillermo de Baskerville.


Para Francisco Bertelloni,

[6]
el tema central del relato es el totalitarismo de la verdad que
puede mover a los hombres a matarse unos a otros. Segn
Bertelloni, este tema le permite a Eco vincular una trama
filosfica e histrica -las luchas doctrinales en la Edad
Media- con una trama policial, una serie de asesinatos cuyas
vctimas tienen un rasgo en comn: estn todos interesados
en el mismo libro y mueren a causa de l.
El relato comienza una maana de noviembre de 1327 en el
norte de Italia y se desarrolla en siete das. La historia gira
alrededor de la bsqueda del autor de los crmenes, un
fantico que mata por extremo amor a la verdad pues no
tolera que la cosmovisin cristiana sea sustituida por las
doctrinas de un Aristteles recin descubierto, distinto al
apropiado por la doctrina oficial de la iglesia. Se torna
peligroso para la Teologa un Aristteles "que mira la tierra
antes que el cielo" y debe ser detenido incluso mediante el
crimen. Eco propone mostrar que en la Edad Media la
filosofa no fue inofensiva, sino que tuvo efectos sociales
relevantes. Eco apela al nominalismo empirista de Guillermo
de Ockham, la nueva filosofa del siglo XIV, y la coloca como

trasfondo de los dilogos de Guillermo de Baskerville y su


discpulo Adso.
El trasfondo histrico se relaciona con las pretensiones del
Papa Bonifacio VIII de conseguir la plenitudo potestatis, la
totalidad del poder espiritual y terrenal en la Bula Unam
Sanctam de 1302, promulgada 25 aos antes del inicio de la
historia contada por Eco, donde define una teocracia papal:
la iglesia como un cuerpo con una cabeza, el Papa. La Bula
concluye que los reyes y emperadores deben subordinarse
al poder espiritual para alcanzar la salvacin. Los franceses
consiguen que se fije la sede del papado en Avin. La
rebelin contra esta imposicin papal se concentra en la
corte imperial de Luis de Baviera en Munich donde
encuentran refugio Ockham, Marsilio de Padua y otros
franciscanos que proponen volver a la iglesia primitiva y
separar la iglesia del Estado. Al cuestionar el derecho de
propiedad como parte del derecho natural, Ockham procura
desmontar la estructura jerrquica de la iglesia eliminando el
papel del clero como intermediario entre Dios y los hombres
del mismo modo que, en el plano de la lgica, haba
eliminado las entidades metafsicas como intermediarias
entre Dios y los individuos, siempre singulares. En la abada,
Guillermo se involucra en una trama policial (al modo de las
novelas de detectives) donde se suceden los crmenes. Para
resolver el problema de contestar quin es el asesino? Eco,

que parti de Ockham, va ms all del contexto medieval y


hace de Guillermo de Baskerville un Sherlock Holmes con
alusiones a Ludwig Wittgenstein y a la semitica
contempornea sin dejar de homenajear, en la imagen del
bibliotecario espaol ciego, Jorge de Burgos, a Jorge Luis
Borges. Como no poda ser de otro modo, el semilogo Eco,
ahora puesto en novelista, hace recaer la solucin del
enigma y la resolucin del caso sobre un detalle lingstico,
la suppositio materialis. Para ello, se trata de develar el
sentido de la frase que permitira abrir una puerta de la
biblioteca laberntica y encontrar la clave para ubicar al
asesino.

La frase dice:
Secretum finis Africae manus supra idolum age
primum et septimum de quatuor.[7] Segn una
primera interpretacin, la traduccin sera "el secreto
del finis Africae consiste en que la mano sobre el dolo
opera sobre el primero y el sptimo de los cuatro".
Ahora el mensaje no tiene sentido ya que no existe el
sptimo entre cuatro cosas.

a. Cul podra ser el sentido del


mensaje?
b. Dnde oprimira para abrir la

puerta?
c. Cmo formulara el enigma para que
fuese fcilmente comprensible?
Ayuda: La puerta est oculta tras un
espejo (dolo-imagen). Sobre la puerta
finis Africae hay una leyenda que
reproduce el versculo 4.4 del
Apocalipsis de San Juan. "Super
thronos viginti quatuor", y la palabra
"quatuor" tiene siete letras.

1.2. La navaja de Ockham y


el Dr. House
El siguiente texto presenta un episodio
de la serie de televisin Dr. House.
Lalo atentamente y luego responda las

preguntas que estn a continuacin:


La serie norteamericana de televisin Dr. House
muestra a un mdico misgino y adicto al consumo de
frmacos que, con una mente deductiva, resuelve los
casos problemticos al modo de un detective o un
semilogo: interpretando signos. Como en las novelas
de Sherlock Holmes, el Dr. House interroga a los
sntomas, como si fueran las pistas que conducen hasta
hallar al culpable: la causa de la enfermedad. Las
analogas entre Sherlock Holmes y el Dr. House son
evidentes, incluso el Dr. Wilson representa una versin
del Dr. Watson. Al igual que Holmes, House es un
solitario, se relaciona de modo conflictivo con las
dems personas y llega a ser incluso cortante y
ofensivo con sus pacientes. Lo nico que le interesa es
"resolver el caso" y todas sus emociones se concentran
en ese fin. En el captulo 3 de la primera temporada de
la serie, titulado originalmente "Occam's Razor",
traducido al espaol como "Principio de parsimonia", se
muestra el caso de Brandon, un chico de 22 aos que
ha sufrido un colapso y es ingresado al hospital. En los
ltimos das ha estado tosiendo y tiene un sarpullido.
Ahora siente un dolor fuerte en el abdomen, nuseas,

fiebre y su presin arterial es baja. Un escner rpido


y un examen no revelan nada, as que House y su
equipo buscan alternativas. Coinciden en que ninguna
enfermedad se corresponde con los siete sntomas que
presenta el paciente. La teora de Foreman, uno de los
mdicos del equipo que dirige House, es que el chico
tiene una endocarditis, lo que explicara varios de los
sntomas, excepto dos: la tos y el sarpullido. An as, es
una posibilidad entre un milln. House examina la lista
de sntomas del paciente y sugiere que dos
enfermedades coinciden con los sntomas de Brandon,
pero sigue siendo una posibilidad entre un milln.
House apuesta 50 dlares con Foreman a que si el
recuento de leucocitos del paciente aumenta, estar en
lo cierto al decir que el chico tiene una infeccin.
Cuando el nmero de glbulos blancos de Brandon
disminuye, los dos mdicos se dan cuenta de que sus
hiptesis eran errneas. House tiene una revelacin y
le pregunta al doctor Wilson cul fue el primer sntoma
de Brandon y sus sospechas se confirman: fue la tos.
House tiene entonces otra revelacin, irrumpe en la
habitacin del paciente y anuncia su diagnstico:
intoxicacin por colchicina, sustancia con la que se

combate la "gota", una enfermedad que Brandon no


sufre ni toma medicacin contra ella. Ahora el
problema es saber: cmo ingiri la colchicina?, quin
le administr esa sustancia?
El enigma se resuelve cuando la madre de Brandon
confiesa que le administr pastillas para la tos, sin
consultar a los mdicos. Primero Foreman y luego
House afirman en momentos cruciales: "Principio de
parsimonia, la mejor explicacin es la ms simple".

a. En qu sentido se alude aqu a la


navaja de Ockham?
b. Qu enuncia, en su versin ms
conocida, este principio?
c. Una navaja se usa, normalmente, para
cortar o rasurar la barba. Investigue a la
barba de quin alude el nombre de
"navaja de Ockham" y cul sera la
analoga que propone la posicin
nominalista y el hecho de cortar las
barbas.

ACTIVIDAD 2. USO Y MENCIN


DEL LENGUAJE
Lea los siguientes versos y luego
conteste:
Si, (como afirma el griego en el Cratilo)
El nombre es arquetipo de la cosa,
En las letras de "rosa" est la rosa
Y todo el Nilo en la palabra "Nilo".
(Jorge Luis Borges, "El Golem")[8]

a. A qu griego se refiere Borges y


cmo interpreta la frase "el nombre es
arquetipo de la cosa"?
b. Tomando en cuenta la distincin entre
uso y mencin del lenguaje, justifique
por qu las palabras "rosa" y "Nilo"
aparecen en un caso escritas con
comillas y en otro sin comillas?
c. En este prrafo: Borges afirma que
en las letras de "rosa" est la rosa y todo

el Nilo en la palabra "Nilo"?


ACTIVIDAD 3. SEMITICA. ACERCA
DE LOS SIGNOS.
En esta actividad se trata de reconocer
distintos tipos de signos segn una
clasificacin cannica, para llegar al
concepto de "smbolo" que es
mencionado como parte de la definicin
del lenguaje, tema central en esta
unidad. Que el lenguaje sea definido
como un "conjunto reglado de smbolos"
implica, bsicamente, que no hay ningn
vnculo natural ni necesario entre el
nombre y el concepto u objeto
nombrado, sino que este vnculo es
arbitrario y convencional.
Dice Charles Morris: "Los signos y la
ciencia estn inextricablemente

conectados, habida cuenta de que la


ciencia, simultneamente, ofrece a los
hombres signos ms fiables y expresa
sus resultados en sistemas de signos.
La civilizacin humana depende de los
signos y de los sistemas de signos, y al
propio tiempo la mente humana es
inseparable del funcionamiento de los
signos, si es que, en verdad, la
mentalidad misma no debe
identificarse con ese
funcionamiento." [9]
Charles Morris llama "semiosis" al
proceso por el cual algo funciona como
signo. Es necesario que concurran tres
factores para que algo funcione como
signo:
El vehculo sgnico, la seal, fenmeno

o cosa que acta como signo (S).


El designatum, el significado del signo
(D).
El intrprete (I).
Ejemplo: Un perro (I) responde al
sonido del silbato (S) que implica la
caza de ardillas (D).
Le proponemos que en cada uno de los
siguientes ejemplos identifique al signo
(S), al designatum (D) y al intrprete (I),
y coloque (S), (D) (I) en los
parntesis, segn corresponda. Indique
de qu clase de signo se trata.
a. Al ver la sea del polica ( ), un
conductor ( ) frena el auto ( ).
b. Un peatn ( ) cruza la calle ( ) al ver
el color de la luz del semforo ( ).
c. Al ver una nube de humo negro que

sale de una ventana del primer piso ( ),


los vecinos de un edificio ( ) salen de
sus departamentos corriendo ( ).
d. La fiebre del nene ( ) lleva al mdico
( ) al diagnstico de una hepatitis ( ).
ACTIVIDAD 4. SEMITICA. ACERCA
DE LOS TRMINOS.
Uno de los temas centrales de esta
unidad es la distincin entre designacin
y denotacin. En el siguiente prrafo de
Alicia en el pas de las maravillas,
Lewis Carroll ilustra la idea de que los
atributos no pueden andar solos (no
tienen denotacin), es decir, la
imposibilidad de concebir la
subsistencia de un atributo luego de la
desaparicin de la cosa.

Dale!", dijo el Gato, y esta vez se


desvaneci muy paulatinamente,
empezando por la punta de la cola y
terminando por la sonrisa, que permaneci flotando en el aire
un rato despus de haber desaparecido
todo el resto.
"Bueno!Muchas veces he visto a un
gato sin sonrisa", pens Alicia,
"pero una sonrisa sin gato!... Esto es
lo ms raro que he visto en toda mi
vida!"
(Carroll, L. 1990)

La posibilidad de poder definir trminos


sin denotacin habilita, en el campo de
las ciencias formales, la posibilidad de
armar sistemas en los que tengamos
trminos sin denotacin, es decir, sin

ninguna representacin en el mundo


emprico (como la sonrisa sin gato),
pero que con ellos podemos construir
sistemas en la geometra, en la
aritmtica o en la lgica. En el captulo
segundo de la obra Symbolic Logic
(1892), Carroll escribi que entre las
clases de cosas estn las cosas
imposibles y dio, como ejemplo, el caso
de las cosas que pesan ms de una
tonelada y un nio puede levantar.
Advierta aqu que el autor sustancializa
el lenguaje al hablar de cosas
imposibles. Lo correcto sera decir
"trminos que nombran cosas
imposibles". Suponemos que lo hace
para crear perplejidad ya que en el uso
comn del lenguaje sustancializamos los

conceptos. Estos trminos que nombran


cosas imposibles seran los trminos sin
denotacin de las ciencias formales
(nmeros, tringulos, lneas, etc.), entre
otros casos.
Hemos tomando como ejemplo el caso
imposible de una "sonrisa sin gato". Le
pedimos, ahora, que:
a. Mencione al menos tres ejemplos de
trminos que nombren atributos que no
tienen denotacin. Justifique.
b. Proponga otros tres ejemplos de
trminos que tengan designacin en las
ciencias formales y justifique por qu
puede afirmarse que no tienen
denotacin.
ACTIVIDAD 5. NOMBRAR Y
CLASIFICAR. VAGUEDAD Y

AMBIGEDAD.
La vaguedad (significado impreciso de
un trmino) y ambigedad (ms de un
significado para el mismo trmino) son
algunos de los temas tratados por la
dimensin semntica de la semitica.
Luego de leer el siguiente prrafo del
libro Alicia en el pas de las
maravillas, determine si el trmino
"pican" es un caso de vaguedad o
ambigedad.
S eguro

que estars pensando que por


qu no te paso el brazo por la cintura",
dijo la Duquesa tras una pausa; "la
razn es que tengo mis dudas sobre el
humor de ese pjaro flamenco que
llevas ah. Qute parece siprobamos

el experimento?"
'A lo mejor le da unpicotazo", replic
Alicia con cautela, sin ningutaagataa
de intentar el experimento.
"Muy cierto ", concedi la duquesa;
"los flamencos y la mostaza, ambos
pican; y la moraleja de esto es.... Dios
los cra y ellos se juntan'".

(Carroll, L. 1990)

ACTIVIDAD 6. LA DEFINICIN
En esta actividad se busca llamar la
atencin y trabajar las nociones de
"definiendum" y "definiens" para
advertir que un mismo trmino puede ser
parte de un definiendum o de un
definiens.
Elabore definiciones para los siguientes
trminos, haciendo corresponder al
definiendum un gnero y una diferencia
apropiados.
Definiendum
Definiens
Gnero Diferencia
a.- soltero
b.- banquete

a.- vstago a.- hembra


b.- caballo b.- macho

c.- muchacho c.- hombre c.- casado


d.- hermano d.- comida d.- no casado
e.- nio

e.e.- muy
progenitor grande

f.- potro
f.- oveja
g.- hija
h.- oveja

g.hermano

i.- padre

h.- mujer

j.- gigante
k.- muchacha
l.- marido

f.- muy
pequeo
g.- joven

ll.- cordero
m.- yegua
n.- enanito
.- madre
o.- pony
p.- carnero
q.- hermana
r.- porcin
s.- hijo
t.- soltero

ACTIVIDAD 7. DEFINICIN.
DESIGNACIN Y EXTENSIN.
Entre designacin y extensin hay una
relacin inversa: a mayor designacin
(notas definitorias), menor extensin
(cantidad de ejemplares). "Hombre"
tiene ms designacin que "animal" pero
hay menos ejemplares de hombres que
de animales. Dicho de otro modo: la
especie tiene mayor designacin que el
gnero y menor extensin.
Le proponemos que ordene los
siguientes trminos segn la mayor
cantidad de ejemplares (extensin):
a. Argentino, hombre, americano,

misionero.
b. Fsico, hombre, cientfico, ser vivo.
c. Hombre, futbolista, deportista, Martn
Palermo.

Temas de la Unidad
2.1. Las leyes lgicas
2.2. Tautologas, contradicciones,
contingencias
2.3. Los razonamientos. Verdad y
validez
2.4. Argumento y consecuencia lgica.
Reglas lgicas
2.5. La induccin y la analoga
2.6. Es formal la induccin? El intento
de Carnap
2.7. Lgica informal y falacias
materiales
2.8. Toulmin, sobre los usos

argumentativos
Bibliografa obligatoria
Unidad 2 en la Gua de Estudio de IPC,
producida por UBA XXI y editada por
Eudeba, a partir de la edicin de 2010.
Asti Vera, C. y Ambrosini, C. (2009).
Captulos 2 y 3. En Argumentos y
teoras. Aproximacin a la
Epistemologa. Buenos Aires:
Educando.
BIBLIOGRAFA COMPLEMENTARIA
Copi, I. (1984). Captulos 1 y 3. En
Introduccin a la lgica. Buenos Aires:
Eudeba.
Dez, J. A. y Moulines, C. U. (1999).
Captulo 2. En Fundamentos de
filosofa de la ciencia. Barcelona:
Ariel.

Bunge, M. (1997). Captulo 3. En


Epistemologa. Mxico: Siglo XXI.
Klimovsky, G. (1994). Captulo 18. En
Las desventuras del
conocimientocientfico. Buenos Aires:
AZ.
Presentacin de la Unidad
" Pero es que a m me gusta estar entre
locos", observ Alicia.
"Eso s que no lo puedes evitar",
repuso el Gato; "todos estamos locos
por aqu. Yo estoy loco; t tambin lo
ests".
"Y cmo sabes t si yo estoy loca?", le
pregunt Alicia.
"Has de estarlo a la fuerza", le
contest el Gato; "de lo contrario no

habras venido aqu".


Alicia pens que eso no probaba nada;
pero continu de todas formas: "Y
cmo sabes que t ests loco?"
"Para empezar", repuso el Gato, "los
perros no estn locos, de acuerdo?''
"Supongo que no", dijo Alicia.
"Bueno, pues entonces", continu
diciendo el Gato, "vers que los perros
gruen cuando algo no les gusta, y
mueven la cola cuando estn contentos.
En cambio, yo gruo cuando estoy
contento y muevo la cola cuando me
enojo; luego estoy loco."
(Carroll, L. Alicia en el pas de las
maravillas)

Esta unidad est reservada a una


revisin de la tercera estructura lgica:
el "razonamiento". En la primera unidad
ya hemos visto la primera estructura

lgica: los "trminos" y la segunda: las


"proposiciones". Aqu estudiaremos el
escenario formal y el escenario
informal. En el primero, encontraremos
los principios lgicos como paso
necesario para introducir los conceptos
de tautologas, contradicciones y
contingencias que aluden a tipos
distintos de proposiciones. Esta
distincin entre proposiciones apunta a
deslindar firmemente entre ciencias
formales y fcticas dado el distinto tipo
de enunciados que involucran sus
teoras. Es importante reconocer la
pertinencia del uso de los conceptos de
verdad y validez que tambin apuntan a
mostrar una distincin entre
proposiciones y razonamientos.

Es muy importante reconocer las reglas


lgicas y el concepto de razonamiento
deductivo.
A continuacin, estudiaremos la
propuesta del filsofo alemn Rudolf
Carnap (1891-1970) y su esfuerzo por
dotar a la induccin (una forma de
razonamiento invlido desde el punto de
vista deductivo) de la rigurosidad
propia de los procedimientos formales
para establecer una "lgica inductiva".
Tambin veremos el razonamiento
analgico como un tipo de razonamiento
no deductivo. Este tipo de razonamiento
es un recurso argumentativo muy usado
en el campo de la ciencia.
En el escenario informal veremos
diferentes manifestaciones de lo que se

llama "lgica informal" o "teoras de la


argumentacin". Para ello estudiaremos,
en principio, distintos tipos de falacias
materiales, donde se distingue entre dos
grupos con caractersticas particulares:
a) falacias de inatinencia (o de
inatingencia) y b) falacias de
ambigedad. Es importante la revisin
de este tipo de argumentaciones falaces
ya que es muy provechoso reconocerlas
en el lenguaje social, para no ser
vctimas de manipulaciones retricas
como as tambin para detectarlas en las
argumentaciones cientficas.
Esta unidad se completa con uno de los
aportes relevantes en el terreno de las
teoras de la argumentacin, surgidas a
partir de la segunda mitad del siglo XX:

la teora de los usos argumentativos del


filsofo estadounidense Stephen
Toulmin (1922-2009). Aqu es
importante que comprenda la distancia
que toma Toulmin del planteo de la
Lgica formal cuando parte de una
analoga jurdica: los argumentos son
comparables a las demandas judiciales y
esta lgica que apunta a la "prctica"
argumentativa sera una suerte de
"jurisprudencia generalizada". El
esquema argumentativo bsico distingue
entre D (datos), C (conclusiones) y G
(garantas). En el ejemplo del libro de la
bibliografa, se muestra el esquema
segn el cual la afirmacin "Juan Carlos
S. naci en Salta, que es una provincia
argentina" (D), permite inferir la

conclusin (C) "Juan Carlos S. es


ciudadano argentino", a partir de la
garanta (G) "Si una persona nace en una
provincia de la Argentina, es argentina".
En un esquema posterior, se agregan
elementos al anlisis del argumento con
los conceptos de "modalizador" (M), de
la fuerza de la conclusin
(presuntamente, probablemente) y las
condiciones de excepcin o refutacin
(E) donde pueden presentarse casos de
excepcin ("a menos que haya sido
naturalizado espaol"). Una nocin
central que debe ser tomada en cuenta
para comprender la propuesta de
Toulmin es la de "campo argumentativo"
ya que esta nocin permite deslindar
entre "argumentos analticos", llamados

tambin "tericos", que seran


independientes del contexto y
asimilables a los razonamientos
deductivos de la lgica formal, y los
"argumentos sustanciales", tambin
llamados "prcticos", que seran
dependientes del contexto y de
importante aplicacin en el mundo
prctico.
Debemos advertir que la presentacin
del escenario informal se completa, en
el texto de la bibliografa, con otras dos
propuestas destacadas como son la del
filsofo del derecho belga Cham
Perelman (1912-1984), y la de los
pensadores contemporneos holandeses
Frans Van Eemeren y Rob Grootendorst
que en este programa no se incluyen

como lectura obligatoria ni forman parte


del material de lectura obligatoria pero
que, de todos modos, y ms all de las
exigencias temticas del curso, conviene
leer y confrontar con la propuesta de
Toulmin para completar la comprensin
de las diferencias entre el escenario
formal y el escenario informal.

Actividades de
aprendizaje
ACTIVIDAD 1. ACERCA DE LA
ESTRUCTURA DE LOS
RAZONAMIENTOS
El razonamiento es la estructura lgica

que se compone de premisas y


conclusin en las que, dada una serie de
enunciados que actan como premisas,
se infiere una conclusin. Los
razonamientos pueden ser "vlidos" o
"invlidos". En el libro Alicia en el pas
de las maravillas, el gato de Cheshire
irrumpe en las escenas de modo
inesperado. En el prrafo citado en la
presentacin de esta unidad, que aqu
transcribimos, se argumenta acerca de la
inevitabilidad de la locura en el pas
donde se encuentra Alicia y para ello
recurre a un extrao razonamiento.
A. Lea atentamente el razonamiento.
B. Identifique las premisas y la
conclusin.
C. Determine si este razonamiento es

vlido o invlido y justifique.

pero es que a m no me gusta estar


entre locos", observ Alicia.
"Eso s que no lo puedes evitar",
repuso el Gato; "todos estamos locos
por aqu. Yo estoy loco; t tambin lo
ests".
"Y cmo sabes t si yo estoy loca?", le
pregunt Alicia.
"Has de estarlo a la fuerza", le
contest el Gato; "de lo contrario no
habras venido aqu".
Alicia pens que eso no probaba nada;
pero continu de todas formas: "Y
cmo sabes que t ests loco?"
"Para empezar", repuso el Gato,
"losperros no estn locos, de

acuerdo?"
"Supongo que no", dijo Alicia.
"Bueno, pues entonces", continu
diciendo el Gato, "vers que los perros
gruen cuando algo no les gusta, y
mueven la cola cuando estn contentos.
En cambio,yo gruo cuando estoy
contento y muevo la cola cuando me
enojo; luego estoy loco. "
(Carroll, L., 1990)

ACTIVIDAD 2. ACERCA DE LOS


CONCEPTOS DE VERDAD Y VALIDEZ
Como afirmamos en la bibliografa de
esta unidad, predicamos la verdad o

falsedad de las proposiciones y la


validez o invalidez de los
razonamientos. A menudo, en el lenguaje
comn, usamos estos conceptos de un
modo impreciso y mezclado. Es
necesario reconocer la pertinencia del
uso de estos dos conceptos para
reconocer dos estructuras lgicas
distintas: las proposiciones y los
razonamientos.
Complete las siguientes expresiones de
modo que se conviertan en enunciados
verdaderos:
a. Si un enunciado tiene premisas falsas
y conclusin verdadera, el razonamiento
puede ser
..................................................................
b. Si un razonamiento es vlido y tiene

premisas falsas, su conclusin puede ser


..................................................................
c. Si un razonamiento tiene premisas
verdaderas y conclusin verdadera, su
forma puede ser
..................................................................
d. Si un razonamiento tiene premisas
falsas y conclusin falsa, su forma puede
ser
..................................................................
e. Si un razonamiento tiene premisas
verdaderas y conclusin falsa, su forma
es
..................................................................
f. Para obtener una conclusin verdadera
se requiere que el razonamiento
sea....................................... y las
premisas......................................

ACTIVIDAD 3. REGLAS LGICAS Y


FALACIAS
3.1. En los cuatro razonamientos
siguientes, determine qu formas son
deductivas (poner el nombre a la regla)
y cules son falacias.
a. Si sumerjo un cubito de hielo en un
vaso de agua caliente, entonces el cubito
se derrite. Sumerg el cubito, por lo
tanto, se derriti.
b. Si sumerjo un cubito de hielo en un
vaso de agua caliente, entonces el cubito
se derrite. No lo sumerg, por lo tanto,
no se derrite.
c. Si sumerjo un cubito de hielo en un
vaso de agua caliente, entonces el cubito
se derrite. El cubito no se derrite, por lo
tanto, no lo sumerg.

d. Si sumerjo un cubito de hielo en un


vaso de agua caliente, entonces el cubito
se derrite. El cubito se derrite, por lo
tanto, lo sumerg.
3.2. En los dos razonamientos
presentados a continuacin hay uno
deductivo y otro invlido, a pesar de
que ambos concluyen en el mismo
enunciado. Identifique cada caso y
explique la diferencia entre ambos.
a.
Si Juan no repite el ao, entonces se va
a Bariloche.
Juan se va a Bariloche.
Juan no repiti el ao.

b.
Si Juan repite el ao, entonces no va a
Bariloche.
Juan fue a Bariloche (no es cierto que no
va).
Juan no repiti el ao.
3.3. Reglas lgicas y falacias formales
Empleando el enunciado: "Si me anoto
en el Ciclo Bsico Comn, entonces
curso el primer ao de mi carrera", qu
conclusin se obtiene construyendo:
a. un Modus ponens,
b. un Modus tollens,
c. una falacia de afirmacin del
consecuente y

d. una falacia de negacin del


antecedente.
3.4. Conclusin correcta de decir "Si p
entonces q"
a. Si alguien promete a Juan: "Si
termins el CBC este ao, te llevo de
viaje a Europa" y, luego, lo lleva a
Europa a fin de ao, aunque no haya
terminado el CBC en un ao, habr
mentido? Explique por qu.
b. Si no termina el CBC en un ao y no
lo lleva a Europa, habr mentido?
Explique por qu.
c. Si termina el CBC en un ao y no lo
lleva a Europa, habr mentido?
Explique por qu.
3.5. Diferencias entre decir "si p
entonces q" a decir "si y slo si p

entonces q"
Qu consecuencias distintas podran
tener las siguientes afirmaciones?
a. "Si me recibo de contador este ao,
me caso".
b. "Si y slo si me recibo de contador
este ao, me caso".
ACTIVIDAD 4. LA INDUCCIN
4.1. Suponiendo que las siguientes
proposiciones son las conclusiones de
distintos razonamientos inductivos, en
cada caso determine cules podran ser
las premisas.
a. "Los planetas se mueven alrededor
del sol".
b. "Todas las modelos se casan con
jugadores de ftbol".
c. "Todos los mamferos tienen pelos".

d. "Todas las aves tienen plumas".


e. "Los argentinos toman mate".
4.2. Dados los siguientes enunciados,
consideradas las premisas de algn
razonamiento inductivo, proponga la
conclusin.
a. "Las peras, manzanas, bananas y
frutillas sirven para hacer un postre".
b. "Los nmeros 2, 4, 10 y 22 son
divisibles por dos".
c. "El mejilln, las almejas, las ostras,
los pulpos, los calamares y las sepias
viven en el mar".
d. "El Aedes aegypty y el Anopheles
transmiten enfermedades".
e. "Las ratas, los conejos y los ratones
son vivparos".
f. "Los peruanos, los bolivianos y los

coreanos venden ropa en la feria de La


Salada".
4.3. Luego de leer los tres textos de
Aristteles citados, determine de qu
tipo de argumento se trata en cada caso.
Deca Aristteles:
a. "Si el mejor de los pilotos es el ms
diestro y el mejor de los aurigas es
tambin el ms diestro, podemos
afirmar, en general, que en cada
profesin el mejor es el ms diestro".
(Tpicos, I, 12)
b. "Los animales sin hiel son de larga
vida; el hombre, el caballo, el mulo,
son todos animales sin hiel; por lo
tanto, todos los animales sin hiel son
de larga vida". (Primeros Analticos, II,
23)

c. "Todo ocurre segn una finalidad,


todo movimiento tiende a un fin, a
veces ese fin se consigue y otras veces
no. En el arte, el escriba, a veces,
comete faltas en la escritura y el
mdico, a veces, administra
equivocadamente un remedio. As
ocurre tambin que la naturaleza,
cuando yerra la generacin, en vez de
reproducir el tipo del engendrador, se
desva de l y nace un monstruo
(trata). El monstruo es un ser
inacabado, informe, mal logrado
(anaperon). Cuando esto ocurre, la
simiente traiciona la forma
desprendida del esperma del
engendrador, que no logr dominar
perfectamente la materia suministrada

por la hembra." (Fsica, II, 8)


ACTIVIDAD 5. LA ANALOGA
La analoga es un modo de razonar que,
a partir de la comparacin entre dos o
ms casos que tienen notas comunes,
permite concluir para alguno de esos
casos, algo admitido nicamente en
alguno de los otros casos. Al igual que
la induccin, es una forma de
razonamiento invlido aunque en muchos
casos puede resultar de inters
pragmtico.
Dados los siguientes casos, reconstruya
un razonamiento por analoga
determinando cul es la conclusin y
cules son las premisas.
a. "Ludwig Wittgenstein (Viena,
Austria, 1889 - Cambridge, Reino

Unido, 1951) sola comparar el pensar


con la natacin. As como en sta
nuestros cuerpos tienen una tendencia
natural a flotar sobre la superficie del
agua, de modo que se necesita un gran
esfuerzo fsico para sumergirse en el
fondo, de igual modo en el pensar se
necesita un gran esfuerzo mental para
alejar a nuestras mentes de lo
superficial." [10]
b. "Uno de los atributos ms naturales
de la mujer es el cuidado de los nios.
De hecho, es correcto afirmar que los
grupos en los que los hombres, y no las
mujeres, cran a los hijos pequeos son
totalmente excepcionales. Puesto que
el enfermo y el discapacitado se
asemejan en muchos aspectos a los

nios, pues no solamente son


fsicamente dbiles y desvalidos, sino
tambin psicolgicamente dependientes
y narcissticamente regresivos, era
bastante fcil suponer que las mujeres
estn especialmente capacitadas
tambin para el cuidado del
enfermo." [11]
ACTIVIDAD 6. LGICA INFORMAL Y
FALACIAS MATERIALES
En el diario La Vanguardia de
Barcelona, el 4 marzo de 2010 aparece
la siguiente noticia. Histricamente la
comunidad de Madrid y la de Barcelona
son antagonistas en muchos sentidos. En
Catalunya, el Parlament est tratando la
prohibicin de la corrida de toros a
partir de los reclamos de las sociedades

defensoras de los derechos de los


animales, mientras que en Madrid hay
una gran resistencia ya que representa un
punto de inters tambin turstico.
Veamos el argumento de la presidenta
de la comunidad de Madrid en defensa
de las corridas de toros.
A. Lea atentamente la noticia.
B. Identifique el argumento de
Esperanza Aguirre.
C. Determine si incurre en alguna
falacia. Justifique.
Los toros, a debate

AGUIRRE:

"LOS TOROS SON


UN ARTE Y MERECEN ESTAR

BIEN PROTEGIDOS"
La presidenta de la comunidad de
Madrid justifica as la intencin de su
gobierno de declarar los toros Bien
de Inters Cultural, que coincide con
el debate de prohibicin que se est
haciendo en el Parlament de
Catalunya El consejero de Cultura de
la Comunidad, Ignacio Gonzlez,
reconoce que "es evidente que tiene
mucha relacin" con el debate que se
est produciendo en Catalunya.
Madrid (EFE).- La presidenta de la
Comunidad de Madrid, Esperanza

Aguirre, ha defendido que "los toros


son un arte" y que "merece la pena
declararlos Bien de Inters Cultural
para que estn bien protegidos".
Aguirre ha hecho estas declaraciones
en los pasillos de la Asamblea,
despus de que su Gobierno haya
acordado iniciar el expediente para
declarar la Fiesta de los toros Bien
de Inters Cultural (BIC). "A nadie le
puede extraar, porque la cultura
taurina es algo que desde tiempo
inmemorial est en la cultura
espaola y en la cultura
mediterrnea", ha afirmado la
presidenta madrilea.
A su juicio, "no hay ms que ver que,
desde Goya, Picasso o Garca Lorca

o, fuera de nuestras fronteras,


Hemingway y Orson Welles, todos se
han ocupado de los toros como arte".
"Qu duda cabe que es un bien
cultural a proteger y, como dice
nuestra ley, no solamente los
monumentos o museos deben
protegerse, tambin los bienes
inmateriales", ha argumentado. [...]
Aguirre: "Los toros son un arte y
merecen estar bien protegidos".
(2010, marzo 4). La Vanguardia.
Disponible en:
<http://www.lavanguardia.es/ciudadan
53896639502/aguirre-los-toros-sonun-arte-y-merecen-estar-bienprotegidos-madrid-esperanzaaguirre-ignacio-gonza.html>

ACTIVIDAD 7. EL CRCULO DE
VIENA
Rudolf Carnap fue uno de los miembros
destacados del Crculo de Viena
(Wiener Kreis) y director de la Revista
Erkenntnis donde se publicaron los
principales aportes de los integrantes de
este crculo de cientficos y
epistemlogos destacados.
Dada la siguiente lista de nombres de
personalidades del siglo XX, investigue
acerca de sus trayectorias y luego
determine qu rol cumplieron en
relacin con el Crculo de Viena. Moritz
Schlick, Ludwig Wittgenstein, Otto
Neurath, Otto Weininger, Sigmund
Freud, Hans Reichenbach, Adolf Hitler,
Kurt Gdel, Bertrand Russell, Carl

Hempel, Alfred Einstein, Alfred Tarski,


Karl Popper, A. J. Ayer.
ACTIVIDAD 8. FALACIAS
MATERIALES. EL ESCENARIO
INFORMAL
Identifique la falacia en la que se incurre
en los siguientes argumentos:
a. "Te dije el ao pasado que si te
casabas con tu novio, al ao estaras
separada. Ahora ests separada y es por
lo que te dije."
b. "Las mujeres no tienen alma. Los
chinos, desde la ms remota antigedad,
han negado que la mujer tenga alma."[12]
c. "Usted no tiene derecho a tomar la
palabra porque es funcionario de un
gobierno corrupto."
d. "No puede hacerme la boleta por

exceso de velocidad porque, si tengo


que pagar la boleta, pierdo la ganancia
del da y no tengo para darles de comer
a mis hijos."
e. "Todo abogado tiene derecho a la
libertad de consultar libros cuando
busca fundamentar mejor una defensa y
todo mdico tiene derecho a consultar
sus libros cuando quiere curar mejor a
un enfermo. Del mismo modo, debe
permitirse a los alumnos ejercer la
libertad de consultar sus libros para
contestar mejor en un parcial."
f. "Las esposas de los hombres exitosos
usan ropa cara, de modo que la mejor
manera que tiene una mujer de ayudar a
su marido en los negocios es gastar
mucha plata en ropa."

g. "Seor Gerente, mi marido merece


que le aumente el sueldo porque lo que
gana apenas alcanza para alimentar a los
nios."
h. "En los ltimos 20 aos ha aumentado
la asistencia de adolescentes en la
Universidad de Buenos Aires y tambin
ha aumentado la delincuencia juvenil.
Por lo tanto, para combatir el delito, es
necesario limitar el ingreso de jvenes a
la Universidad de Buenos Aires."
i. "Est comprobado que los seres
extraterrestres ayudaron a construir las
pirmides de Egipto porque hasta ahora
nadie ha podido demostrar que ellos no
intervinieron."
j. "Si un automvil funciona, entonces
tiene nafta en el tanque. Pero ahora el

auto no funciona, eso quiere decir que


no tiene nafta en el tanque." k. "Si
Marilyn Monroe fue asesinada, entonces
est muerta y, efectivamente, est
muerta. Eso quiere decir que fue
asesinada." l. "Si el amor es ciego y
Dios es amor, entonces, Dios es ciego."
ACTIVIDAD 9. TOULMIN. MAPAS
ARGUMENTATIVOS
Stephen Toulmin afirm que las
argumentaciones cotidianas no siguen el
clsico modelo riguroso del silogismo.
Para l, en una argumentacin directa, un
"sujeto argumentador" presenta
explcitamente una "tesis" u opinin y
expone una serie de argumentos o
"razones lgicas" que deben desembocar
en una "conclusin" que confirma la

tesis propuesta.

Una exclamacin como "Tienes una


infeccin y por tanto debes tomar
penicilina" es un argumento que parte de
una dato ("tienes una infeccin") y
postula una conclusin ("debes tomar
penicilina"), pero su credibilidad
depende de una garanta (la penicilina
cura infecciones, por ejemplo) que a su
vez se basa en un respaldo (pruebas de
laboratorio, experiencia mdica, etc.),
condicin que hace posible el uso del

modalizador ("por lo tanto"). Y el


argumento puede tener tambin una
refutacin ("a menos que seas alrgico"
podra ser una). Todos los argumentos
parten de un esquema lgico de este
tipo, en el que la capacidad persuasiva
depende de las premisas en que se funda
y de su capacidad de vincularse a las
creencias previas del pblico. Por ello,
una misma conclusin puede ser
defendida o refutada desde diversos
puntos de partida. El diagrama de
Toulmin puede ayudarnos a
esquematizar esta actividad en el
momento de organizar la comunicacin.
Por ejemplo, yo puedo exhortar a
alguien a adquirir un libro con distintos
argumentos como:

"Este libro es barato, adquirelo."


(Basado en una premisa cuantitativa)
"Este libro est muy bien escrito,
adquirelo." (Basado en una premisa
cualitativa)
"Este libro no volvers a verlo,
adquirelo." (Basado en una premisa
que remite al valor de lo nico)
"Si no compras el libro, te
reprueban." (Basado en una premisa
que remite al criterio del mal menor)
[13]

9.1. Tomando lo estudiado en la unidad,


conteste:
a. Segn Toulmin, en qu se diferencian
los argumentos deductivos de la lgica
formal y los argumentos prcticos de la

vida social?
b. Por qu la lgica prctica que
propone Toulmin incluye el concepto de
campos argumentativos? Qu crticas
recibe su teora?
9.2. Seale con una (X) cules de las
siguientes caractersticas corresponden
a los argumentos prcticos de la vida
social segn Toulmin:
a. Son analticos.
b. Son independientes del contexto.
c. Dependen del contexto.
d. Fundamentan la conclusin de un
modo absoluto.
e. Fundamentan la conclusin de un
modo probabilstico.

Temas de la Unidad
3.1. La matemtica: constructos formales
y realidad
3.2. Sistemas axiomticos
3.3. Propiedades de los sistemas
axiomticos
3.4. Interpretacin y modelos de los
sistemas axiomticos
Bibliografa obligatoria
Unidad 3 en la Gua de Estudio de IPC,
producida por UBA XXI y editada por
Eudeba, a partir de la edicin de 2010.
Asti Vera, C. y Ambrosini, C. (2009).
Captulo 4. En Argumentos y teoras.
Aproximacin a la Epistemologa.

Buenos Aires: Educando.


BIBLIOGRAFA COMPLEMENTARIA
Dez, J. A. y Moulines, C. U. (1999).
Captulos 4 y 5. En Fundamentos de
filosofa de la ciencia. Barcelona:
Ariel.
Klimovsky, G. (1994). Captulos 2, 3, 4
y 5. En Las desventuras del
conocimiento cientfico. Buenos Aires:
AZ.
Presentacin de la Unidad

Te apetece unpoco de vino? ", insinu


meliflua la Liebre de Marzo. Alicia
mir por toda la mesa sin ver ms que
t, por lo qt observ: "No veo ese vino
por ninguna parte. "
"No lo hay ", replic enseguida la

Liebre de Marzo.
"Entonces, no ha sido nada amable
elofrecrmelo ", dijo Alicia enojada
"Tampoco lo ha sido el sentarse a esta
mesa sin haber sido invitada " repuso
la Liebre.
(Carroll, L. Alicia en el pas de las
maravillas)

De acuerdo con el cuadro de


clasificacin de las ciencias presentado
en la Unidad 1, esta unidad est
destinada a completar las caractersticas
sealadas all como propias de las
ciencias formales. Es decir,
estudiaremos el "mtodo axiomtico" y
los conceptos que incluye su

formulacin. En este punto conviene


detenerse y prestar atencin a la
distincin entre la concepcin clsica de
las ciencias formales, identificada con
Aristteles y la geometra de Euclides
en el siglo III a.C. y la ruptura que se
produjo en este terreno a partir del
cuestionamiento del V Postulado de
Euclides, conocido como el Postulado
de las paralelas, a mediados del siglo
XIX de nuestra era. Para comprender
mejor esta ruptura y cambio radical de
teoras en el campo de las ciencias
formales, conviene que considere los
principales aportes a las geometras no
euclidianas, sealadas en este captulo.
Para ejemplificar los componentes de un
sistema axiomtico, donde como en todo

lenguaje, desde el punto de vista


sintctico, encontramos trminos
(definidos y no definidos),
proposiciones (demostrables: teoremas
y no demostrables: axiomas) y
razonamientos (reglas deductivas),
deber detenerse en la presentacin de
estos conceptos a partir de la Aritmtica
de Peano (matemtico y filsofo
italiano, 1858-1932). En la revisin de
las propiedades de los sistemas
axiomticos, es necesario advertir la
importancia del aporte del lgico,
matemtico y filsofo estadounidense
Kurt Gdel (1906-1978) y su
cuestionamiento al requisito de
completitud para algunas regiones de las
ciencias formales. Finalmente,

revisaremos las nociones de


interpretacin y modelo en las ciencias
formales. Estas nociones sern
retomadas en los captulos siguientes ya
que una importante corriente
epistemolgica considera a las teoras
de las ciencias fcticas como "sistemas
axiomticos interpretados" y encuentra
elementos comunes en el anlisis lgico
de las teoras fcticas y de las teoras
formales.

Actividades de
aprendizaje
ACTIVIDAD 1. CONSTRUCTOS

FORMALES Y REALIDAD
La relacin entre ciencias formales y
realidad reintroduce, en parte, las
disputas o debates en torno a la relacin
entre lenguaje y realidad, ya presentados
en la primera unidad puesto que las
ciencias formales son de muy productiva
aplicacin en la realidad (pensemos que
no podramos siquiera pagar el boleto
del colectivo si no tuvisemos un
dominio de las operaciones matemticas
elementales; mucho menos podran
construirse puentes o edificios de gran
altura ni tampoco planificar actividades
econmicas, para citar algunos
ejemplos). Pero estas ciencias justifican
el valor de verdad de sus enunciados
por medio de mtodos totalmente

formales, sin recurrir a la contrastacin


emprica ni esperando su validacin o
justificacin ms que en razonamientos
deductivos. En el fragmento que se
presenta a continuacin, se trata de
destacar la diferencia entre un
conocimiento formal (tautolgico) y un
conocimiento emprico (contingente) y
se propone volver al cuadro de
clasificacin de las ciencias presentado
en el libro de la bibliografa (cap. 1)
que, como dijimos, sirve como un
"mapa" para ir completando a lo largo
de todo el curso.
A. Lea el siguiente fragmento:
Definir una estructura es lo mismo que formular su
teora. Hay que especificar cules son los
conceptores, qu combinaciones de conceptores
son los axiomas y qu lgica determina la relacin

de consecuencia entre axiomas y teoremas. [...]


todo esto es independiente de la realidad emprica
del mundo, todo esto es mera matemtica. En
efecto, la matemtica suele definirse como la
ciencia de las estructuras. En este sentido, todas las
teoras son matemticas. [...] Lo que nos interesa
es, en primer lugar, el abigarrado yjugoso mundo
perceptual que nos rodea, y en segundo lugar, el
mundo que simblicamente captamos con nuestro
lenguaje y con nuestros conceptos, en resumen, la
historia. La teora es un mero instrumento para
iluminar la historia. Pero la historia es siempre
hipottica e insegura. Slo los fros y vacos
teoremas de la teora son seguros, pero no dicen
nada acerca del mundo. [...] En definitiva,
poseemos un saber perfecto y seguro sobre lo
irreal, vaco y formal (las estructuras, objeto de las
teoras), pero slo un saber imperfecto e inseguro
sobre lo real, lo vivo y lo material (los sistemas
objeto de la historia).
Somos como las araas, y las teoras son como las
redes o telas de araa con que tratamos de captar
o capturar el mundo. No hay que confundir estas

redes o telas de araa con el mundo real pero, sin


ellas, cunto ms alejados estaramos de poder
captarlo y, en ltimo trmino, gozarlo![14]

B. Ahora le proponemos que conteste


las siguientes preguntas:
a. Teniendo en cuenta las caractersticas
de las ciencias formales, cmo
interpreta la frase "la matemtica suele
definirse como la ciencia de las
estructuras"?
b. Mostern asocia el conocimiento
perfecto y seguro a lo formal y el
imperfecto e inseguro a lo real. Explique
esta distincin tomando en cuenta el
cuadro de clasificacin de las ciencias
de la bibliografa (cap. 1).
c. Este autor realiza una analoga entre
los cientficos y las araas. Explique
esta analoga.

ACTIVIDAD 2. SISTEMAS
AXIOMTICOS. ACERCA DE LA
INTENCIONALIDAD DE TODO
ORDENAMIENTO LGICO Y DE LA
NECESIDAD DE TENER UN
MTODO.
Una de las caractersticas destacadas
del conocimiento cientfico es la
utilizacin de mtodos o procedimientos
que nos permitan llegar a la afirmacin
de teoras. En esta unidad se presentan
las caractersticas del "mtodo
axiomtico" en tanto es el adecuado para
la construccin de teoras en ciencias
formales. Recordemos que la palabra
mtodo, derivada del griego,
etimolgicamente significa camino. En
el pasaje que se cita a continuacin,

Lewis Carroll nos indica que es


necesario definir adnde queremos
llegar antes de saber qu mtodo o
camino seguir ya que, por definicin,
todo camino nos conduce a algn lugar.
A. Lea el siguiente dilogo entre Alicia
y el gato de Cheshire.
B. Luego, explique por qu Alicia
afirma que la proposicin: "Si slo
caminas bastante, puedes estar segura
de llegar a algn lado", es irrefutable.

"Gatito de Cheshire" empez con un


poco de temor, porque no Y V estaba
muy segura de que a lle gustara el
nombre. Sin embargo, el Gato sonri
algo ms ampliamente. "Vamos, hasta
ahora le gusta", pens Alicia y

continu:
"Querra decirme, por favor, qu
camino debo tomar para irme de
aqu?"
"Eso depende mucho del lugar
adndequieras llegar"dijo el Gato.
"Me da lo mismo ellugar[...]"dijo
Alicia.
"Entonces no importa qu camino
tomes"dijo el Gato.
"[...]siempre y cuando llegue a algn
lado agreg Alicia a modo de
explicacin."

"Oh, puedes estar segura de llegar a


algn lado "dijo el Gato, "si slo
caminas bastante."
Alicia comprendi que esto era
irrefutable, de modo que prob con
otra pregunta [...]
(Carroll, L. 1990)

ACTIVIDAD 3. ACERCA DE LAS


PROPIEDADES DE UN SISTEMA
AXIOMTICO
Dado el siguiente sistema axiomtico
formal (SAF):
Trminos primitivos: M, I, U.
Axiomas: MI
Reglas:

RI: Si se tiene una cadena cuya ltima


letra sea I, se le puede agregar una U al
final.
RII: Supongamos que se tenga Mx. En
tal caso, puede agregarse Mxx a la
coleccin. ("x" es cualquier cadena, por
ejemplo: I, o IU, etc. As podemos pasar
de MI a MII siendo I "x" e II "xx", o
pasar de MIU a MIUIU siendo IU "x" e
IUIU "xx"). Las cadenas nunca pueden
contener "x".
RIII: Si en una de las cadenas de la
coleccin aparece la secuencia I I I,
puede elaborarse una nueva cadena
sustituyendo I I I por U.
RIV: Si aparece UU en el interior de
una de las cadenas, est permitida su
eliminacin.[15]

Puede usted producir la palabra MU


como teorema? El desafo consiste en
probar o bien que la palabra MU es un
teorema en el sistema MIU, o bien que
no lo es. Si MU es un teorema, entonces
puede ser generado a partir de su nico
axioma y las cuatro reglas de
transformacin de frmulas, y en tal
caso habr que mostrar su derivacin. Si
MU no es un teorema de MIU, entonces
habr que probar que bajo ninguna
circunstancia puede ser generado por el
axioma y las cuatro reglas de MIU. Esta
prueba no es una derivacin en MIU,
sino que tendr la forma de una
demostracin en el metalenguaje. se es
el desafo.
MATERIALES DE LECTURA I

ACERCA DE LAS CARACTERSTICAS


DEL CONOCIMIENTO DE LAS
CIENCIAS FORMALES Y SU
DIFERENCIA CON EL
CONOCIMIENTO DE LA REALIDAD
TAL COMO LO PRESENTA
BERTRAND RUSSELL
En el siguiente texto, de modo algo
irnico, Bertrand Russell destaca la
diferencia entre un mundo "ucrnico"
(no afectado por el paso del tiempo) y el
mundo real donde todo es perecedero y
el tiempo es una magnitud primordial.
Con esta nota seala la diferencia entre
el conocimiento formal y el
conocimiento fctico. Aqu Russell
alude al mundo de las ciencias formales
como un "mundo ucrnico", intemporal,

es decir, donde el paso del tiempo no


cambia nada y donde no hay progreso,
en el sentido en que podramos decir
que no hay progreso entre adoptar como
un juego al ftbol o al tenis,
sencillamente son dos juegos distintos.
Como veremos en las siguientes
unidades, se debate acerca del progreso
de las ciencias en el campo de las
ciencias fcticas. De todos modos, la
aplicacin de la nocin de progreso a
las ciencias formales es motivo de
debates epistemolgicos, por ejemplo,
si representa un progreso cientfico la
aparicin de las geometras no
euclidianas.
Bertrand Arthur William Russell,

tercer conde Russell, naci el 18 de


mayo de 1872 y muri, a los 97 aos, el
2 de febrero de 1970. Durante casi un
siglo vivi una vida asombrosamente
variada. Bertrand Russell fue el segundo
hijo del Vizconde de Amberley, un ttulo
creado en 1861 para su abuelo, Lord
John Russell, el primer ministro liberal
que luch por la Ley de Reforma de
1831 y fue partidario decidido de las
doctrinas de John Stuart Mill. Su madre
muri cuando tena tres aos y fue
educado por su padre en el
agnosticismo, pero al ao siguiente
tambin l muri. Su abuela le imparti
una educacin estrictamente victoriana
en franca contravencin a la voluntad de
sus padres muertos. Su abuelo, John

Russell, haba sido primer ministro de


la Reina Victoria de 1846 a 1852 y de
nuevo de 1865 a 1866. De adulto,
Russell recordaba cuando se sentaba en
las rodillas regias de Victoria durante
las visitas de sta a la mansin de su
abuelo. Segn su propio relato, fue un
joven solitario y reprimido. Aludiendo a
s mismo confes: "Yo no nac feliz[...]
Durante mi adolescencia aborrec la
vida y estuve continuamente al borde
del suicidio, del cual me apartaba
debido a mi deseo de aprender ms
matemticas". Llegado el momento,
march al Trinity College, en
Cambridge, la misma institucin que
acogi al joven Isaac Newton, ms de
dos siglos antes y que luego recibira a

otros genios de la talla de John Maynard


Keynes y Ludwig Wittgenstein, con
quienes entabl una relacin de
profunda colaboracin intelectual.
Inmediatamente luego de entrar a la vida
universitaria, Russell se entreg a las
matemticas, algo impersonal que, en
sus propias palabras, poda amar sin ser
amado en reciprocidad. Para Russell,
las matemticas ofrecan una nica va
para la certeza y perfeccin. "Me
desagradaba el mundo real, y busqu
refugio en el mundo ucrnico, sin
cambio ni corrupcin ni el fuego fatuo
del progreso."

MATERIALES DE LECTURA II
ACERCA DE BORGES Y LOS

INFINITOS
El primer prrafo del cuento de Borges
"El libro de arena",[16] dice:
La lnea consta de un nmero infinito
de puntos; el plano, de un nmero
infinito de lneas; el volumen, de un
nmero infinito de planos; el
hipervolumen, de un nmero infinito
de volmenes... No, decididamente
no es ste, more geomtrico, el mejor
modo de iniciar mi relato. Afirmar
que es verdico es ahora una
convencin de todo relato fantstico;
el mo, sin embargo, es verdico.
En este cuento Borges problematiza la
idea de infinito. Segn el relato, el

protagonista compra a un vendedor que


toca a su puerta un curioso libro, muy
pesado. La particularidad de este libro
es que una vez abierta una hoja ya nunca
se la volver a encontrar. En este
curioso libro entre la tapa y la primera
pgina hay infinitas pginas de modo
que se presenta la situacin anmala de
que nunca podramos acceder a la
primera pgina porque siempre
aparecer otra. "Ninguna es la primera,
ninguna es la ltima", afirma el
vendedor. El protagonista lo compra
pero luego de un tiempo se deshace del
libro que considera tan monstruoso
como l mismo. As nos lo cuenta
Borges:

Declinaba el verano, y comprend que


el libro era monstruoso. De nada me
sirvi considerar que no menos
monstruoso era yo, que lo perciba
con ojos y lo palpaba con diez dedos
con uas. Sent que era un objeto de
pesadilla, una cosa obscena que
infamaba y corrompa la realidad.
Pens en el fuego, pero tem que la
combustin de un libro infinito fuera
parejamente infinita y sofocara de
humo al planeta.
Record haber ledo que el mejor
lugar para ocultar una hoja es un
bosque. Antes de jubilarme trabajaba
en la Biblioteca Nacional, que guarda
novecientos mil libros; s que a mano

derecha del vestbulo una escalera


curva se hunde en el stano, donde
estn los peridicos y los mapas.
Aprovech un descuido de los
empleados para perder el Libro de
Arena en uno de los hmedos
anaqueles. Trat de no fijarme a qu
altura ni a qu distancia de la puerta.
Siento un poco de alivio, pero no
quiero ni pasar por la calle Mxico.
En el siguiente texto, el escritor y
matemtico Guillermo Martnez
profundiza sobre este tema en su libro
Borges y la matemtica:[17]
Dice Guillermo Martnez (Argentina,
1962): "Hay elementos de matemtica
muy variados a lo largo de la obra de

Borges. Los cuentos 'El disco', 'El libro


de arena', 'La biblioteca de Babel', 'La
lotera de Babilonia', 'Del rigor en la
ciencia', 'Examen de la obra de Herbert
Quain', Argumentum ornithologicum'; los
ensayos La perpetua carrera de Aquiles
y la tortuga' junto con 'Avatares de la
tortuga', 'El idioma analtico de John
Wilkins', 'La doctrina de los ciclos',
'Pascal' junto con 'La esfera de Pascal',
etc. Hay textos que son incluso pequeas
lecciones de matemtica. Uno de los
cuentos sealados es 'El libro de arena'.
Qu ocurrir cuando consideramos los
nmeros fraccionarios? Los nmeros
fraccionarios son muy importantes en el
pensamiento de Borges. Por qu?
Recordemos que los nmeros

fraccionarios, que tambin se llaman


quebrados, o nmeros racionales, son
los que se obtienen al dividir nmeros
enteros, los podemos pensar como pares
de enteros: un nmero entero en el
numerador y un nmero entero (distinto
de cero) en el denominador. 3/5, 5/4,
7/6, 7/16...
Cul es la propiedad que tienen
estos nmeros, la propiedad que usa
Borges en sus relatos? Entre dos
nmeros fraccionarios cualesquiera
siempre hay uno en el medio. Entre 0
y 1 est 1/2, entre 0 y 1/2 est 1/4,
entre 0 y 1/4 est 1/8, etc. Digamos,
siempre se puede dividir por 2.

De modo que cuando yo quiero saltar


del 0 al primer nmero fraccionario,
nunca puedo encontrar ese primer
nmero en el orden usual, porque
siempre hay uno en el medio. sta es
exactamente la propiedad que toma
prestada Borges en 'El libro de
Arena'. Recordarn que hay un
momento en este cuento en que al
personaje de Borges lo desafan a
abrir por la primera hoja el Libro de
Arena. 'Me dijo que su libro se

llamaba el Libro de Arena porque ni


el libro ni la arena tienen principio
ni fin.
Me pidi que buscara la primera
hoja. Apoy la mano izquierda sobre
la portada y abr con el dedo pulgar
casi pegado al ndice. Todo fue
intil: siempre se interponan varias
hojas entre la portada y la mano.
Era como si brotaran del libro'."

MATERIALES DE LECTURA III


ACERCA DE KURT GDEL Y
JOHN NASH. MENTES QUE
BRILLAN
Es frecuente y parte del imaginario

popular la asociacin entre genios


matemticos y esquizofrenia. La
pelcula norteamericana A Beautiful
Mind, traducida como una mente
brillante, alude a los trastornos
psiquitricos del matemtico
norteamericano John Nash, Premio
Nobel de Economa de 1994, por sus
aportes a la teora de juegos y los
procesos de negociacin, junto con
Reinhard Selten y John Harsanyi.
Existe una vasta literatura tanto
cientfica como no cientfica que
asimila al genio, es decir, a una
persona con talentos intelectuales ms
desarrollados o un coeficiente
intelectual superior, con un carcter
extravagante, insociable o patolgico.

En esta actividad presentamos


algunas de las circunstancias de la
vida de Gdel, no tanto para
detenernos en detalles biogrficos,
sino para mostrar el ambiente
intelectual de la produccin de
teoras revolucionarias en ciencias
formales en las primeras cinco
dcadas del siglo XX, visto desde los
avatares personales de algunos de sus
principales protagonistas que
padecieron las consecuencias de dos
guerras mundiales, la persecucin
racial o poltica, el exilio y, en
muchos casos, la incomprensin de
sus contemporneos.
Kurt Gdel naci el 28 de abril de
1906 en Brnn, Moravia, en la poca

parte del Imperio Austro-hngaro.


Ingres en la Universidad de Viena en
1924 planeando estudiar Fsica
Terica. Hacia 1926 su atencin
volvi a las matemticas y se produjo
su unin a lo que ms tarde fue
conocido como el Crculo de Viena.
Gdel estuvo asociado con este grupo
durante muchos aos.
Gdel se fue interesando
progresivamente en Teora de
Nmeros y, despus, en Lgica
Matemtica durante estos aos. En
1930, Gdel se doctor en
matemticas dirigido por H. Hahn, un
notable matemtico miembro del
Crculo de Viena. A partir de aqu
Gdel comienza a trabajar en sus ms

importantes teoras sobre la


completitud de sistemas formales.
Viaj a los Estados Unidos dando un
ciclo de conferencias y se encontr
por primera vez con Albert Einstein
en 1933. Gdel pas el ao
acadmico 1933-34 en Princeton, en
el recin fundado Instituto de
Estudios Avanzados, donde disert
sobre sus resultados de
incompletitud. Fue invitado a volver
al ao siguiente, pero al poco de
regresar a Viena sufri una grave
crisis mental. Se recuper a tiempo
para retornar a Princeton en el otoo
de 1935; al mes de su llegada sufri
una recada, y no volvi a impartir
enseanza hasta la primavera de

1937, en Viena.
Dedic alguno de los aos siguientes
al estudio de problemas de Fsica y
de Psicologa. Durante esta poca
tuvo que ser ingresado varias veces
en hospitales por problemas de salud.
Por ser confidencial el historial
mdico de Gdel, la diagnosis de su
mal sigue siendo desconocida. Sus
problemas parecen haber comenzado
con hipocondra: estaba obsesionado
por su dieta y por sus hbitos
intestinales. Durante veinte aos
llev un registro diario de su
temperatura corporal y de su consumo
de leche de magnesia. Tema sufrir un
envenenamiento accidental; con los
aos, le aterraba ser objeto de una

intoxicacin deliberada. Esta fobia le


llev a no querer tomar alimentos,
con la consiguiente desnutricin. Lo
que no le impeda ingerir pldoras de
diversa condicin para un imaginario
problema cardaco. Gdel se cas
con Adele Porkert en 1938 y
decidieron trasladarse
definitivamente a los Estados Unidos
en 1940. Ms de una vez Adele,
sirvindole de catadora de alimentos,
contribuy a paliar los temores de
Gdel, cada vez ms fuertes, de que
buscaban envenenarlo. Se asentaron
en Princeton, New Jersey, donde
residieron hasta el final de sus vidas.
Lleg a ser un gran amigo de Einstein,
y trabajaron juntos los aspectos

filosficos y matemticos de la
Teora General de la Relatividad.
Gdel incluso trabaj con xito en las
ecuaciones del campo gravitatorio,
encontrando soluciones
sorprendentes. Tambin dedic gran
parte de esa poca al estudio del
concepto de tiempo, publicando
varios artculos y dando varias
conferencias sobre el tema. El ltimo
de sus artculos publicados en vida
apareci en 1958. Despus, se sumi
en la introversin, cada vez ms
demacrado, paranoide e
hipocondraco. Su ltima aparicin
pblica aconteci en 1972, al recibir
un doctorado honorfico por la
Universidad Rockefeller. Tres aos

despus le fue otorgada la Medalla


Nacional de Ciencias, pero Gdel
disculp su asistencia por razones de
salud. El 1 de julio de 1976,
alcanzados los 70 aos, edad de
jubilacin obligatoria, Gdel se
convirti en profesor emrito. Sus
responsabilidades empero no
disminuyeron, porque su esposa, que
durante tantos aos lo haba
alimentado y protegido, haba sufrido
pocos meses antes un ataque cardaco
que la dej invlida. Ahora le
corresponda a l cuidarla. Y as lo
hizo, con devocin, hasta julio de
1977, cuando ella hubo de someterse
a una operacin de urgencia y
permaneci hospitalizada durante

casi seis meses. Por aquellas fechas,


Morgenstern, el amigo que haba
contribuido a cuidar de Gdel tras
fallecer Einstein en 1955, muri de
cncer. Gdel tuvo entonces que
luchar por s solo contra su cada vez
ms acusada paranoia. Solo frente a
ella, su declive se aceler. Temeroso
de ser envenenado, dej de comer y
acab muriendo por desnutricin el
14 de enero de 1978.

MATERIALES DE LECTURA IV
GDEL PARA TODOS
En la Feria del Libro de la ciudad de
Buenos Aires realizada en abril de 2009
se present el libro de Guillermo
Martnez y Gustavo Pieiro, Gdel para

todos.[18] En una nota de la poca, se


hace un comentario del libro en el que
encontramos una breve resea de los
principales hitos en la historia de las
ciencias formales.
Gdel para todos
En la evolucin de cada disciplina
existen hitos, momentos de ruptura.
En el caso de la matemtica, el
Teorema de Incompletitud de Gdel
marca un antes y un despus. El
mtodo axiomtico-deductivo, fuente
de toda verdad desde que fuera
introducido por Platn y Aristteles y
plasmado por Euclides, mostrar
importantes limitaciones: existen

verdades no demostrables. En Gdel


para todos, Guillermo Martnez y
Gustavo Pieiro se plantean el
inmenso desafo de desarrollar la
demostracin completa del Teorema
de Gdel con rigor absoluto y
accesible a todos.
Los egipcios y los babilonios
comienzan a construir la matemtica
en torno de problemas ligados a la
medicin de la Tierra. Los griegos
toman el relevo y constatan que, para
mejorar su eficacia, la matemtica
debe revisar sus fundamentos. As, en
los Segundos Analticos, Aristteles
seala: "La ciencia demostrativa
debe partir de premisas verdaderas,
primeras, inmediatas, ms

familiares, anteriores, causas de la


conclusin". En este contexto surgen
los elementos de Euclides (325-265
a.C.), que organizan por primera vez
los principales resultados de la
Geometra a partir de diez verdades
primeras. Euclides distingue entre
postulados y nociones comunes o
axiomas. Las nociones comunes son
verdaderas en todos los campos del
pensamiento e incluyen afirmaciones
tales como: "Las cosas iguales a una
misma cosa son iguales entre s", y
los postulados se aplican a temas
especficos de la geometra como:
"Dos puntos determinan una recta".
Desde Euclides, para validar sus
resultados, la matemtica recurre a un

mtodo especial, la demostracin


deductiva a partir de estas verdades
primeras. En la formulacin de los
postulados de Euclides hubo cierta
incomodidad. Las sospechas caan
sobre el Postulado de las paralelas
(quinto postulado): "Dados una recta
y un punto exterior a ella puede
trazarse una nica recta paralela a la
recta dada que pase por ese punto".
Era necesario considerarlo un
postulado o se podra deducir de los
otros, en cuyo caso era en realidad un
teorema? En 1799, Karl Friederich
Gauss escribe: "He hecho algunos
progresos en mi trabajo que no
conducen en absoluto a la meta que
buscamos (la deduccin del quinto

postulado); ms bien parecen


obligarme a dudar de la geometra
misma". A partir de 1831, Gauss
desarrolla una geometra basada en la
afirmacin de todos los postulados y
la negacin del quinto, y concluye que
su trabajo no conlleva contradiccin.
Muy asombrado afirma: " Los
teoremas de esta geometra parecen
paradjicos y, para los no iniciados,
absurdos, pero una reflexin
tranquila y sosegada revela que no
contiene en absoluto nada
imposible". Los trabajos de Nicoli
Lobachevski, Jnos Bolyai y Georg
Riemann profundizan las ideas de
Gauss, mostrando que es posible
negar el postulado de las paralelas

sin generar contradicciones y


desarrollar las consecuencias del
nuevo conjunto de axiomas. Dan
fundamento al descubrimiento de
nuevas geometras -no euclidianas- e
instalan la idea de que los axiomas y
postulados de Euclides son verdades
empricas ms que verdades
evidentes por s mismas. En
particular, deja de tener sentido
distinguir entre postulados (verdades
de la matemtica) y nociones
comunes (verdades fuera de la
matemtica). Las verdades primeras
pasan a considerarse todas axiomas,
independientes de la intuicin y la
experiencia. En la segunda mitad del
siglo XIX, los resultados sobre las

geometras no euclidianas hacen que


los matemticos pongan en duda los
fundamentos de otras zonas de la
disciplina. Esto lleva a un trabajo de
revisin y a una preocupacin por
elaborar una fundamentacin slida
eliminando los conceptos vagos y las
contradicciones, que resultan en
axiomas. Como ejemplo de conceptos
que se busca precisar, podemos citar
la ambigedad de Euclides al hablar
de puntos que estn entre otros puntos
sin aclarar la nocin de "estar entre",
y al definir una recta como "aquella
que yace por igual respecto de los
puntos que estn en ella", sin aclarar
qu entiende por "yacer por igual".
David Hilbert, principal impulsor de

la matemtica de la segunda mitad del


siglo XIX y primera del XX, propone
no definir los conceptos bsicos y
explcita: "Las nociones de puntos,
rectas y planos pueden reemplazarse
por mesas, sillas y jarros de
cerveza". Siguiendo a Hilbert, basta
que los axiomas hagan afirmaciones
acerca de conceptos no definidos. Si
el punto o la recta son conceptos no
definidos, el axioma que dice que dos
puntos determinan una sola recta
proporciona una afirmacin que
puede ser utilizada para obtener
posteriores resultados acerca de
puntos y rectas. En trminos
coloquiales, no importa quin eres,
basta saber con quin andas. La

intencin de Hilbert era refundar la


matemtica sobre bases slidas a
partir de sistemas axiomticos (o
teoras), entendidos como conjuntos
de axiomas con reglas lgicas que
permiten desarrollar demostraciones.
El desafo era encontrar un sistema
axiomtico consistente (libre de
contradicciones) y completo, es
decir, que permita obtener a travs de
demostraciones todos los resultados
verdaderos de la matemtica.
En 1929, el jovencsimo Kurt Gdel
(1906-1978) presenta su tesis
doctoral, en la que demuestra su
Teorema de Completitud, que refiere
a la lgica de predicados, esto es, a
las afirmaciones, vlidas en todo

contexto, que sustentan el


razonamiento matemtico. Es posible
dar axiomas que permiten demostrar
todas las afirmaciones de esta clase.
En 1930, casi todos los matemticos
estaban convencidos de que en todos
los sistemas axiomticos sera
posible encontrar teoremas de
completitud similares; elegidos
adecuadamente los axiomas, toda
afirmacin verdadera en la teora
sera deducible. Sin embargo, Gdel
demuestra que no es as en su famoso
primer Teorema de Incompletitud
(1931), en el que prueba que la
aritmtica elemental es incompleta:
no es posible dar axiomas que
permitan demostrar todas las

verdades de esta teora. [...] Se lee en


la Introduccin: "A diferencia de la
teora de la relatividad, en que por
la sofisticacin de las ecuaciones,
los mejores intentos de divulgacin
parecen condenados a ejemplos con
relojes y personas que no envejecen
en viajes por el espacio, en el caso
del Teorema de incompletitud hay
una buena noticia, y es que puede
darse una exposicin a la vez
rigurosa y accesible que no requiere
ninguna formacin matemtica, ms
que el recuerdo de la suma y la
multiplicacin tal como se ensean
en la escuela primaria".
Extrado de: Gdel para todos.
(2009, junio 6). Revista . [en lnea].

[consulta: 26 de julio de 2010].


<http://www.revistaenie.clarin.com/no
01932842.htm>
Actividad de integracin de los temas
de la Unidad
Determine si las siguientes afirmaciones
son verdaderas o falsas (V F).
Justifique en cada caso.
a. En una regla de inferencia deductiva
nunca se da el caso de que de una
falsedad se obtenga una verdad.
b. En una regla de inferencia deductiva
nunca se da el caso de que de una
verdad se obtenga una falsedad.
c. En un sistema axiomtico, todos los
enunciados se demuestran.
d. En un sistema axiomtico, todos los

trminos se definen.
e. Las geometras no euclidianas
surgieron al cuestionarse la
independencia del V postulado de
Euclides.
f. Los teoremas son relativos a su
sistema de pertenencia; esto es, en otro
sistema pueden funcionar como axiomas.
g. Las geometras no euclidianas
representan un avance cientfico que
resta inters a la geometra de Euclides.
h. La independencia de los axiomas
permite que unos se puedan deducir de
otros.
i. Si todos los teoremas se demostraran,
los sistemas axiomticos seran viciosos
o infinitos.

Temas de la Unidad
4.1. Estructura de las teoras empricas
4.2. El problema del mtodo cientfico
4.3. Estrategias metodolgicas bsicas
de las ciencias fcticas
4.4. Mtodo inductivo: inductivismo
"estrecho" e inductivismo "sofisticado"
4.5. Mtodo hipottico-deductivo
Bibliografa obligatoria
Unidad 4 en la Gua de Estudio de ipc,
producida por UBA XXI y editada por
Eudeba, a partir de la edicin de 2010.
Asti Vera, C. y Ambrosini, C. (2009).
Captulo 5. En Argumentos y teoras.

Aproximacin a la Epistemologa.
Buenos Aires: Educando.
BIBLIOGRAFA COMPLEMENTARIA
Cohen, I. y Nagel, E. (1980). Captulos
X, XI, XIV y XVI. En Introduccin a la
lgica y al mtodo cientfico (vol. II).
Buenos Aires: Amorrortu.
Hempel, C. G. (1985). Captulos 2, 3 y
4. En Filosofa de la ciencia natural.
Madrid: Alianza.
Klimovsky, G. (1994). Captulos 9, 13 y
14. En Las desventuras del
conocimiento cientfico. Buenos Aires:
AZ.
Presentacin de la Unidad
Quin soy yo? Ah!Eso s que es un
misterio!" Y con esto se puso a en

pensar en todas las nias de su edad


que conoca, para ver si se haba
transformado en una de ellas. [...] "Voy
a ver si al menos s las cosas que antes
saba. Veamos: cuatro por cinco son
doce, cuatro por seis son trece, y
cuatro por siete... Ay, Dios mo! As
no llegar nunca a veinte! Bueno, de
todas formas la tabla de multiplicar no
me vale; probemos con la Geografa.
Londres es la capital de Pars, y Pars
es la capital de Roma, y Roma es[...]"
(Carroll, L. Alicia en el pas de las
maravillas)

La distincin entre ciencias formales y


fcticas reconoce diferencias entre
ambos tipos de saberes, lo que no
significa necesariamente que no existen
condiciones constructivas comunes.
Segn algunos autores, una teora

emprica puede considerarse como un


conjunto de hiptesis de partida y sus
consecuencias lgicas (Popper, Bunge,
Klimovsky). De all que esta unidad
comienza con la exposicin del lenguaje
y la estructura de una teora fctica,
realizando luego una presentacin de la
cuestin del mtodo en las ciencias
fcticas, tema que es motivo de
importantes debates en el mbito de la
epistemologa contempornea. En su
desarrollo examinaremos las estrategias
bsicas de las ciencias empricas, tanto
como las discusiones ms importantes
en el contexto de las metodologas
rectoras en el siglo XX. Entre otras
cuestiones anticipan las controversias,
an no canceladas, en torno al monismo

metodolgico y al pluralismo
metodolgico que estudiaremos de
manera ms especfica en las unidades
siguientes y que afectan especialmente a
las ciencias sociales, a su constitucin, a
la legitimacin de sus mtodos y a su
proyeccin histrica. En esta unidad
veremos distintas versiones acerca del
modo de interpretar la relacin teoraobservacin. Estas diferencias se
presentan a partir de la distincin
propuesta por el filsofo alemn Hans
Reichenbach (1891-1953) entre contexto
de descubrimiento y contexto de
justificacin, a los que el matemtico y
filsofo argentino Gregorio Klimovsky
(1922-2009) agrega el contexto de
aplicacin de las ciencias antes de

presentar la posicin del filsofo,


socilogo y terico de la ciencia
austraco Karl Popper (1902-1942),
quien confina la metodologa al contexto
de justificacin. Esta diferenciacin
entre contextos y la reclusin de la
Epistemologa a alguno de estos
contextos con exclusin de otros es
motivo de fuertes polmicas, en especial
entre las posiciones de Popper y del
epistemlogo estadounidense Thomas
Kuhn (1922-1996), y creemos que es
importante advertir la presencia de
dichas distinciones en esta unidad ya
que luego esas diferencias sern
recuperadas en las unidades siguientes.
A continuacin conviene que se detenga
a estudiar con especial inters, en el

captulo 5 de la bibliografa, las


distintas estrategias metodolgicas
bsicas en las ciencias fcticas donde
encontrar sealada la especificidad de
las ciencias sociales en la postulacin
de mtodos distintos a los de las
ciencias naturales. Este punto es
conveniente que lo relacione con el
Documento de Ctedra de Gastn
Beraldi de la Unidad 5.
En la Unidad 2, desde distintos
enfoques, hemos visto la induccin
como forma de razonamiento y los
esfuerzos de Carnap por legitimar este
tipo de inferencia como un
procedimiento formal. En esta unidad,
enfocaremos el problema del
inductivismo considerado como una

posicin metodolgica, es decir, la


posicin epistemolgica que considera a
la induccin como mtodo de
investigacin cientfica. El llamado por
sus crticos "inductivismo ingenuo"
defiende la induccin por enumeracin
simple, cree que la ciencia empieza con
la observacin y sostiene que la
observacin ofrece una base segura a
partir de la cual se puede derivar el
conocimiento. En su versin refinada, el
inductivismo tiene mtodos ms
sofisticados que la enumeracin simple,
pero sigue reconociendo a la
observacin como primera fuente de
conocimiento. En referencia al llamado
"inductivismo ingenuo", debe detenerse
en la lectura y comprensin de los

llamados "mtodos de Mill", que toman


el nombre de su creador, el filsofo
ingls John Stuart Mill (1806-1873). Se
conocen estos cnones o reglas de
investigacin como "mtodo de la
concordancia", "mtodo de la
diferencia", "mtodo conjunto de la
concordancia y la diferencia", "mtodo
de los residuos" y "mtodo de la
variacin concomitante". Estos mtodos
buscan descubrir conexiones causales
entre fenmenos. La insistencia de Mill
en la utilidad de estos mtodos para
descubrir conexiones causales lo llev a
largas polmicas con sus
contemporneos y a reafirmar sus
convicciones en defensa de la lgica
inductiva. Dada la importancia central

de esta posicin en la historia de la


epistemologa de los siglos XIX y XX,
en esta unidad encontrar expuestas
distintas versiones del inductivismo,
comenzando con la presentacin de los
mtodos de Mill y las crticas del
filsofo alemn Carl Hempel (19051997) cuando lo caratula como
inductivismo ingenuo antes de sealar
las versiones ms elaboradas de Carnap
y de Reichenbach. El estudio de este
tramo del captulo 5 de la bibliografa
es necesario para luego contextualizar
las distintas posiciones epistemolgicas
presentadas en la Unidad 6.
Ya avanzando en la historia del siglo
XX, se presenta en este captulo al
"mtodo hipottico-deductivo" en las

versiones confirmacionistas de Carnap y


Hempel, representantes del llamado
"inductivismo sofisticado". Se presenta
luego la versin refutacionista de
Popper y el llamado "refutacionismo
sofisticado" del matemtico y filsofo
de la ciencia hngaro Imre Lakatos
(1922-1974). En la presentacin de la
posicin refutacionista, adjudicada a
Popper, conviene que advierta la
importancia del Modus Tollens como un
argumento deductivo que permite la
refutacin de una hiptesis y ligada a la
importancia de este tema, acceder a la
comprensin del concepto de
"falsabilidad" como criterio de
demarcacin para considerar cientfica a
una hiptesis. En el desarrollo de esta

parte de la unidad, conviene que lea


atentamente los pasos del mtodo
hipottico-deductivo y los distintos tipos
de hiptesis involucradas en la puesta a
prueba de las teoras cientficas
(hiptesis fundamental, hiptesis
auxiliar e hiptesis ad hoc). Estas
distinciones son importantes ya que
sern consideradas como temas
centrales en la presentacin de la
posicin de Lakatos en la Unidad 6.
Dentro de la metodologa de las ciencias
fcticas es importante la utilizacin de
mtodos estadsticos en ciencias
naturales y en ciencias sociales, de all
que ste sea el tema que cierra esta
unidad. Si bien la utilizacin de los
mtodos estadsticos ha tenido un gran

desarrollo en el campo de las ciencias,


su justificacin no deja de ser
problemtica y de dar lugar a la
incursin en diversos tipos de
argumentos falaces. Al comienzo de este
tema se realiza una breve resea
histrica de los estudios sobre clculos
de probabilidad aludiendo a los
matemticos franceses Blas Pascal y
Pierre de Fermat (1608-1665). Se
presenta el concepto de "promedio
estadstico" visto como elemento
importante para luego revisar las
limitaciones de estos mtodos segn han
sido sealadas por los filsofos
estadounidenses Morris R. Cohen
(1880-1947) y Ernest Nagel (19011985). En este punto es importante

reconocer la conexin de este tema con


el uso de procedimientos inductivos y
los problemas que se presentan cuando
se trata de justificar este tipo de
inferencias.

Actividades de
aprendizaje
ACTIVIDAD 1. INDUCTIVISMO
El inductivismo es la posicin
epistemolgica que postula la induccin
como nico mtodo de investigacin
cientfica. Ya se ha visto que la
induccin es una forma de razonamiento
que no tiene validez lgica; sin embargo,

para conocer el mundo emprico y


ampliar nuestro conocimiento acerca de
l, con todas sus falencias, la induccin
sigue siendo imprescindible.
El inductivismo ingenuo defiende la
induccin por enumeracin simple, cree
que la ciencia empieza con la
observacin y sostiene que la
observacin ofrece una base segura a
partir de la cual se puede derivar el
conocimiento. En su versin refinada el
inductivismo tiene mtodos ms
sofisticados que la enumeracin simple
pero sigue reconociendo a la
observacin como primera fuente de
conocimiento.
Para los inductivistas ingenuos la
investigacin tiene los siguientes pasos:

1) Observacin neutral de la realidad.


2) Captacin de una propiedad en
diversos individuos.
3) Generalizacin de dicha propiedad a
todos los individuos.
4) Formulacin de una ley o
generalizacin emprica.
El supuesto que permite ir del paso 2 al
paso 3 es el llamado Principio de
Regularidad de los Fenmenos,
principio por el cual se cree que la
naturaleza es constante, de modo que se
hace valer para todos los casos lo
observado slo en algunos.
Lea el siguiente texto e identifique los
pasos de la investigacin inductiva
apuntados ms arriba:

Beber alarga la vida


Un estudio sobre los hbitos de
ingestin de alcohol de los
norteamericanos ha arrojado un
resultado sorprendente. Los hombres
que toman entre 0,8 litros y 2,4 litros
de cerveza a la semana tienen ms
oportunidades de vivir durante ms
tiempo. Segn el profesor David
Williams, de la Universidad de
Gales, en Cardiff (Reino Unido)
existen pruebas fehacientes de que las
personas que beben cerveza
moderadamente tienen una
expectativa de vida cinco aos mayor
que los abstemios.

ACTIVIDAD 2. INDICTIVISMO. LOS


MTODOS DE JOHN STUART MILL
En el texto de la bibliografa, se
mencionan los que se han considerado,
en su formulacin clsica, "mtodos de
Mill" de la inferencia inductiva. Mill ha
insistido en la utilidad de estos mtodos
para descubrir conexiones causales y
reafirmado sus convicciones en defensa
de la lgica inductiva: "La misin de la
lgica inductiva es proveer de reglas y
modelos (como el silogismo y sus
reglas son modelos para el raciocinio)
que, si los razonamientos inductivos se
adecuan a ellos, son concluyentes, pero
en caso contrario no lo son."
Segn Mill, sus mtodos permitiran

descubrir y probar conexiones causales.


Es indudable que esta pretensin no se
ha realizado, afirma Copi, puesto que la
utilizacin de mtodos mecnicos no
alcanz, en muchos casos, para
descubrir la causa de algunos fenmenos
ya que no hay ningn recurso simple o
mtodo mecnico que por s mismo haya
permitido conquistar el conocimiento
cientfico. "Para hacer un anlisis
correcto en la aplicacin de estos
mtodos se necesita el conocimiento
previo de teoras", concluye Copi.
(1984. 452-453)
2.1. El bebedor cientfico
Irving Copi en su libro introduccin a
la lgica (1984) cita el caso de un
bebedor que todas las noches se

emborracha. Estaba arruinando su


carrera y su salud. Uno de los pocos
amigos que le quedaba le aconsej
abandonar la bebida. Resolvi llevar a
cabo un cuidadoso experimento para
descubrir la causa exacta de las
borracheras al tomar distintas mezclas
de bebidas. Durante cinco noches
seguidas tom gin con soda, whisky con
soda, aguardiente con soda, cognac con
soda y ron con soda. Usando el mtodo
de la concordancia de Mill lleg a la
conclusin: "Nunca ms tomo soda".
Responda: El mtodo de la
concordancia de Mill avalara esta
conclusin? Dnde radica la falacia?
2.2. Pasteur y el experimento con la
vacuna contra el carbunco

En la primavera de 1881, Pasteur


puso a prueba su hiptesis de que la
vacuna de carbunco produce
inmunidad a la enfermedad, hiptesis
ridiculizada por los veterinarios de la
poca. En una granja de Poully-leFort se administr la vacuna a 24
ovejas, una cabra y varias vacas. Se
tom como muestra de control otras
veinticuatro ovejas, una cabra y
varias vacas que quedaron sin
vacunar. Dos meses despus las 48
ovejas, las dos cabras y todas las
vacas fueron inoculadas con una
dosis letal de virulentos bacilos de
carbunco. Tres das despus los
animales del primer grupo brincaban
y coman normalmente como si nunca

hubieran estado en contacto con el


bacilo, mientras que los animales no
vacunados estaban muertos o en una
irreversible agona.
As mostrado, este experimento parece
responder al mtodo conjunto de la
concordancia y la diferencia. Identifique
los pasos de la investigacin en el texto.
Este experimento prueba que es
verdadero el enunciado "la vacuna
inmuniza contra la enfermedad"? Por
qu?
2.3. El piojo y el tifus
Se supuso durante mucho tiempo que
el virus del tifus es inoculado por el
piojo al chupar la sangre, pero no es

as. La infeccin no est en la saliva,


como en el caso del mosquito, sino en
las heces. Al ser picado, el animal se
rasca y la infeccin entra en contacto
con la sangre de tal modo que
"picadura" e "infeccin" son
fenmenos inseparables. En 1922 dos
bilogos pusieron piojos infectados
sobre un mono, tomando la
precaucin de que las heces no
entraran en contacto con la piel del
mono. A pesar de las picaduras, el
mono no se infect.
Responda: Cul de los mtodos de Mill
puede advertirse en este experimento?
Los resultados del experimento son
concluyentes para afirmar que "las heces

del piojo contagian la enfermedad"?


ACTIVIDAD 3. ACERCA DEL
MTODO HIPOTTICO-DEDUCTIVO
El amor es una droga dura es el ttulo
de una novela de la escritora uruguaya
Cristina Peri Rossi,[19] residente en
Barcelona. La historia trata sobre un
fotgrafo de 50 aos, felizmente casado
y asentado en su profesin, que super
las secuelas de una vida de excesos y
adicciones al alcohol y las drogas
pesadas pero que sucumbe a la ltima
tentacin: Nora. Del mismo lugar, el
diario La vanguardia (Barcelona), el 2
de junio de 2005, publica una nota
donde da cuenta de investigaciones
cientficas acerca del funcionamiento
del cerebro humano, titulada "El amor

como adiccin".
Lea atentamente el texto y, luego,
conteste las preguntas.
La Vanguardia
Josep Corbella
Barcelona

EL AMOR COMO ADICCIN


El cerebro humano experimenta el
enamoramiento igual que una
adiccin, segn investigadores de
Estados Unidos que han observado
por resonancia magntica qu ocurre

en el cerebro de personas que han


iniciado hace poco una relacin de
pareja. La investigacin responde a
viejos debates sobre la relacin entre
amor y sexo. Segn resultados
publicados el martes en The Journal
of Neurophysiology, la atraccin
fsica y el enamoramiento son
procesos diferentes que activan
regiones distintas del cerebro. Esto
explicara, segn los autores de la
investigacin, por qu una persona
puede encontrar atractivas a mltiples
parejas potenciales, pero difcilmente
enamorarse de ms de una a la vez: la
regin que procesa el enamoramiento
tiende a la monogamia, mientras que
la que procesa la atraccin fsica

tiende a la poligamia.
Los investigadores -de la
Universidad del Estado de Nueva
York, la Universidad Rutgers de
Nueva Jersey y la Escuela de
Medicina Albert Einstein de Nueva
York- han analizado a 10 mujeres y 7
hombres de 18 a 26 aos que
declararon llevar entre 1 y 17 meses
"intensamente enamorados".
Las resonancias magnticas indican
que las regiones del cerebro que se
activan cambian a medida que una
relacin de pareja madura. En las
parejas que llevan entre uno y siete
meses juntas se observa una
hiperactividad en reas involucradas
en las adicciones, sobre todo el rea

tegmental ventral y el ncleo


caudado. Pero en parejas que llevan
ms tiempo juntas empieza a
activarse el plido ventral, que
parece vital para establecer
relaciones duraderas. El estudio
muestra que, a medida que pasan los
meses y unas reas del cerebro se
activan, otras se desactivan. Esto
explicara por qu una relacin
duradera y gratificante no impide que
una pareja pueda verse sorprendida
por un enamoramiento imprevisto:
mientras el rea del cerebro que
garantiza la relacin estable
permanece activa, la habitacin de la
pasin, en el rea tegmental ventral,
se encuentra disponible.

Pero el resultado que los autores de


la investigacin consideran ms
relevante es que el enamoramiento no
es una emocin sino ms bien una
adiccin. "No hemos encontrado
ningn patrn emocional consistente",
declara en un comunicado Arthur
Aron, codirector de la investigacin
de la Universidad del Estado de
Nueva York,. Las emociones del
enamoramiento, advierte Aron,
pueden oscilar de manera catica
entre la euforia, la ansiedad, el
enfado, la tristeza o la alegra. Por el
contrario, "todos nuestros voluntarios
mostraron una actividad intensa en las
regiones de motivacin y recompensa
del cerebro". Estas regiones son las

mismas que se activan en las


adicciones. Igual que en una adiccin,
los investigadores han observado que
el enamoramiento se asocia a intensas
descargas de dopamina en el centro
del cerebro. Y otro punto en comn
con las adicciones: cuando una
persona que se encuentra en esta fase
efervescente es rechazada por su
pareja, presenta un patrn de
actividad cerebral similar al de un
sndrome de abstinencia, segn un
nuevo estudio del mismo equipo de
investigacin difundido por The New
York Times. Este sndrome de
abstinencia explicara por qu
muchas rupturas, ms que inhibir el
deseo de estar con la otra persona, lo

acrecientan. Ignasi Morgado,


catedrtico de Psicobiologa de la
Universitat Autnoma de Barcelona
(UAB), record ayer que los estudios
de Samir Zeki y Andreas Bartels en el
Colegio Universitario de Londres han
mostrado una inhibicin de la
actividad del crtex prefrontal -la
principal sede de la racionalidad en
el cerebro- en personas que declaran
estar muy enamoradas. "Por lo tanto,
parece hacer falta una cierta
irracionalidad para el
enamoramiento", seal.
Las principales reas del cerebro
involucradas en el enamoramiento
trabajan a nivel inconsciente y son
comunes a todos los mamferos. Los

autores del estudio recuerdan adems


que hay una especie de ratones de las
praderas que establecen relaciones
mongamas duraderas gracias a la
activacin del mismo rea del
cerebro que las personas.
Todo ello apunta a que "el
enamoramiento es posiblemente un
tipo de impulso bsico de los
mamferos para optimizar el proceso
de apareamiento", afirma Helen
Fisher, codirectora del estudio. Lo
cual, sin embargo, no significa que la
experiencia del enamoramiento sea
igual en la especie humana que en
otros mamferos, ya que el cerebro
humano tiene una capacidad de ser
consciente del proceso, superior al de

cualquier otra especie. Y tampoco


significa, advierten los
investigadores, que el enamoramiento
pueda reducirse a los procesos
fisiolgicos que se observan en las
resonancias magnticas, ya que es un
fenmeno complejo condicionado por
influencias culturales.
Josep Corbella. (2005, junio 2).
El amor como adiccin. La vanguardia.
A. Segn el mtodo hipotticodeductivo en su versin falsacionista:
a. Cul es el problema?
b. Cul o cules son las hiptesis de
partida de los neurlogos?
c. Qu consecuencias observacionales
pondran a prueba estas hiptesis?

d. Considera suficiente la base


observacional ofrecida por los
neurlogos mencionados? Justifique su
respuesta.
e. Se ha demostrado la verdad de
alguna hiptesis, segn este informe?
Justifique su respuesta.
B. Encuentra falsables los siguientes
enunciados? Justifique sus respuestas en
cada caso.
a. "El enamoramiento es un fenmeno
complejo condicionado por influencias
culturales".
b. "El enamoramiento es posiblemente
un tipo de impulso bsico de los
mamferos para optimizar el proceso de
apareamiento."
ACTIVIDAD 4. MTODO

HIPOTTICO-DEDUCTIVO. ACERCA
DE LOS TIPOS DE HIPTESIS
En el libro de la bibliografa encontrar
la distincin entre distintos tipos de
hiptesis que pueden participar en la
puesta a prueba de una teora: hiptesis
fundamental, hiptesis auxiliar e
hiptesis ad hoc. Cuando hay que
derivar consecuencias observacionales
de las hiptesis principales, a menudo
hay que hacer suposiciones adicionales
ya que estas hiptesis principales, por s
solas, no permiten derivar directamente
consecuencias observacionales. A estas
suposiciones adicionales se las llama
"hiptesis auxiliares". Se supone que
estas hiptesis han sido contrastadas con
anterioridad, de forma independiente,

con la hiptesis principal y pueden o no


formar parte de la disciplina a la que
pertenece la hiptesis principal. Una
hiptesis auxiliar se considera ad hoc
cuando no puede ser contrastada de
forma independiente de la principal y es
usada para salvar a la principal de ser
refutada.
Lea el siguiente texto y, luego, responda
las preguntas.
Martes 6 de enero de 2009

LA ESFERIDAD DE LA TIERRA
2: DE LA ANTIGEDAD A
COLN

La idea de una Tierra esfrica es muy


antigua y se remonta al menos a los
tiempos de la filosofa griega clsica
y posiblemente tambin a la filosofa
india antigua.
El concepto de una Tierra esfrica
desplaz a antiguas creencias sobre
una Tierra plana. En el pensamiento
mesopotmico se consideraba que el
mundo era un disco plano flotando en
el ocano y este retrato de la Tierra
fue plasmado en los primeros mapas
de la Tierra realizados por
Anaximandro y Hecateo de Mileto.
[...] Los primeros filsofos griegos se
refieren a una Tierra esfrica aunque
un tanto ambiguamente. Pitgoras
(nacido en 570 a.C.) basaba la

creencia de que la Tierra y los dems


planetas eran esfricos porque, para
l, el slido geomtrico ms
armnico era la esfera. [... ]
Eratstenes (276-194 a.C.) no slo
crea que la Tierra era redonda, sino
que adems estim sus dimensiones.
No posea computadoras, ni satlites
de reconocimiento, tan slo una mente
brillante y conocimientos de
trigonometra.

Eratstenes escuch que al sur en


Siena durante el solsticio de verano
el Sol permaneca en el cnit de
forma que los objetos no arrojaban
sombra alguna. Se dice que

Eratstenes contrat a otra persona


para medir la distancia entre Siena y
Alejandra mediante la cuenta de sus
pasos. Al llegar el solsticio de
verano se hicieron dos observaciones
simultneas. Mientras que en Siena
una estaca no arrojaba sombra
ninguna a medioda, en Alejandra
exista una pequea sombra.
Eratstenes midi el ngulo de la
sombra que result ser de unos 7,2,
como conoca la distancia a Siena,
por un sencillo clculo
trigonomtrico calcul la
circunferencia de la Tierra en
250.000 estadios (una medida griega
de longitud) traducido a valores
actuales nos damos cuenta de que

Eratstenes se equivoc en apenas un


3% en su estimacin, sin duda un
logro impresionante. [...]
[20]

a. Cul era la hiptesis principal de


Eratstenes?
b. Qu consecuencias observacionales
se derivaban de ella?
c. Qu pudo probarse, finalmente?
d. Encuentra alguna hiptesis auxiliar
implcita en este experimento, entre las
que se mencionan a continuacin?
Justifique su eleccin.
i) La forma en que caan los rayos
del sol.
ii) La distancia de Siena a
Alejandra.

iii) La existencia de dos obeliscos.


e. Reflexione acerca de la importancia
de las hiptesis auxiliares. Puede una
hiptesis auxiliar falsa malograr la
aceptacin de la hiptesis principal?
Por qu?

Temas de la Unidad
5.1. Qu significa "explicar"?
5.2. Concepto de explicacin cientfica
5.3. Modelos de explicacin cientfica
5.4. La especificidad de las ciencias
sociales: intencionalidad y explicacin
por mecanismos
5.5. Dimensin explicativa y dimensin
predictiva
5.6. El enfoque comprensivista en
ciencias sociales
Bibliografa obligatoria
Unidad 5 en la Gua de Estudio de IPC,
producida por UBA XXI y editada por
Eudeba, a partir de la edicin de 2010.

Asti Vera, C. y Ambrosini, C. (2009).


Captulo 6. En Argumentos y teoras.
Aproximacin a la Epistemologa.
Buenos Aires: Educando. Beraldi, G.
(2010). Documento de ctedra: La
tensin entre explicacin y
comprensin. El problema de la
explicacin en las ciencias sociales.
En esta Gua de Estudio de IPC, Buenos
Aires: Eudeba; y en el Campus virtual
de UBA XXI.
BIBLIOGRAFA COMPLEMENTARIA
Dez, J. A y Moulines, C. U. (1999).
Captulo 7. En Fundamentos de
filosofa de la ciencia. Barcelona:
Ariel.
Schuster, G. (1986). Captulos 2, 3, 4 y
5. En Explicacin y prediccin. Buenos

Aires: Clacso.
Wright, G. H. von (1979). Explicacin y
comprensin. Madrid: Alianza.
Presentacin de la Unidad

Sabes por qu son blancas las


pescadillas?"
"Nunca me lo he preguntado",
respondi Alicia. "Porqu?"
"Pues porque sirven para darles
brillo a los zapatos y a las botas",
explic el Grifo con gran
solemnidad, "por lo blancas que son
".
Alicia se qued de una pieza. "Para
sacar brillo!", repeta, sin saber
cmo explicrselo.
"Pues claro! A ver! Cmo se

limpian los zapatos?", le pregunt el


Grifo. "Quiero decir, cmo se les
saca brillo?
Alicia se mir los pies y reflexion
un poco antes de dar una
contestacin: "Con negro de betn
me parece".
"Pues bajo el mar a las botas y a los
zapatos se les da con blanco de
pescadilla", interpuso el Grifo con
voz pretenciosa: "Ya lo sabes."
(Carroll, L. Alicia en el pas de las
maravillas)

En el captulo 6 de la bibliografa, se
examinan los modelos de explicacin
cientfica ms transitados por la
Epistemologa contempornea as como
los debates que se generan en torno a las
modalidades explicativas en ciencias
sociales. Adems de los modelos de
explicacin nomolgico-deductivo y
estadstico-inductivo, en esta parte se
debe prestar atencin a las
caractersticas de la explicacin
gentica y de la explicacin teleolgica.

En este texto resulta de especial inters


la revisin de la particular posicin del
filsofo noruego Jon Elster (n. 1940)
quien, a partir de sus estudios sobre
distintos modelos de racionalidad,
considera que el criterio de
clasificacin de las ciencias debe tomar
en cuenta las distintas estrategias
explicativas. Reconoce tres
modalidades bsicas de explicacin:
causal, funcional e intencional. En este
orden, los campos de investigacin a los
que se aplica cada una de stas son:
ciencias fsicas, ciencias biolgicas y
ciencias sociales. La explicacin causal,
que es el modelo nico y cannico de
explicacin en el campo de la fsica,
tambin se emplea frecuentemente en los

otros dos campos de investigacin.


Elster critica el uso de explicaciones
funcionales en ciencias sociales y
reserva para estas ciencias el uso de las
explicaciones por mecanismos, de all la
presentacin de este tema que cierra el
captulo dedicado a dicho tema en la
bibliografa.
El material de lectura de esta unidad se
completa con el Documento de Ctedra
escrito por Gastn Beraldi, La tensin
entre explicacin y comprensin. El
problema de la explicacin en las
ciencias sociales, en el que aparece el
cuestionamiento a las versiones
explicacionistas en las ciencias sociales
y la bsqueda de mtodos propios. En
este artculo se presentan a los

principales representantes del


"comprensivismo", siguiendo el anlisis
de los filsofos Wilhelm Dilthey (18331911, alemn) y Georg H. von Wright
(1916-2003, finlands), identificados en
oposicin al Positivismo como
antipositivistas aunque no representan un
grupo homogneo ni permiten reducir
sus posiciones a unos pocos postulados
comunes. El rasgo comn del enfoque
positivista es el monismo metodolgico
y una posicin "explicacionista" para el
caso de las ciencias sociales. Aqu es
importante advertir la oposicin de
estos autores al monismo metodolgico
y la bsqueda de estrategias explicativas
propias para las ciencias sociales
alrededor del concepto de

"comprensin".

Actividades de
aprendizaje
MATERIALES DE LECTURA I
LA EXPLICACIN CIENTFICA
Mientras que describir es responder a
la pregunta acerca de cmo es algo,
explicar es responder a la pregunta de
por qu ese algo es o sucede. O sea
que explicar es dar cuenta de la
produccin y/o existencia de algn
fenmeno. Toda teora cientfica
puede ser vista como una gran

explicacin acerca de algn


fenmeno, evento, suceso o proceso.
Mas no toda explicacin cientfica
constituye por s sola una teora
completa. Sin embargo, no hay teora
sin explicacin, de all la importancia
que reviste este tema. Empleamos el
trmino "explicar" en distintos
sentidos: Explicar qua narrar: "Ana
me explic su viaje a China".
Explicar qua elucidar: "Ana me
explica las reglas de la gramtica".
Explicar qua respuesta a una
pregunta por qu": "Ana me explic
por qu decidi viajar a China".
Partes. Toda explicacin tiene dos
partes: el explanandum y el
explanans. El enunciado que contiene

el hecho a explicar se denomina


explanandum y el conjunto de
enunciados que conforman la
explicacin propiamente dicha, se
denomina explanans. El explanandum
puede describir un hecho puntual, por
ejemplo "se inunda el patio de mi
casa", o una regularidad, "el corcho
flota en el agua". En ambos casos el
explanandum encierra el enigma de
por qu ocurren tales cosas, enigma
que el explanans va a resolver.
Requisitos. Para que una explicacin
sea cientfica debe ser atinente y
contrastable. La atinencia o
relevancia explicativa es la
pertinencia del explanans respecto al
explanandum y la contrastabilidad es

la condicin por la cual todo los


enunciados de la explicacin tienen
que poder ser puestos a prueba.
Tipos. Hay varios tipos de
explicacin: genticas, teleolgicas o
funcionales, intencionales,
nomolgico-deductivas y las
estadstico-inductivas. Explicaciones
nomolgico-deductivas y
estadstico-inductivas. En ambos
tipos de explicacin el explanans est
formado por leyes y condiciones
iniciales. Las leyes son los
enunciados ms generales a los que
los enunciados observacionales se
subsumen y las condiciones iniciales
son los enunciados que describen las
situaciones de hecho que

acompaaron la produccin del


fenmeno que el explanandum
describe.

Explanans

Modelo nomolgicodeductivo
L1,L2,L3..................Ln
C1,C2,C3................Ln

Explanandum

La diferencia entre los dos tipos de


explicacin deriva de la diferencia

que hay entre las leyes que estn en el


explanans de uno u otro modelo. En
efecto, las leyes pueden ser de
probabilidad mxima, o bien de alta
probabilidad. Las leyes de
probabilidad mxima son aquellas
tales como la Ley de Gravedad, la
Ley de Boyle y Mariot, el Principio
de Arqumedes, etc., todos ellos
enunciados nomolgicos que
describen regularidades de la
naturaleza. En cambio, las leyes de
alta probabilidad son enunciados
estadsticos que describen
regularidades probables, con un
considerable margen de excepciones.
Se encuentran en este ltimo grupo la
mayor parte de las leyes estadsticas

de las ciencias sociales, por ejemplo


las que vinculan enfermedades
virsicas y contagio (Salud), o
pobreza e ndice de mortalidad
infantil (Sociologa), ausencia de
imagen paterna y anorexia u
homosexualidad (Psicologa), etc.
Esta diferencia en las leyes es muy
importante porque en las
explicaciones nomolgicodeductivas, dndose por verdaderas
las leyes de probabilidad mxima que
cubren la totalidad de los casos,
puede inferirse el explanandum del
explanans como una conclusin
deductiva. En cambio, como en el
explanans del modelo estadsticoinductivo, las leyes probabilsticas no

cubren la totalidad de los casos -hay


un cono de sombra en donde estn los
que no siguen la ley-, aun siendo este
explanans verdadero, no hay garanta
de verdad para el explanandum. Esta
situacin es la que se indica con la
doble raya que antecede al
explanandum en el esquema de las
explicaciones estadstico-inductivas
(EI).

ACTIVIDAD 1. ACERCA DE LOS


DISTINTOS MODELOS DE
EXPLICACIN
En los siguientes ejemplos, determine
qu modelo de explicacin podra
aplicarse en cada caso y justifique su
eleccin:

a. El enfermo se cur porque lo trataron


con penicilina.
b. Juan contrajo hepatitis por usar
jeringas contaminadas.
c. Mara compr un auto para dormir
dos horas ms a la maana.
d. Ernesto choc con el auto porque
manejaba alcoholizado.
e. Los huesos sirven para que el cuerpo
se mantenga erguido.
f. Las golondrinas emigran en otoo
porque necesitan vivir en lugares
clidos.
g. El Papa convoc a un Concilio
porque quiere reformar el ritual de la
misa.
h. La babosa vive en lugares oscuros y
tiene hbitos nocturnos porque no tiene

caparazn.
i. El supermercado X vende la gaseosa
por debajo del precio de costo porque
quiere eliminar a la competencia.
j. El corazn late porque la mdula
determina su ritmo y fuerza.
k. Juan se enferm de los pulmones
porque fumaba mucho.
l. Mi vecino perdi su fortuna porque es
adicto a las carreras de caballos.
ACTIVIDAD 2. MODELOS DE
EXPLICACIN CIENTFICA. ACERCA
DE LA EXPLICACIN
NOMOLGICO-DEDUCTIVA
Hemos visto que la explicacin
nomolgico-deductiva se ajusta mejor a
los casos de la Fsica. En el artculo que
se encuentra a continuacin, se presenta

el especial comportamiento del agua en


relacin con la Ley general de expansin
trmica (la materia se dilata cuando se
calienta y se contrae cuando se enfra)
que presentara un comportamiento
distinto. Luego de leer atentamente el
artculo:
a. Seale cul sera el explanandum y el
explanans en el caso presentado del
comportamiento del agua a distintas
temperaturas.
b. Determine si el explanandum
menciona una regularidad o un caso
particular.
Por qu el agua no obedece a la
Ley general de expansin trmica?
Con pocas excepciones, las formas de

la materia -slidos, lquidos, gases y


plasmas- se dilatan cuando se
calientan y se contraen cuando se
enfran. En los slidos, por lo
general, esos cambios de volumen no
son muy notables, pero existen: las
lneas telefnicas, por ejemplo, se
estiran y cuelgan ms en un da clido
que en uno de invierno; los lquidos,
por su parte, se dilatan en forma
aprecia-ble al aumentar su
temperatura; pero el agua tiene
comportamientos notables: a
temperaturas cercanas a su punto de
congelacin, a menos de 0 grados
cuando el agua ya es hielo macizo, su
volumen es bastante mayor y su
densidad es menor (por esto el hielo

flota en el agua). Al calentarse el


agua, hasta que su temperatura llega a
4 grados, contina contrayndose.
Slo a temperaturas ms altas se
comienza a dilatar y la expansin
contina hasta llegar al punto de
ebullicin.
Revista Muy interesante. N 290 (2009,
diciembre).

ACTIVIDAD 3. LA ESPECIFICIDAD
DE LAS CIENCIAS SOCIALES. LA
MODALIDAD EXPLICATIVA COMO
CRITERIO DE CLASIFICACIN DE
LAS CIENCIAS
Elster plantea que hay diferentes
modalidades de explicacin: la causal,

la funcional y la intencional. En su libro


El cambio tecnolgico[21] propone el
siguiente esquema que solamente
agregamos a los fines de visualizar la
distincin entre conducta intencional y
conducta racional.

En el texto, a continuacin, Elster dice


que puede haber intencionalidad sin
racionalidad.
A. Lea el texto.

B. Explique con sus palabras por qu


puede haber intencionalidad sin
racionalidad.
C. Elabore tres ejemplos de conducta
intencional-racional.
Intencionalidad y racionalidad
segn Jon Elster
Puede haber intencionalidad sin
racionalidad? O racionalidad sin
intencionalidad? [...] Evidentemente
todo depende de cmo definimos el
concepto de racionalidad. Si
solamente queremos decir
"adaptacin en el sentido de
maximizacin local", hemos visto que
puede haber racionalidad no
intencional. Pero cualquiera sea el

modo en que definimos la


racionalidad, creo que debera
reservarse para los casos en que tiene
poder explicativo. Es decir que nunca
habra que caracterizar una creencia,
una accin o un modelo de conducta
como racional a no ser que se est
dispuesto a afirmar que la
racionalidad explica que lo que se
dice es racional. [...] Para los
propsitos presentes es suficiente
observar que la racionalidad
mnimamente implica consistencia de
metas y creencias. Para calzar una
cua entre intencionalidad y
racionalidad, debemos demostrar que
puede haber deseos inconsistentes y
creencias inconsistentes. Con

respecto a las creencias


inconsistentes, demostrar su
posibilidad mediante una historia
sobre Niels Bohr, que cierta vez tena
una herradura sobre la puerta. Cuando
se le pregunt si la haba colocado
all porque crea que le traera suerte,
contest: "No, pero me dijeron que
traen suerte incluso a quienes no
creen en ellas". Arreglando un poco
la historia, resulta lo siguiente:
1) Niels Bohr cree "La herradura no
me traer suerte".
2) Niels Bohr cree "Las herraduras le
traen suerte a quienes no creen que
les traer suerte".
Aunque las creencias entre comillas

son consistentes entre s, ambas no


pueden ser ciertas y ser credas (por
Bohr). Pero un sistema de creencias
es consistente solamente si existe un
mundo posible en el que son todas
ciertas y credas. Si a favor del
argumento, suponemos que Bohr no
estaba haciendo una broma y que en
realidad coloc una herradura sobre
la puerta porque quera suerte y crea
aunque inconsistentemente que le
traera suerte, tenemos el caso de una
accin claramente irracional y, sin
embargo, explicada intencionalmente.
[22]

ACTIVIDAD 4. DIMENSIN
EXPLICATIVA Y DIMENSIN

PREDICTIVA. EL HOMBRE NO
LOGR CUMPLIR SU GRAN
ILUSIN: PREDECIR EL FUTURO
Despus de leer el siguiente texto,
realice la actividad propuesta a
continuacin:
Eduardo Flichman (1932-2005),
argentino, fue un fsico y filsofo de
la ciencia, dedicado a la
investigacin y docencia en el mbito
de la epistemologa y la enseanza de
las ciencias. En una nota periodstica
publicada en el diario Clarn,
Flichman dice: "la inquietud por
predecir la conducta humana
todava tiene ms interrogantes que
certezas". Para l, si pudisemos

predecir el futuro, si pudisemos


saber con antelacin lo que nos va a
pasar, dejara de tener sentido el
concepto de libertad y los hombres
estaran condenados a seguir un
destino. Y proporciona un ejemplo:
"se pueden estudiar las constantes
en las variaciones del trnsito y
determinar que a las 7 de la tarde se
va a ir ms rpido por una calle
lateral que por la autopista porque
sta ltima est colapsada". Pero
cuando se dan a conocer los
resultados de la investigacin, ya ese
conocimiento altera los
comportamientos de los conductores
y, si todos eligen ir por las calles
laterales, entonces resultar falsa la

prediccin y all se ir mucho ms


lento y ms rpido por la autopista.
En ciencias sociales, expresa
Flichman, "el conocimiento modifica
las conductas".
Dada esta situacin:
"Frente a la prediccin de una epidemia
de gripe A durante el mes de julio de
2009, en Argentina se tomaron medidas
preventivas. La epidemia no se produjo
en los niveles previstos y eso llev a
algunos a pensar que se trat de una
falsa alarma destinada a sembrar miedo
en la poblacin y a aumentar la venta de
medicamentos".
Responda: El hecho de que no se haya
cumplido la epidemia en el grado

previsto, indica que los bilogos se


equivocaron en la prediccin?
ACTIVIDAD 5. ENFOQUE
COMPRENSIVISTA DE LAS CIENCIAS
SOCIALES
5.1 Teniendo en cuenta lo estudiado en
el enfoque comprensivista de las
ciencias sociales, ubique los trminos en
las columnas segn corresponda.
Trminos
Explicacin Comp
Por qu?
Hempel
Pluralistas
Popper
Dilthey
Cmo?

Positivistas
Descripcin
Busca explicar
Teleolgico
Monistas
Antipositivistas
Aristteles
Rechazo de las
ciencias naturales
como patrn metdico
Simmel
Intencionalidad

Galileo
Comte
Historia
Platn
Unidad de mtodo
Hermenutica
Busca comprender
Droysen
Reduccin del mtodo
de las ciencias
sociales
a las naturales
Causalidad

Geisteswissenschaften
Empata
Reduccionismo
Sentido
Vertehen
Weber
Tiempo
Situacin
Prediccin
Accin
Dray
5.2. Responda el siguiente cuestionario:
a. Qu pregunta se hacen los
partidarios de la comprensin en las
ciencias sociales y cul los de la
explicacin?

b. Realice un cuadro comparativo entre


las posiciones de Hempel, Popper,
Dilthey, Droysen y von Wright.
c. Cul es la diferencia entre monismo
metodolgico y pluralismo
metodolgico?
d. Von Wright sostiene que el problema
de la explicacin y la comprensin en
las ciencias se remonta a dos tradiciones
cientficas. Cules son esas dos
tradiciones y a qu problema se orienta
cada una? Indique, asimismo, con qu
corrientes epistemolgicas
contemporneas puede trazar una
correspondencia.
e. Por qu motivo los partidarios del
enfoque comprensivista rechazan el
monismo metodolgico? Qu razones

brinda Dilthey al respecto?


f. Quin introdujo la dicotoma
metodolgica entre "explicacin" y
"comprensin"?
g. Describa brevemente cul es la tarea
de la comprensin (hermenutica) como
mtodo cientfico a partir de lo expuesto
por Simmel, Dilthey, von Wright, Weber
y Echeverra.
h. Cul es la posicin de von Wright en
esta tensin entre explicacin y
comprensin?
i. Mencione cul sera el objetivo de la
ciencia para cada una de estas
posiciones.
5.3. Dadas las siguientes situaciones:
A. Supongamos que un alumno se copia
en un examen y es descubierto por el

profesor.
De acuerdo con el modelo
comprensivista de von Wright,
responda:
a. Qu deberamos preguntarnos?
b. Cmo daramos cuenta de ese hecho
temporal y causalmente?
c. Cmo puede relacionar este hecho
con los conceptos de intencin, accin,
libertad y responsabilidad?
d. Se podra brindar una explicacin
que siempre fuese la misma para ese
hecho?
B. Supongamos ahora que una piedra
cae desde una montaa y que muere una
persona al ser golpeada por esa piedra.
Conteste las mismas preguntas
anteriores.

5.4. Dada la siguiente situacin:


Supongamos el caso de un conocido
hecho: el incendio en el centro
comercial de Paraguay. All mueren ms
de cien personas. Se hace la
investigacin judicial y se solicita la
opinin de distintos expertos para
comprender por qu o cmo murieron
esas personas.
A. De acuerdo con el modelo de van
Fraassen, qu respondera...
a. ... el perito mdico
b. ... el fiscal que investig el caso
c. ... un ingeniero
d. ... un urbanista
e. ... la polica
f. ... el abogado defensor
g. ... el resto del pblico asistente al

centro comercial, pero que no fue


alcanzado por el incendio
h. ... el dueo del centro comercial
B. Segn van Fraassen, todas esas
respuestas son legtimas?, de qu
depende su legitimidad?
5.5. Dada la siguiente situacin:
Imagine que usted debe realizar una
investigacin sobre un autor ya fallecido
(filsofo, epistemlogo, socilogo,
jurista, poeta, arquitecto, economista,
etc.) y desea "comprender" su
pensamiento y "explicar" el mismo
mediante un escrito monogrfico para
que el pblico tambin lo conozca, o lo
conozca mejor.
De acuerdo con el modelo de la
comprensin (hermenutica) brindado

por los aportes de Simmel, Dilthey,


Schltz, Gadamer, Habermas, Vattimo,
Ricoeur, responda:
a. Qu debera hacer usted?
b. Qu datos debera investigar y de
qu manera?
c. Por qu debera indagar ciertos datos
que, en apariencia, no seran tiles?
d. Se llegara al mismo resultado de su
investigacin si no recabase toda esa
informacin? Justifique.

Introduccin

En Argumentos y teoras. Aproximacin


a la epistemologa de Carlos Asti Vera
y Cristina Ambrosini, encontramos que:
"[...] el criterio que distingue entre ciencias
naturales y sociales es menos firme que el que
diferencia a las ciencias formales y a las
fcticas, ya que, a menudo no hay acuerdo
acerca de dnde ubicar la lnea divisoria [entre
un tipo de ciencia y otro], lo que tambin afecta
a las consideraciones sobre los mtodos a
emplear." (2009, 189)

Esta cuestin de la metodologa de las


ciencias fcticas comienza a presentarse
como problemtica hacia mediados y
fines del siglo XIX y principios del XX
con las consideraciones realizadas por
Johann Gustav Droysen (historiador
alemn, 1808-1884) y,
fundamentalmente, por Wilhelm Dilthey
(filsofo alemn, 1833-1911). Hasta ese

entonces se entenda que el mtodo de


justificacin, modelo en las ciencias
fcticas, era el correspondiente a las
ciencias naturales (el inductivista y
posteriormente, ya en el siglo XX, el
hipottico-deductivo), y en ese sentido,
las ciencias sociales se subsuman bajo
dicho modelo. Esta concepcin, en
lneas generales, se la conoce bajo la
denominacin de monismo
metodolgico.[23]
La problemtica de la distincin
(pluralismo metodolgico)[24] o no
(monismo metodolgico) entre los
mtodos de la ciencias fcticas trae
aparejada la cuestin de si estas
ciencias (tanto naturales como sociales)
explican -causalmente-o describen -para

comprender. Y aqu nuevamente nos


encontramos con estas dos posiciones
epistemolgicas: monismo y pluralismo.
Al inicio del captulo 6 de Asti Vera y
Ambrosini (2009. 214), se indica que en
las ciencias fcticas hay una diferencia
entre explicar y describir, dando cuenta
de que la primera responde a la pregunta
del "porqu" y la segunda a la pregunta
del "cmo".
As, si nos preguntramos cul sera el
objetivo de la ciencia: explicar o
describir?, la posicin mayoritaria
dentro de la comunidad cientfica se
inclinara por la afirmacin que la
ciencia debe explicar y no describir,
incluso frente a las diferencias
existentes entre ciencias naturales y

ciencias sociales o humanas o del


espritu.
Por otra parte, el campo del
conocimiento social estuvo dominado
por el empirismo lgico hasta la
aparicin de un conjunto de teoras
epistemolgicas que en los ltimos
treinta aos cambiaron el panorama
rechazando la idea de que pudiera haber
observaciones tericamente neutrales.
Este distanciamiento con el criterio de
neutralidad en el campo de la ciencia
natural permiti que emergieran con
renovada vitalidad tradiciones ya
existentes como la fenomenologa y la
hermenutica; esta ltima tal como la
desarrollaron Hans-Georg Gadamer
(filsofo alemn, 1900-2002) y Paul

Ricoeur (filsofo francs, 1913-2005)


siguiendo y recreando a Dilthey. Y por
otra parte, adquiri cierta relevancia la
teora crtica representada por Jrgen
Habermas (filsofo alemn, n. 1929). Lo
ms importante de la situacin de la
epistemolga de los ltimos aos es la
consideracin de la ciencia como una
actividad interpretativa y, entonces, los
problemas de significado y
comunicacin adquieren relevancia en la
epistemologa.

1. El problema de la
"explicacin" en las
ciencias fcticas

Particularmente respecto del problema


de la explicacin en las ciencias
sociales, Carl Hempel (filsofo,
epistemlogo y lgico germanoestadounidense, 1905-1997) en La
explicacin en la ciencia y en la
historia (1981), sostiene que los dos
tipos bsicos de explicacin cientfica,
el modelo nomolgico-deductivo y el
modelo estadstico-inductivo, sirven de
soporte al modelo de explicacin
caractersticamente histrico. Y
defiende que la explicacin histrica
aspira, adems, a demostrar que los
hechos no ocurren por azar, sino que
poda esperarse su ocurrencia en vista
de ciertos antecedentes o condiciones
simultneas (von Wright, 1979. 44) y si

no podemos formular leyes generales en


las explicaciones histricas, eso se debe
a la excesiva complejidad de tales leyes
y a la insuficiente imprecisin con que
las conocemos. De esta manera, las
explicaciones histricas constituyen
esbozos de explicacin (von Wright,
1979. 45).
Algo similar sostiene Karl Popper
(socilogo y espistemlogo austrobritnico, 1902-1994), representante
tambin del modelo de cobertura legal
(nomolgico-deductivo), quien afirma
que la razn de que las leyes no sean
formuladas en las explicaciones
histricas es que ellas son demasiado
triviales para merecer una mencin
explcita, y que las damos

implcitamente por supuestas (von


Wright, 1979. 45).
Sin embargo, la cuestin acerca de si la
ciencia, y particularmente las ciencias
sociales, deben explicar o describir no
est zanjada.
Respecto de esta cuestin, Georg
Henrick von Wright (filsofo finlands,
1916-2003), en su obra Explicacin y
Comprensin (1979), emprende el
tratamiento de la explicacin en la
historia y en las ciencias sociales, y
analiza las diferencias en los mtodos
explicativos entre las ciencias humanas
y las ciencias naturales. Desde el punto
de vista metodolgico, se suelen sealar
dos actitudes con relacin al problema
del mtodo, lo que se llama monismo

metodolgico (hay un solo modelo tanto


para las ciencias naturales como para
las sociales), y lo que se denomina,
sensu contrario, pluralismo
metodolgico.
En este sentido, en la base de la
discusin entre explicacin y
comprensin (descripcin) est la
cuestin de dnde reside el origen de la
controversia. Hempel indica que los
factores que han estimulado la
investigacin cientfica son
fundamentalmente dos: "[...] Una es el
deseo persistente del hombre por
mejorar su posicin estratgica en el
mundo por medio de mtodos
confiables para la prediccin y, cuando
sea posible, el control de los

acontecimientos. [...] Pero adems, una


segunda motivacin es la insaciable
curiosidad intelectual del hombre [...]"
(Hempel, 1981).
1.1. ORIGEN DE LA CONTROVERSIA
Recordemos que la dicotoma existente
entre explicacin y comprensin hunde
sus races en la historia de las ideas y se
relaciona con el problema de si la
construccin terica es intrnsecamente
un mismo gnero de empresa tanto en las
ciencias naturales como en las ciencias
sociales, humanas o del espritu.
von Wright sostiene que el problema
parte de dos tradiciones cientficas en la
historia de las ideas. Una es la

aristotlica y la otra la galileana que se


remonta a Platn. La primera se vincula
a los esfuerzos del hombre por
"comprender" las cosas
"teleolgicamente", es decir,
comprender las cosas como tendientes a
un fin, a un objetivo; y la segunda por
"explicarlas causalmente" (von Wright,
1979. 18). Y considera que cuando el
filsofo se cuestiona el tipo de
conocimiento, se encuentra con estas dos
tradiciones que difieren en el
planteamiento de las condiciones a
satisfacer por una explicacin cientfica.
En el mismo sentido, en la historia de la
ideas, y particularmente de las ciencias,
nos encontramos frente a dos corrientes
epistemolgicas fuertes, la positivista y

la antipositivista. Y segn von Wright,


la positivista, cuyos representantes
principales son Auguste Comte
(socilogo francs, 1798-1857) y John
Stuart Mill (filsofo y economista
ingls, 1806-1873), queda vinculada a
la tradicin galileana a travs de su
monismo metodolgico, y por otro lado,
la antipositivista, cuyas figuras ms
significativas son Dilthey y Max Weber
(filsofo, economista y socilogo
alemn, 1864-1920), entre varios otros,
representa una tendencia mucho ms
diversificada y heterognea que el
Positivismo, rechazando el monismo
metodolgico y rehusndose a
establecer como patrn metodolgico a
las ciencias naturales exactas como

ideal regulador nico y supremo de la


comprensin racional de la realidad.
Para caracterizar a esta ltima corriente,
von Wright utiliza el nombre de
hermenutica -cuestin que trataremos
en detalle en los prximos pargrafos- y
la vincula con la tradicin aristotlica
(von Wright, 1979. 23-24).
1.2. EL PROBLEMA DEL MTODO Y
EL REDUCCIONISMO
Antes de incursionar en las
concepciones de la corriente
"comprensivista" y en su enfoque
hermenutico, hay que tener en cuenta
que, por un lado, desde el Positivismo,
se sostiene la unidad de mtodo

tendiendo a subsumir el mtodo de las


ciencias sociales al de las naturales;
establecindose como ideal
metodolgico las ciencias fsicomatemticas; ponindose nfasis en la
explicacin y prediccin; dando cuenta
de que la explicacin es causal; y
rechazando a las explicaciones finalistas
o teleolgicas como acientficas. Y
mientras, por otro lado, el
antipositivismo, la otra posicin, tiende
a sostener un contraste entre, por una
parte, las ciencias que al modo de la
fsica, la qumica o la fisiologa, aspiran
a generalizaciones sobre fenmenos
reproducibles, y por otro parte, las
ciencias que, como la historia, buscan
"comprender" las peculiaridades

individuales y nicas de sus objetos,


impugnando el enfoque positivista de la
explicacin.
Las ciencias sociales o humanas no
pueden, sostiene Dilthey, pretender la
comprensin de la vida a travs de
categoras externas a ella -como lo
pretende el Positivismo-, sino a travs
de categoras intrnsecas, derivadas de
ella misma, por tal motivo no deben
explicar, sino comprender y no lo
pueden hacer mediante los mtodos de
las ciencias naturales sino mediante los
suyos propios. Es decir, para "explicar"
(entre comillas porque Dilthey utiliza
"comprender") la vida, para comprender
las estructuras del sentido de la vida, no
podemos usar categoras, conceptos de

las ciencias naturales como leyes


lgicas o razonamientos deductivos o
inductivos, porque todo esto son
construcciones formales realizadas por
el hombre, y no son "la vida", es decir,
no pertenecen a la categora de la vida
esas categoras cientficas, ya que son
slo objetivaciones que realiza el
hombre para explicarla, pero no son
categoras propias de ella. Las
categoras propias de la vida tienen que
ver con lo que Dilthey llama "vivencias"
y que son, a grandes rasgos, lo que nos
ocurre, y lo que nos ocurre
histricamente. Son los acontecimientos
de la vida, el querer, el sentir y el
representar. Y por eso, para Dilthey, la
comprensin en las "ciencias del

espritu" se da porque la vida se


exterioriza en textos, que dan cuenta de
las vivencias del que lo escribi y,
entonces, nos permiten comprender,
adems del texto, a su autor.

2. La "comprensin" en las
ciencias sociales
La corriente antipositivista, por su parte
y de diversas maneras, sostiene que el
objetivo de las ciencias naturales es
"explicar", y el de la historia y las
ciencias sociales, es "comprender" los
fenmenos que ocurren en su mbito; y
es el mismo Droysen el que introdujo tal
dicotoma metodolgica y acu los

nombres de explicacin y comprensin.


Estas ideas metodolgicas fueron luego
desarrolladas con mayor profundidad y
sistematizadas por Dilthey, quien
rechaza la tendencia de fundar un
conocimiento humano siguiendo los
procedimientos de las ciencias
naturales. Es el mismo Dilthey quien
para designar todo el dominio de
aplicacin del mtodo de comprensin
se sirvi del trmino "ciencias del
espritu" (Geisteswissenschaften) que
conocemos tambin por ciencias
humanas o ciencias sociales, y en el
habla inglesa por Moral science (von
Wright, 1979. 24).
El uso cotidiano del lenguaje no hace
una distincin aguda entre los trminos

"explicar" y "comprender", y as se
podra decir que, cualquier tipo de
explicacin proporciona una
comprensin de las cosas. Pero
"comprensin" cuenta adems con una
connotacin psicolgica de la que
carece el trmino "explicacin". Se
considera que la comprensin, como
mtodo caracterstico de las ciencias del
espritu, es segn Simmel (filsofo y
socilogo alemn, 1858-1918) una
forma de empata o recreacin en la
mente del estudioso de la atmsfera
espiritual, pensamientos, sentimientos y
motivos, de sus objetos de estudio (von
Wright, 1979. 24).
Sin embargo, no es esta la nica
caracterstica que distingue a la

comprensin de la explicacin. Von


Wright sostiene que la comprensin se
encuentra ligada adems con la
"intencionalidad" de una manera en que
la explicacin no lo est. Se
comprenden los objetivos y propsitos
de un agente, el significado de un signo
o de un smbolo, el sentido de una
institucin social, de un rito religioso.
Esta dimensin intencional, sostiene el
mismo von Wright, ha llegado a jugar un
papel relevante en la discusin
metodolgica ms reciente a travs de la
hermenutica (von Wright, 1979. 24-28).
El gran objetivo de Dilthey consisti en
desarrollar una metodologa apropiada
para el "entendimiento" (Verstehen) de
las obras humanas, que eluda el

reduccionismo y mecanicismo de las


ciencias naturales, ya que la vida debe
ser entendida a partir de la propia
experiencia de vida.
En tal sentido es central para Dilthey
esta distincin entre ciencias naturales y
ciencias del espritu. Las primeras
descansan en el concepto de "fuerza"
propuesto por la fsica y en las
matemticas; las ciencias humanas, en
cambio, se apoyan en el concepto de
"sentido" y en la historia. Y as, el
concepto clave de las ciencias del
espritu es el de "entendimiento" o
"comprensin" (Verstehen).
Si en sociologa (ciencia social), Comte
era el principal representante del
positivismo, Weber se inclinaba hacia el

lado opuesto y de una manera ms slida


que Comte. Uno de los elementos que
aparecen en la concepcin weberiana
del mtodo cientfico es el de
"comprensin" (Verstehen). Esta
comprensin no significaba una
identificacin completa con el sujeto
investigado, sino que, para comprender,
por ejemplo las obras de Shakespeare,
yo no necesito ser Shakespeare, pero s
tengo que poder de alguna manera
concebir qu puede estar pensando
Shakespeare en una determinada
situacin. As, comprender es la
capacidad que tiene cualquier ser
humano de entender a otro ser humano
en una situacin, aunque l no haya
vivido exactamente esa situacin. Es,

aunque no en el ms completo sentido,


lo que comnmente conocemos por
"ponerse en el lugar del otro" para
comprender una actitud, una accin, etc.
El sentido propio de la comprensin (o
entendimiento) siempre se halla en un
contexto de horizonte que se extiende
hacia el pasado y el futuro. La
historicidad y la temporalidad son
dimensiones inherentes e inevitables de
toda comprensin. Por ejemplo, si
quisiramos comprender a Scrates y su
pensamiento, lo que deberamos hacer
es tender un puente con el pasado y con
el presente, es decir, no podramos
comprenderlo plenamente si slo lo
analizramos desde la actualidad, pero
tampoco lo podramos comprender

plenamente si slo lo analizramos en su


contexto, con lo cual hay que tender un
puente entre pasado y presente, un
puente que nos permita abrir las
posibilidades del pensamiento de
Scrates. De esta manera, el todo recibe
su sentido de las partes y las partes slo
pueden comprenderse en relacin con el
todo, conformando como dira Gadamer
un crculo hermenutico. Como sucede
con una obra literaria, no podemos
comprenderla leyendo slo un captulo
(una parte), pero tampoco podemos
comprenderla completa si cada captulo
no le da un sentido unificador a la obra.
El sentido es histrico, dinmico y
contextual, es siempre parte de una
determinada situacin. La interpretacin

siempre remite a la situacin en la cual


se halla el intrprete, pero esto no
significa que el sentido sea subjetivo y
nos lleve al relativismo; el sentido es
una percepcin de una relacin real
dentro de un nexo anterior a la
separacin sujeto-objeto en el
pensamiento (Echeverra, 1993. 203205).
La tradicin antipositivista procura
sustituir las nociones cientficas de
explicacin, prediccin y control por las
interpretativas de comprensin,
significado y accin.
Por su parte, von Wright sostiene que la
metodologa de las ciencias humanas
comenz gradualmente a atraer la
atencin de los filsofos analticos[25]

(positivistas y neopositivistas
vinculados por la pretensin de excluir
por completo a la metafsica de las
ciencias), y que la filosofa analtica de
la ciencia se vio inmersa en el campo de
batalla de la metodologa positivista y
antipositivista (von Wright, 1979. 29).
Es as que hasta aproximadamente los
aos 60 rein una coincidencia general
en cuanto a que el modelo explicativo
causal suministraba el marco de
referencia idneo para el estudio de los
fenmenos sociales, pero luego, y hasta
cerca de 1980, la alternativa
interpretativa, el modelo de la
comprensin, comenz a ganar adeptos
en los pases de habla inglesa. Los
trabajos de varios filsofos de

raigambre analtica, positivista y


neopositivista, reflejan el creciente
inters, en el seno de la filosofa
analtica, por el concepto de accin que
est vinculado con la intencionalidad.
Tal es as que, William Dray
(canadiense, filsofo de la historia, n.
1921), en Leyes y Explicacin en
Historia, expresa que las explicaciones
de la historia son cientficas y
racionales, a diferencia de lo que
sostena Hempel, aunque no son ciencia
de leyes generales. Dray sostiene que la
historia explica y lo hace en el sentido
de mostrar que una accin fue el
proceder adecuado teniendo en cuenta
los antecedentes. As aborda la pregunta
de cmo fue posible? y no la pregunta

de por qu? Von Wright indicar,


entonces, que este giro del "porqu" al
"cmo" lo sita cerca de la tradicin
hermenutica (von Wright, 1979. 45-52).
De esta manera, en virtud del inters de
algunos epistemlogos positivistas por
la explicacin en las ciencias sociales,
von Wright trata de tender un puente
entre "explicacin" y "comprensin"
demostrando que, tanto como sostenan
Hempel, por un lado, o Dilthey, por el
otro, no son incompatibles, sino
complementarias. Influenciado por las
consideraciones filosficas del segundo
Ludwig Wittgenstein (filsofo y
lingista austro-britnico, 1889-1951) y
la filosofa neowittgensteniana, y
tambin por la hermenutica, von Wright

postula que la filosofa analtica es afn


a la hermenutica a partir de los
conceptos de significado,
intencionalidad, interpretacin y
comprensin, y por compartir, adems,
la preocupacin por la metodologa y la
filosofa de la ciencia. Sostiene en este
sentido que la comprensin en el
lenguaje de la filosofa hermenutica
debera diferenciarse de la empata,
afirmando que se trata de una categora
antes semntica que psicolgica (von
Wright, 1979. 53).
As von Wright indica que las
explicaciones causales son tpicamente
cuasi-teleolgicas: explicaciones
susceptibles de formulacin en trminos
teleolgicos pero cuya validez depende

de la efectividad de conexiones nmicas


(leyes). Las explicaciones de este
gnero suelen responder a las preguntas
sobre cmo es? o cmo llega a ser
posible algo? Y estas "explicaciones"
tienen un lugar predominante en las
ciencias humanas, permitiendo
"comprender" lo que algo es o cul es la
razn de que algo ocurra (von Wright,
1979.108-109). Sealando de este modo
que los acontecimientos que resultan de
acciones bsicas tienen lugar cuando los
investimos de intencionalidad.
2.1. EL MODELO COMPRENSIVISTA
DE VON WRIGHT
De acuerdo con las diversas posiciones

epistemolgicas y metodolgicas que


antes tratamos, podemos decir que la
nocin de causalidad vale con ms
facilidad para el mbito de los
fenmenos naturales que en el de los
humanos. Sin embargo, von Wright
sostiene que si rastreamos el concepto
de causalidad, veremos que se origina
en nuestro conocimiento inmediato,
ntimo, de la accin humana. Es decir,
este concepto proviene de una
proyeccin de nuestras propias acciones
sobre el mundo natural. Nosotros,
cultural o histricamente,
experimentamos la accin, y la nocin
de causa surge por una especie de
proyeccin de nuestras propias acciones
al mundo natural. As, si nosotros no

acturamos, no tendramos el concepto


de causa.
De esta manera, en lugar de pensar que
yo soy la causa por la cual, supongamos,
"x" muri por mi accin o mi accin fue
la causa de la muerte de "x"; la
causalidad en la naturaleza deriva del
"saber que yo puedo" matar a una
persona. O sea, invierte el orden con
respecto a la idea que previamente uno
poda tener.
Todos los comentarios de von Wright
estn dirigidos al problema de mostrar
que la conducta humana es algo
completamente distinto de los hechos
que nos encontramos en el mundo
natural, y hasta formula hiptesis de
cmo en el mbito humano puede haber

procesos causales de tipo retroactivo, en


los que el efecto es anterior a la causa.
Por ejemplo, se piensa que un cierto
proceso cerebral tiene que ser de alguna
manera anterior a un movimiento
muscular, porque se supone que el
cerebro ordena al brazo, por ejemplo, a
extenderse, pero von Wright piensa que
es al revs. Es el levantar el brazo, el
extenderlo, lo que produce el fenmeno
cerebral, pero lo produce antes. Es
decir, no es que la causa de que apret
el dedo haya sido el fenmeno cerebral,
sino que el fenmeno cerebral se
produce porque "voy a" apretar el dedo,
o "a" extender el brazo. Y sostiene que
el brazo se extiende, o el dedo se aprieta
porque "yo decid hacerlo". Yo lo que

decido es extender el brazo, es una


decisin que tomo, decido extender el
brazo y lo extiendo. Con lo cual, en el
momento que decid hacerlo, y lo hago
efectivamente, ese hecho, para von
Wright, es primario causalmente, no
temporalmente, y no lo es temporalmente
porque l reconoce que no hubiera
extendido el brazo si no hubiese
producido un cierto estado cerebral
antes. As, causalmente, primero est la
decisin, segundo, el proceso cerebral y
tercero, la accin. Pero temporalmente,
primero est el proceso cerebral y
segundo la accin.
Esta cuestin se emparenta directamente
con el tema de la "intencionalidad" y la
"responsabilidad". Un ser humano

directamente levanta el brazo, aprieta el


botn, etc., y al hacer esto, l es
responsable de su accin, y slo podr
ser responsable si la decisin la toma l.
Y esto es lo que le lleva a decir que el
fenmeno A (proceso cerebral) se da
antes que B (apretar el dedo), pero B
(apretar el dedo) es la causa de A
(proceso cerebral).
Un concepto fundamental en von Wright
es el concepto de accin. La
caracterizacin de este concepto
consiste en la produccin de cambios
que no hubieran tenido lugar de no haber
mediado nuestra intervencin. Por
ejemplo, en el caso de un crimen,
acusamos a una persona de haberlo
cometido porque no habra tenido lugar

la muerte de la otra persona de no haber


mediado la accin de quien,
supongamos, accion el gatillo. Pero una
accin es tanto la accin propiamente
dicha como la omisin, que es una
accin negativa.
As, la "explicacin" en el caso de las
acciones humanas es completamente
distinta de la explicacin de los
fenmenos de tipo natural, porque en el
caso de las acciones no voy a buscar
leyes, y, en el caso de los fenmenos
naturales, no puedo adjudicarles ni
intencin, ni decisin, ni
responsabilidad.
von Wright sostiene entonces que lo ms
adecuado para "comprender" las
acciones humanas no es el esquema de

Hempel (leyes), sino el de Aristteles


(interpretacin), que es el esquema que
se conoce como silogismo prctico,
esquema sobre las explicaciones
intencionales que ya se ha visto en la
bibliografa.[26]
De esta manera, es posible afirmar que,
en el modelo de "explicacin
comprensivista" de von Wright decir
que el concepto de causalidad supone el
de libertad es correcto en el sentido de
que nicamente a travs de la idea de
hacer cosas se llega a "comprender" las
ideas de causa y efecto (von Wright,
1979. 106).
2.2. EL MODELO PRAGMTICO DE
VAN FRAASSEN

Ya se haba anticipado que los modelos


de explicacin en ciencias sociales se
vinculaban fundamentalmente con el
concepto de comprensin de origen
hermenutico, y habamos indicado que
en la comprensin eran inescindibles
nociones tales como intencionalidad y
contexto, como tambin las de
temporalidad, significado e
interpretacin.
Precedentemente se ha tratado la
cuestin del "modelo de comprensin"
vinculado a la intencionalidad, lo que se
denomin el modelo comprensivista de
von Wright. Se iniciar ahora el
recorrido al modelo pragmtico de Bas
van Fraassen (filsofo holands, n.
1941), vinculado fundamentalmente con

el contexto, y aunque este epistemlogo


no se vincule directamente con el
comprensivismo, podemos observar
ciertas semejanzas con esa posicin
metodolgica.
Van Fraassen considera los problemas
epistemolgicos ms all, no
exclusivamente desde el punto de vista
del anlisis de las relaciones lgicas y
relaciones conceptuales (dimensin
sintctica y semntica), sino tomando en
cuenta lo que se denomina la dimensin
pragmtica. Recordemos que la
dimensin pragmtica toma en cuenta los
usuarios o las circunstancias en las
cuales se usan las expresiones.
Con Dilthey decamos que el sentido era
contextual, y la interpretacin se remite

siempre a la situacin en la cual se halla


el intrprete. La circularidad del
entendimiento (crculo hermenutico)
tiene otra importante consecuencia, no
existe realmente un punto de partida
verdadero para el entendimiento. Ello
significa que no es posible concebir un
entendimiento carente de presupuestos.
Todo acto de entendimiento, de
comprensin, tiene lugar en el interior
de un determinado contexto u horizonte.
Ello es igualmente vlido para las
explicaciones cientficas. Estas siempre
requieren de un marco de referencia.
As, comprendemos slo por referencia
a nuestra experiencia.
Van Fraassen piensa que es intil hacer
una caracterizacin de las explicaciones

que no tenga en cuenta las situaciones en


las cuales las explicaciones se
producen, es decir, su contexto. As, si
yo pregunto por qu (algo que Hempel
dira que genera una explicacin) Adn
comi la manzana?, por lo pronto, el
tipo de respuesta que voy a admitir
como adecuada tiene que ver con
determinado contexto. As, una
explicacin que podra ser
perfectamente vlida para un contexto,
podra estar fuera de lugar en otro. Lo
que sostiene van Fraassen es que una
pregunta, en primer lugar, siempre se
refiere a un tpico, y el tpico ser el
explanandum. Y as, la propia
suposicin de que el explanandum es
verdadero depende de una cuestin

contextual.
Por otro lado, van Fraassen sostiene que
puede haber respuestas distintas a la
misma pregunta de "por qu?" e incluso
puede haber respuestas completamente
distintas, pero todas vlidas, cada una
en relacin con un contexto. Por
ejemplo, en un accidente automovilstico
muri Juan X. Se hace una investigacin
judicial y se solicita la opinin de
distintos expertos para "comprender"
cmo o por qu muri Juan X. Las
respuestas variarn dependiendo de
quien las responda, as el mdico podr
responder "por un paro cardaco", el
mecnico "porque los frenos no estaban
en buenas condiciones y fallaron", y un
urbanista responder "porque los

rboles tapaban el semforo". Todas


estas respuestas son legtimas segn van
Fraassen, pero eso no quiere decir que
cualquier respuesta en cualquier
contexto sea legtima. Con lo cual, esto
significa slo que puede haber
explicaciones vlidas alternativas.
Van Fraassen indica que puedo explicar
un hecho A a partir de un hecho B, o
puedo hacer al revs, explicar B a partir
de A. Y en esto parece contradecir lo
que sostena Hempel respecto a que la
explicacin era unidireccional, es decir,
que se va de las leyes a las
consecuencias y no al revs. Pero, en
este caso, si uno acepta un determinado
contexto, es perfectamente legtima una
explicacin y no lo es la otra.

Una de las consecuencias que extrae van


Fraassen es que es completamente
equivocado pensar que las
explicaciones constituyen uno de los
objetivos esenciales de las ciencias, ya
que una explicacin no puede ser
juzgada como correcta o no sobre la
base de las caractersticas que Hempel
haba tomado en cuenta, relativizando de
esta manera el concepto de explicacin.

A modo de conclusin
Finalmente podemos decir que el punto
crtico se encuentra entonces en los
conceptos de mtodo y objetividad
propios de la ciencia moderna.
Actualmente, desde la epistemologa

comprensivista podemos destacar los


aportes de:
a) Alfred Schtz (socilogo y
filsofo austraco, 1899-1959),
para quien la comprensin es la
reconstruccin de las perspectivas
que los individuos tienen de la
sociedad. La accin social es
entendida como una vivencia
guiada por un plan o proyecto. La
interpretacin surge entonces de los
motivos del actor (Schtz, 1932),
pero el lmite es la imposibilidad
de generalizar y predecir.
b) Hans-Georg Gadamer aplica el
mtodo al anlisis de textos
histricos, y sostiene la naturaleza

histrica de la comprensin misma.


Sostiene que hay una tensin entre
la objetividad del texto y la
subjetividad del intrprete, y la
forma de resolver esta tensin es
mediante la fusin de horizontes,
[27]

es decir, lograr un acuerdo de


ajuste de nuestros prejuicios
(Gadamer, 1981). Segn Gadamer,
la etimologa del trmino
"hermenutica" significa
explicacin, y se trata de explicar
unos enunciados analizndolos
mediante otros enunciados. Es,
pues, el arte o la teora de la
interpretacin (Gadamer, 1998).

Para el filsofo alemn, "[...] la


hermenutica no constituye un
mtodo determinado que pudiera
caracterizar por ejemplo a un
grupo de disciplinas cientficas
frente a las ciencias naturales. La
hermenutica se refiere ms bien
a todo el mbito de comunicacin
infrahumana [...]" (Gadamer,
1998. 85).
c) Jrgen Habermas, por su parte,
destaca la capacidad de persuasin
como estrategia para la dominacin
a partir de la comprensin de las
significaciones dadas en la accin
comunicativa. La comprensin es
entendida como experiencia

comunicativa que implica una


actitud reflexiva y la posibilidad no
slo de intercambios de mensajes,
sino tambin para cambiar y formar
actitudes. As, la hermenutica se
vincula con la retrica, con la
argumentacin de tipo informal. En
el mismo sentido, Gadamer
sostiene que "el gran legado de la
retrica sigue influyendo en
puntos decisivos en relacin con
la nueva labor de interpretacin
de los textos". De manera que
retrica y hermenutica son
profundamente afines, y
particularmente, por ejemplo, las
clases de retrica de Melanchthon
[28]

(telogo alemn, 14971560)


ejercieron una influencia
determinante en la configuracin
del sistema escolar (Gadamer,
1998. 271-272).
d) Paul Ricoeur, en Del texto a la
accin (1969), tambin
problematizar el intento de
resolver la apora central de la
hermenutica, es decir, la
alternativa entre explicacin y
comprensin, apora que a su juicio
es infundada. l buscar la
complementariedad de las dos
actitudes, dado que la objetivacin
del discurso en la obra y el

carcter estructural de la
composicin, a lo cual se agregar
el distanciamiento mediante la
escritura, lo obliga a poner en tela
de juicio la oposicin recibida de
Dilthey entre comprender y
explicar.
Ahora bien, sera ciertamente ilusorio,
sostiene von Wright, creer que la verdad
reside inequvocamente en una de estas
dos posiciones opuestas
(explicacionista-comprensivista), sino
que existe una confrontacin de base, y
sta se funda en la eleccin de
conceptos primitivos bsicos para la
argumentacin en conjunto. Podra
calificarse esta eleccin de "existencial"
consistiendo en la opcin por un punto

de vista no susceptible de ulterior


fundamento (von Wright, 1979. 57).
Sin embargo, se puede considerar desde
Dilthey, que las ciencias del espritu y,
por ende, el mtodo de comprensin,
son epistemolgicamente anteriores a
las de la naturaleza, a las que por lo
dems abarcan, pues toda ciencia natural
es tambin un producto histrico. Pero
como ya hemos dicho anteriormente, la
cuestin an no est zanjada, y sigue en
pie precisamente por ser un problema
filosfico.
Bibliografa
Asti Vera, C. y Ambrosini, C. (2009).
Argumentos y teoras. Aproximacin a
la Epistemologa. Buenos Aires:
Educando.

Braithwaite, R. B. (1965). La
explicacin cientfica. Madrid: Tecnos.
Comesaa, J. M. (1998). Lgica
informal, falacias y argumentos
filosficos.
Buenos Aires: Eudeba. Dilthey, W. F.
(2003). La esencia de la filosofa.
Buenos Aires: Losada.
Dray, W. (1957) Ley y Explicacin en
Historia. Londres: Oxford University
Press.
Echeverra, R. (1993). El Bho de
Minerva. Santiago de Chile: Dolen
Ediciones.
Fernndez, C. (1967). Dilthey. Buenos
Aires: Centro Editor de Amrica Latina.
Gadamer, H. G. (1981). Hermenutica
como filosofa prctica. En La razn en

la poca de la ciencia. Madrid: Alfa.


Gadamer, H. G. (1998). El giro
hermenutico. Madrid: Ctedra.
Gadamer, H. G. (1992-3). Verdad y
Mtodo I y II. Salamanca: Sgueme.
Giddens, A. y Turner, J. (1995). La
teora social hoy. Buenos Aires:
Alianza.
Hempel, C. G. (1981). La explicacin en
la ciencia y en la historia. En Teora de
la historia. Mxico: Terra Nova.
Hempel, C. G. (1983). Filosofa de la
ciencia natural. Madrid: Alianza.
Ricoeur, P. (2003). Del texto a la
accin. Ensayos de hermenutica ii.
Buenos Aires: Fondo de Cultura
Econmica.
Schtz, A. (1973). El problema de la

realidad social. Buenos Aires:


Amorrortu.
Windelband, W. (1943). Historia
general de la filosofa. Mxico: Ateneo.
Wrigth, G. H. von (1979). Explicacin y
comprensin. Madrid: Alianza.

Temas de la Unidad
6.1. La Epistemologa del siglo XX:
problemas fundamentales
6.2. La concepcin epistemolgica del
Positivismo lgico
6.3. Las perspectivas falsacionistas:
Popper y Lakatos
6.4. La ruptura: del racionalismo de
justificacin al anlisis de la historia de
la ciencia
6.5. El pensamiento de Kuhn en
perspectiva
Bibliografa obligatoria
Unidad 6 en la Gua de Estudio de IPC,

producida por UBA XXI y editada por


Eudeba, a partir de la edicin de 2010.
Asti Vera, C. y Ambrosini, C. (2009).
Captulo 7. En Argumentos y teoras.
Aproximacin a la Epistemologa.
Buenos Aires: Educando.
BIBLIOGRAFA COMPLEMENTARIA
Echeverra, J. (1999). Captulos 1, 2, 3,
4, 5 y 6. En Introduccin a la
metodologa de la ciencia. La filosofa
de la Ciencia en el siglo XX. Madrid:
Ctedra.
Klimovsky, G. (1994). Captulos 21, 22
y 23. En Las desventuras del
conocimiento cientfico. Buenos Aires:
AZ.
presentacin de la Unidad

Creo que podran ustedes hacer algo


ms til para matar el tiempo que
malgastarlo con adivinanzas que no
tienen solucin".
"Ay! Si conocieras al Tiempo tan
bien como lo conozco yo", exclam
el Sombrerero, no hablaras de
malgastarlo y menos de matarlo! Se
trata de un tipo de mucho cuidado y
no de una cosa cualquiera."
"Me parece que sigo sin
comprenderle", dijo Alicia.
"Naturalmente que no me
comprendes!", dijo el Sombrerero

elevando orgullosamente la nariz.


"Con toda seguridad; ni siquiera
habrs hablado con el Tiempo!"
"Puede que no", contest Alicia con
cautela. "Pero s s", aadi
esperanzada, "que en las lecciones
de msica marco el tiempo a
palmadas".
"Ah!Ah!Eso lo explica todo!",
afirm el Sombrerero. "El Tiempo no
tolera que le den de palmadas [...]"
(Carroll, L. Alicia en el pas de las
maravillas)

Esta Unidad 6 corresponde a la lectura


del captulo 7 de la bibliografa y
supone un enfoque metaterico ya que
aqu se sealan algunos de los aspectos
centrales de la Epistemologa

contempornea, y reitera algunos de los


problemas abordados en las unidades
anteriores desde un ngulo diferente.
La expresin "la concepcin heredada",
mencionada en este captulo, para
referirse bsicamente a la epistemologa
del Crculo de Viena, se adjudica al
filsofo estadounidense Hilary Putnam
(n. 1926) y se identifica con las crticas
a esta epistemologa caratulada tambin
como "cientificista".
Encontrar en el captulo, una mencin a
los aportes del psiclogo suizo Jean
Piaget (1896-1980) identificando su
posicin con la llamada Epistemologa
gentica y algunos representantes de la
filosofa francesa que acentan el
importante papel de la historia de la

ciencia en el anlisis epistemolgico.


Para hacer explcitos los postulados
filosficos del llamado Positivismo
lgico, se centra la atencin en los
postulados del Crculo de Viena y la
extendida influencia que ejercieron sus
miembros sobre otros grupos de
cientficos y epistemlogos de la poca.
Un punto central dentro de los temas
tratados, y sobre el que ya se
presentaron varios aspectos, es la
posicin falsacionista a travs de las
figuras de Popper y de Imre Lakatos,
tomando en cuenta sus ideas acerca de la
produccin de teoras cientficas
aludiendo a la posicin del primero
como "realismo crtico" y, al segundo, a
partir de su propuesta de los "programas

de investigacin". Deber detenerse


especialmente en estos prrafos, en la
presentacin de las ideas fundamentales
de la epistemologa de Thomas Kuhn
que pueden ser vistas como una ruptura
respecto a las posiciones anteriores,
enfocadas en la bsqueda del mtodo
cientfico (monismo metodolgico) y en
el anlisis de la estructura interna de las
teoras cientficas para interesarse por
la historia de la ciencia y el proceso de
gnesis de las teoras cientficas. Las
nociones centrales de "paradigma",
"ciencia normal", "revolucin
cientfica", propias de esta posicin,
muestran un quiebre y una
transformacin de la concepcin de la
epistemologa centrada en el anlisis

lgico y en las estructuras formales de


las teoras.
Luego de la presentacin de la
epistemologa de Kuhn y el
sealamiento de algunas de las crticas
de las que ha sido objeto, se seala un
panorama del complejo entramado de
teoras en las que se mencionan distintas
estrategias para el anlisis de las teoras
cientficas. Debemos advertir que estos
temas no estn incluidos como temas del
programa y no son de lectura
obligatoria. A pesar de ello, como en
otros casos, recomendamos su lectura
para tener una comprensin cabal de las
posiciones presentadas.

Actividades de
aprendizaje
ACTIVIDAD 1. LA CONCEPCIN
HEREDADA DE LA CIENCIA
Uno de los puntos ms polmicos de la
posicin de Popper, en especial
respecto al monismo metodolgico y a
la propuesta de considerar "cientfico"
un mismo mtodo para todo tipo de
ciencias, aun en las ciencias sociales, es
el estricto criterio de demarcacin que
propone entre "ciencia" y
"pseudociencia" a partir de considerar
el criterio falsacionista como el nico
vlido para legitimar el conocimiento

cientfico. En sus anlisis de las teoras


cientficas, haciendo lugar con esto a un
debate todava vigente, Popper lleg a
la conclusin de que dos teoras
consagradas en el campo de las ciencias
sociales (el Marxismo y el Psicoanlisis
freudiano) eran suficientemente flexibles
como para acomodarse a cualquier
situacin histrica o de comportamiento
humano, dando la falsa sensacin de que
explicaban los hechos. Popper advierte
que una teora que parece explicarlo
todo en realidad no explica nada. Segn
Popper, una teora con una capacidad
explicativa genuina hace predicciones
"arriesgadas", que excluyen la mayor
parte de posibles resultados. El xito en
la prediccin es impactante slo hasta

donde el fracaso sea una verdadera


posibilidad. Popper compara esta
situacin con la forma, completamente
diferente, en que la comunidad cientfica
plante la comprobacin de la Teora
General de la Relatividad de Einstein.
La teora predeca que la luz, al
propagarse, debera sentir la accin de
la gravedad como cualquier forma de
energa y, en consecuencia, su
trayectoria debera curvarse al pasar
cerca de un cuerpo de masa elevada,
como es el caso del Sol. En
consecuencia, un observador que
apuntase su telescopio a una estrella, la
vera en posiciones distintas si lo rayos
de luz pasaban cerca del sol que si ste
no se encontraba en su camino. En 1919,

el astrnomo britnico Arthur Stanley


Eddington (1882-1944) llev a cabo
este experimento durante un eclipse y
comprob que los resultados estaban de
acuerdo con las predicciones de
Einstein y, en consecuencia, la Teora
General de la Relatividad fue aceptada.
Lo verdaderamente importante, en
trminos de Popper, no era el hecho de
que la observacin de Eddington
implicara la validez de la Teora
General de la Relatividad, sino que la
Teora General de la Relatividad era
susceptible de ser comprobada. Exista
un procedimiento bien establecido, de
acuerdo con el cual poda demostrarse
que una teora dada era falsa. Popper
emple esta posibilidad de

"falibilidad", como caracterstica


esencial que serva para separar las
creencias, explicaciones y teoras en dos
categoras: cientficas y
pseudocientficas. Siguiendo estas ideas
y profundizando en la polmica, el fsico
y filsofo argentino Mario Bunge (n.
1919) extrema la tesis central del
monismo metodolgico en el siguiente
artculo y, en este caso, critica
especialmente al Psicoanlisis al
ponerlo a la altura de la magia o la
supersticin.
A. Lea el artculo de Bunge, a
continuacin:
Una pseudociencia es un montn de
macanas que se vende como ciencia.

Ejemplos: alquimia, astrologa,


caracterologa, comunismo cientfico,
creacionismo cientfico, grafologa,
ovniloga, parapsicologa y
psicoanlisis. Una pseudociencia se
reconoce por poseer al menos un par
de las caractersticas siguientes:
- Invoca entes inmateriales o
sobrenaturales inaccesibles al
examen emprico, tales como fuerza
vital, alma, superego, creacin
divina, destino, memoria colectiva y
necesidad histrica.
- Es crdula: no somete sus
especulaciones a prueba alguna. Por
ejemplo, no hay laboratorios
homeopticos ni psicoanalticos.
Correccin: en la Universidad Duke

existi en un tiempo el laboratorio


parapsicolgico de J. B. Rhine; y en
la de Pars existi el laboratorio
homeoptico del doctor Benveniste.
Pero ambos fueron clausurados
cuando se descubri que haban
cometido fraudes.
- Es dogmtica: no cambia sus
principios cuando fallan ni como
resultado de nuevos hallazgos. No
busca novedades, sino que queda
atada a un cuerpo de creencias.
Cuando cambia lo hace slo en
detalles y como resultado de
disensiones dentro de la grey.
-Rechaza la crtica, matayuyos
normal en la actividad cientfica,
alegando que est motivada por

dogmatismo o por resistencia


psicolgica. Recurre pues al
argumento ad hominem en lugar del
argumento honesto.
- No encuentra ni utiliza leyes
generales. Los cientficos, en cambio,
buscan o usan leyes generales.
- Sus principios son incompatibles
con algunos de los principios ms
seguros de la ciencia. Por ejemplo, la
telequinesis contradice el principio
de conservacin de la energa. Y el
concepto de memoria colectiva
contradice la perogrullada de que
slo un cerebro individual puede
recordar.
- No interacta con ninguna ciencia
propiamente dicha. En particular, ni

psicoanalistas ni parapsiclogos
tienen tratos con la psicologa
experimental o con la neurociencia. A
primera vista, la astrologa es la
excepcin, ya que emplea datos
astronmicos para confeccionar
horscopos. Pero toma sin dar nada a
cambio. Las ciencias propiamente
dichas forman un sistema de
componentes interdependientes.
- Es fcil: no requiere un largo
aprendizaje. El motivo es que no se
funda sobre un cuerpo de
conocimientos autnticos. Por
ejemplo, quien pretenda investigar
los mecanismos neurales del olvido o
del placer tendr que empezar por
estudiar neurobiologa y psicologa,

dedicando varios aos a trabajos de


laboratorio. En cambio, cualquiera
puede recitar el dogma de que el
olvido es efecto de la represin, o de
que la bsqueda del placer obedece
al principio del placer. Buscar
conocimiento nuevo no es lo mismo
que repetir o siquiera inventar
frmulas huecas.
- Slo le interesa lo que pueda tener
uso prctico: no busca la verdad
desinteresada. Ni admite ignorar
algo: tiene explicaciones para todo.
Pero sus procedimientos y recetas son
ineficaces por no fundarse sobre
conocimientos autnticos. Al igual
que la magia, tiene aspiraciones
tcnicas infundadas.

- Se mantiene al margen de la
comunidad cientfica. Es decir, sus
cultores no publican en revistas
cientficas ni participan de
seminarios ni de congresos abiertos a
la comunidad cientfica. Los
cientficos, en cambio, someten sus
ideas a la crtica de sus pares:
someten sus artculos a publicaciones
cientficas y presentan sus resultados
en seminarios, conferencias y
congresos.
Veamos en un ejemplo cmo obran
los cientficos cuando abordan
problemas que tambin interesan a
los pseudocientficos. En 1998 los
psicobilogos J. S. Morris, A. Ohman
y R. J. Dolan publicaron en la clebre

revista Nature un trabajo sobre


aprendizaje emocional consciente e
inconsciente en la amgdala humana.
Ya que este artculo trata de
emociones conscientes e
inconscientes, parecera que debiera
interesar a los psicoanalistas. Pero no
les interesa porque los autores
estudiaron el cerebro, mientras que
los analistas se ocupan del alma: no
sabran qu hacer con cerebros,
ajenos o propios, en un laboratorio de
psicobiologa. Pues bien, la amgdala
cerebral es un rgano diminuto pero
evolutivamente muy antiguo, que
siente emociones bsicas tales como
el miedo y la furia. Dada la
importancia de estas emociones en la

vida social, es fcil imaginar los


trastornos de conducta que sufre una
persona con una amgdala anormal, ya
sea atrofiada o hipertrfica. Si lo
primero, no reconocer signos
peligrosos. Si lo segundo, ser
propensa a la violencia. La actividad
de la amgdala cerebral puede
registrarse mediante un escner PET.
Este aparato permite detectar
objetivamente las emociones de un
sujeto en cada lado de su amgdala.
Sin embargo, tal actividad emocional
puede no aflorar a la conciencia. O
sea, una persona puede estar asustada
o enojada sin advertirlo. Cmo se
sabe? Agregando un test psicolgico
a la observacin neurobiolgica. Por

ejemplo, si a un sujeto normal se le


muestra brevemente una cara enojada
e inmediatamente despus una cara
sin expresin, informar que vio la
segunda pero no la primera.
Represin? Los cientficos citados
no se contentaron con bautizar el
fenmeno. Repitieron el experimento,
pero ahora asociaron la cara enojada
con un estmulo negativo: un intenso y
molesto ruido blanco, es decir, no
significativo. En este caso, la
amgdala fue activada por la imagen
visual, aun cuando el sujeto no
recordara haberla visto. O sea que la
amgdala cerebral sabe algo que
ignora el rgano de la conciencia
(cualquiera que ste sea).

En principio, con el mtodo que


acabo de describir escuetamente se
podra medir la intensidad de una
emocin. Por ejemplo, se podra
medir la intensidad del odio que,
segn Freud, un varn siente por su
padre. Sin embargo, antes de
proceder a tal medicin habra que
establecer la existencia del complejo
de Edipo. Pero ste no existe, como
lo mostraron las extensas
investigaciones de campo del
profesor Arthur P. Wolf condensadas
en su grueso tomo Sexual Attraction
and Childhood Association (Stanford
University Press, 1995).
Las pseudociencias son como las
pesadillas: se desvanecen cuando se

las examina a la luz de la ciencia.


Pero mientras tanto infectan la cultura
y algunas de ellas son de gran
provecho pecuniario para sus
cultores. Por ejemplo, un
psicoanalista latinoamericano puede
ganar en un da lo que su compatriota
cientfico gana en un mes. Lo que
refuta el refrn: "no es oro todo lo
que reluce".
[29]

B. En base a lo estudiado sobre el tema:


a. Determine cules son los objetivos
del Positivismo lgico, y por qu se
denomina de esta manera a esta
corriente de pensamiento.
b. Mencione qu dos tipos de objetos

pueden ser nicamente objeto de


investigacin y de conocimiento
cientfico para el Positivismo lgico, y
seale cules no seran objetos de
conocimiento para ellos.
c. Responda: Cmo es posible
demarcar el campo entre la ciencia y la
pseudociencia segn Popper? Qu
mtodo utiliz l para separarlas?
Cules son las caractersticas que
tendra que cumplir una teora para que
sea cientfica, segn el mismo autor?
d. Explique brevemente cmo se decide
cundo una teora es aceptable
(evitamos el trmino "verdadera"
porque justamente es lo que se discute
en distintas posiciones) para los
positivistas lgicos, para Popper y para

Kuhn.
e. Responda: Qu supone que podra
oponer a las afirmaciones de Bunge un
partidario del pluralismo metodolgico?
f. En los ltimos prrafos del artculo de
Bunge se afirma: "Las pseudociencias
son como las pesadillas: se desvanecen
cuando se las examina a la luz de la
ciencia. Pero mientras tanto infectan la
cultura". Considera pertinente la
extrapolacin de un concepto de la
biologa, "infeccin", al anlisis de las
sociedades? Por qu?
ACTIVIDAD 2. CONCEPCIN
EPISTEMOLGICA DEL
POSITIVISMO LGICO. KUHN,
TRES PENSADORES EN UNO
A. Lea el siguiente artculo de Bunge:

Todos los universitarios han odo


hablar de Thomas S. Kuhn (19221996). Parecera que no se puede
pasar por culto sin citarlo. De hecho,
Kuhn es el ms citado, aunque no
necesariamente el ms ledo, de todos
los autores no literarios. Hasta la
fecha su libro ms conocido ha
vendido ms de un milln de
ejemplares en veinte lenguas. Sin
embargo, pocos saben que Kuhn no
fue uno sino trino, como dira un
telogo cristiano. Y lo peor es que el
ms influyente de los tres no es el que
el propio Thomas hubiera querido
ser, o sea, un historiador de la ciencia
venerado por sus pares como lo fue,

por ejemplo, George Sarton en su


tiempo. En efecto, el Kuhn popular es
el de los paradigmas y
desplazamientos de tales, o
revoluciones cientficas. stas eran
las ideas centrales (aunque oscuras)
de su libro La estructura de las
revoluciones cientficas, que en 1962
le otorg fama de la noche al da. Un
ao despus lo vi ocupar el centro de
la primera reunin de historiadores
de la ciencia, en Filadelfia. En 1965,
en Londres, volvi a atraer la
atencin en el simposio dedicado a
Popper, y ello por dos motivos. Uno
de stos fue que Margaret Masterman,
una filsofa desconocida, expuso una
ponencia clara y combativa en la que

mostraba que Kuhn haba metido por


lo menos dos docenas de conceptos
distintos en la bolsa "paradigma".
Entre ellos figuraban los de
cosmovisin, modelo a imitar y
programa de investigacin. Kuhn
aprendi esta leccin. Unos aos
despus, cuando vino a hablar a mi
universidad sobre los orgenes de la
teora cuntica, un asistente le
pregunt algo sobre los paradigmas, y
l lo par en seco: "Estoy harto de
eso. Ahora estoy en otra cosa".
Controversia con Karl Popper: Al
terminar su conferencia le pregunt
cul sera su prximo proyecto y me
contest que pensaba estudiar la tesis
de Mary Hesse, de que las teoras

cientficas son modelos visualizables,


como el modelo atmico de
Rutherford-Bohr. Tom no tena idea
de que las teoras son sistemas de
hiptesis, ni de que la teora cuntica
moderna no alienta los modelos
visuales, porque se ocupa de cosas
que carecen de forma propia. El otro
motivo por el cual Kuhn descoll en
aquel memorable simposio de 1965
fue la resonante controversia que
sostuvo con Karl Popper. El contraste
entre ambos era fsico, psicolgico y
filosfico. Kuhn era un gigantn,
hablaba fuerte y fumaba un enorme
habano. En cambio, Popper era
menudo, hablaba bajito y odiaba el
tabaco. La incompatibilidad

filosfica entre ambos no era menos


obvia, pese a que Karl intent
minimizarla. Mientras Popper era
racionalista, Kuhn sostena la tesis
irracionalista de que los cambios de
teora son tan irracionales como las
conversiones religiosas. Sin embargo,
paradjicamente, ambos concordaban
en que no hace falta justificar la
adopcin de una teora; en particular,
los datos favorables no seran
importantes. Pero volvamos a mi
tesis. Mi tesis es que hubo tres
Thomas S. Kuhn en una misma
persona: el historiador, el filsofo y
el socilogo de la ciencia. El primero
fue ignorado o fuertemente criticado
por sus colegas y no form escuela.

El segundo logr la popularidad que


sabemos. Y el tercero, aunque
igualmente popular, slo existi en la
imaginacin de ciertos socilogos de
la ciencia: que lo consideran, junto
con su amigo Paul K. Feyerabend,
como el cofundador o al menos
padrino de la nueva escuela en ese
campo. Esta escuela niega la
existencia de verdades objetivas y
afirma que las ideas, e incluso los
hechos, son construcciones o
convenciones de grupos o
comunidades de investigadores. Se
llaman a s mismos constructivistas
(por oposicin a realistas) y
relativistas (por negar la existencia
de verdades universales,

independientes de las circunstancias


sociales). Lo curioso es que, aunque
Kuhn sostuviera que la sociedad
cambia de teoras cientficas como de
modas sartoriales, sus trabajos
histricos son tan internalistas como
los tradicionales. O sea, no practic
como profesional lo que predic en
su libro ms popular. No menos
curioso es que este libro fuera
publicado originariamente como el
ltimo fascculo de la Encyclopedia
of Unified Science, de orientacin
positivista. Esto es curioso porque
Kuhn era netamente antipositivista.
En efecto, no conceda mayor valor a
los datos empricos y crea ms en la
analoga que en la induccin

(generalizacin a partir de datos


empricos). Pero volvamos al
constructivismo-relativismo.
Contradicciones reveladoras: Hace
unos aos, un periodista de Scientific
American entrevist a Kuhn y le
pregunt si crea que, cada vez que
cambia la cosmovisin dominante,
tambin cambia el propio mundo.
"Por supuesto!", contest Tom con su
vozarrn. Segunda pregunta: "Cree
que el mundo que lo rodea existe
independientemente de usted?"
Respuesta: "Por supuesto!". Esta
contradiccin muestra a las claras la
ingenuidad filosfica de Tom. Hacia
el final de su vida, particularmente en
una conferencia que pronunci en

Harvard en 1991, Kuhn se distanci


explcitamente de los constructivistas,
que niegan la existencia autnoma del
mundo. Aunque sigui admitiendo
(como toda persona razonable) que la
poltica desempea un rol en la vida
cientfica, neg que ste fuese el
principal. Desgraciadamente, Tom no
dijo cules son las motivaciones de
los investigadores bsicos. El gran
Robert K. Merton lo dijo y con razn:
son la curiosidad y el deseo de ganar
prestigio. Quienes buscan poder se
dedican a los negocios o a la poltica.
Consejo a los admiradores del triple
Kuhn: Decdanse a cul de ellos
venerar, porque no slo son
diferentes, sino que no armonizan

entre s. A menos, claro est, que


estn dispuestos a reconocer que
tampoco esta trinidad es inteligible.
Mario Bunge (2000, diciembre 4). La
Nacin.
B. Segn lo estudiado sobre el tema:
a. Caracterice esquemticamente el
concepto de "progreso" en la ciencia
segn el Positivismo lgico, Popper,
Kuhn y Lakatos.
b. Indique, de acuerdo con la lectura de
los dos artculos de Bunge, a qu
corriente epistemolgica cree usted que
pertenece l? Justifique.
c. Determine a qu epistemlogo realiza
crticas, y cules son.

d. En relacin con la siguiente expresin


de dicho artculo, responda las
preguntas a continuacin:
[... ] pocos saben que Kuhn no fue uno sino
trino, como dira un telogo cristiano. Y lo peor
es que el ms influyente de los tres no es el que
el propio Thomas hubiera querido ser, o sea, un
historiador de la ciencia venerado por sus pares
[...]. En efecto, el Kuhn popular es el de los
paradigmas y desplazamientos de tales, o
revoluciones cientficas. Estas eran las ideas
centrales (aunque oscuras) de su libro La
estructura de las revoluciones cientficas,
que en 1962 le otorg fama de la noche al da
[...]

Explique por qu afirma Bunge que


Kuhn no fue uno sino trino? d.1. Por
qu usa la frase de un telogo?
d.2. Cul sera la relacin que quiere
mostrar Bunge entre Kuhn y la religin?
Con qu objetivo intenta mostrar esa

relacin? Segn lo estudiado en la


unidad, elabore una crtica breve a lo
planteado por Bunge.
C. La falacia del espantapjaros o del
hombre de paja
Esta falacia est citada en el libro de la
bibliografa, se refiere al recurso que
consiste en "inventar" un oponente ms
fcil de destruir que el oponente real. El
nombre viene del uso de muecos de
paja para entrenarse los soldados antes
de un combate. Es bastante comn
encontrar este tipo de recursos en las
polmicas entre partidarios de
posiciones rivales.
Considera que respecto a Freud, a
Marx a Kuhn, Bunge est construyendo
un espantapjaros? Explique su

posicin.
Actividad 3. Thomas Kuhn y la visin
del ojo del conejo
A. Lea el siguiente prrafo de Thomas
Kuhn en La estructura de las
revoluciones cientficas:[30]

Desde la atalaya de la historiografa


contempornea, el historiador de la ciencia puede
sentirse tentado a proclamar que cuando cambian
los paradigmas, el mundo mismo cambia con ellos.
Guiados por un nuevo paradigma, los cientficos

adoptan nuevos instrumentos y buscan en lugares


nuevos. Lo que es todava ms importante, los
cientficos ven cosas nuevas y diferentes al mirar
con instrumentos conocidos y en lugares en los que
ya haban buscado antes [...] los cambios de
paradigmas hacen que los cientficos vean el mundo
de investigacin que les es propio, de manera
diferente [...] Lo que antes de la revolucin eran
patos en el mundo del cientfico, se convierten en
conejos despus.

En este prrafo Kuhn alude a la figura


pato-conejo para aludir a la visin
paradigmtica. Lo que desde un
paradigma se ve como "pato" en el otro
puede verse como "conejo". Dicho de
otro modo, viendo lo mismo, ven cosas
distintas. Podemos decir, entonces, que
respecto a teoras rivales como pueden
haber sido el geocentrismo (Tolomeo) o
el heliocentrismo (Coprnico), algunos

vieron "patos" (la Tierra est en el


centro del Universo) donde otros vieron
"conejos" despus (el Sol est en el
centro de nuestro sistema planetario).
Hasta el siglo XVII, el paradigma o
marco cientfico era el de Aristteles,
completado por la Astronoma de
Tolomeo, que presentaba el Universo
dividido en dos partes de naturaleza
diferente, con la Tierra en el centro: la
imperfecta, corruptible y cambiante que
llegaba hasta la esfera de la Luna; y la
perfecta, inmutable e incorruptible, sin
movimiento alguno, que se extenda ms
all de la esfera lunar. Pero fueron las
interpretaciones que Galileo hizo,
elaboradas por Kepler despus, y de las
suyas propias empleando el telescopio,

las que destruyeron ese paradigma


imponiendo la nueva visin que se ha
llamado "copernicana". Un caso
parecido de cambio revolucionario es el
introducido por la Teora de la
evolucin de Charles Darwin respecto a
las teoras creacionistas anteriores, en el
campo de la Biologa.
B. En base a lo estudiado sobre la
posicin de Kuhn:
a. Explique cul es el papel que cumple
la historia de la ciencia en la
epistemologa de este pensador, y en qu
se diferencia de las epistemologas
anteriores.
b. Identifique en las dos modalidades de
produccin cientfica de Kuhn (modo
normal y modo extraordinario, y en cada

uno de sus elementos), el cambio de la


concepcin "creacionista" a la
"evolucionista", y/o de la "geocntrica"
a la "heliocntrica".
c. Intente definir mediante sinnimos el
concepto de "paradigma" segn este
autor.
d. Explique brevemente por qu podra
decirse que Kuhn afirma que dos
observadores que contemplan los
mismos hechos desde supuestos
diferentes ven cosas diferentes, ven
"mundos" diferentes.
e. En virtud de la consigna anterior,
explique qu significa que las teoras
son "inconmensurables".
MATERIALES DE LECTURA I

PERSPECTIVA FALSACIONISTA.

LA SUPERVIVENCIA DEL MS
APTO. VIDA Y MUERTE DE UNA
HIPTESIS
Segn el epistemlogo Gregorio
Klimovsky, la concepcin hipottica de
la ciencia supone admitir lisa y
llanamente que los enunciados
cientficos son aceptados a ttulo de
hiptesis, de conjeturas, y no de
enunciados justificados ya que no
ofrecen garantas para su veificacin.
Esta conclusin puede parecer pesimista
pero tambin es posible concebirla
como una honesta y modesta posicin al
admitirse que en las ciencias fcticas
algunas de las teoras ms consagradas
han terminado siendo reemplazadas por
otras consideradas ms eficaces o

abarcativas. Supongamos que frente a un


problema tenemos una hiptesis que
trata de resolverlo pero all no podemos
dar por terminada la investigacin
puesto que el conocimiento cientfico no
concluye en la mera formulacin de
hiptesis. Proponer hiptesis no es
sinnimo de obtener conocimiento.
Cmo sigue entonces la investigacin?
Parece razonable preguntarnos qu se
deduce de esta hiptesis? Esto nos lleva
a la formulacin de otras hiptesis y a
enunciados observacionales que nos
permitan controlar la hiptesis por
medio de la experiencia y puede ocurrir
que la hiptesis salga exitosa de la
contrastacin emprica. Segn Popper, y
es difcil no coincidir con l (afirma

Klimovsky), la operacin de
contrastacin exitosa no aumenta en
ningn sentido (ni absoluto ni
probabilstico) nuestro conocimiento
acerca de la verdad de la hiptesis. Si la
hiptesis ha resistido todos los intentos
de falsacin, se puede admitir que es
"fuerte" en el sentido de que ha
mostrado su capacidad de
supervivencia. Qu pasa cuando hay
otras hiptesis rivales que compiten
como teoras rivales? En general, no hay
ms remedio que investigar cada
hiptesis por separado y confiar en que
alguna de las hiptesis resulte refutada y
otra ha mostrado ser lo suficientemente
"fuerte" para sobrevivir. Esto lleva a
algunos autores a sealar que el proceso

hipottico-deductivo opera
metafricamente de un modo similar a la
supervivencia darwiniana en la que se
admite la supervivencia del ms apto.
Dice Klimovsky: "En esta concepcin
del mtodo, la vida de una hiptesis (o
de una teora) es dura, trgica, y para
cada una de ellas podra escribirse
acerca de su nacimiento, pasin y
muerte. Ante el problema que intenta
resolver, la hiptesis nace, pero luego
empieza el terrible proceso de
contrastacin por medio del cual se la
intenta aniquilar. Ella resiste, pero
constantemente es agraviada por
nuevos episodios de contrastacin
hasta que, finalmente, la refutacin
termina con ella en un dramtico

episodio de muerte epistemolgica."


(Klimovsky, 2005. 140)
MATERIALES DE LECTURA II

PERSPECTIVA FALSACIONISTA
George Soros, millonario
falsacionista
El financiero y billonario George
Soros, hngaro de nacimiento, antiguo
alumno de Popper, se sinti tan
inspirado por su maestro que bautiz
su Fundacin Sociedad Abierta en su
honor. Soros gan millones en la
bolsa invirtiendo segn las lneas
establecidas por Popper. Popper
pensaba que deba considerarse que
las teoras cientficas ms slidas
eran aquellas que estaban dispuestas

a someterse a las pruebas ms


severas, sobreviviendo a ellas. Soros
aplic este principio a Wall Street.
De este modo hizo una fortuna al
invertir en bonos de seguros
hipotecarios de una compaa de
California que haba sido sometida a
severas crticas por una crisis
inmobiliaria; pens que el hecho de
haber sobrevivido era una prueba
decisiva de que la compaa era, en
el fondo, solvente. La Fundacin
Sociedad Abierta era la teora
poltica de Popper puesta en prctica,
poniendo a prueba el poder
transformador de la apertura.
[31]

El xito econmico de George Soros


podra ser visto como un resultado
exitoso de la adopcin de las ideas de
Popper en el campo de los negocios.
Como el mismo Popper admite, la
contrastacin exitosa no agrega
conocimiento acerca de la verdad de la
teora, el xito de Soros en los negocios
podra deberse a otros factores o
variables.
ACTIVIDAD 4. LAS PERSPECTIVAS
FALSACIONISTAS. LAKATOS
Imre Lakatos critica el tipo de
falsacionismo que l denomina
"ingenuo", sin embargo, su propsito no
es atacarlo sino recoger y profundizar la
heurstica positiva de aqul, generando
una nueva versin del falsacionismo que

denomina "sofisticado", hacindose


cargo -en su reconstruccin- de las
"refutaciones historiogrficas" que Kuhn
opuso al programa de investigacin
falsacionista.
En el texto de la bibliografa se presenta
un caso hipottico de investigacin
cientfica donde se veran cuestiones
centrales de un programa de
investigacin como el que propone
Lakatos. En vista a este ejemplo:
a. Determine qu constituye el ncleo
duro de ese programa.
b. Identifique algunas de las hiptesis
auxiliares que constituyen el cinturn
protector.
c. Explique por qu el cientfico
incorpora tales hiptesis a medida que

se presentan los fracasos


experimentales.
ACTIVIDAD 5. ACERCA DE
DISTINTAS POSICIONES
EPISTEMOLGICAS
Determine si los siguientes enunciados,
son verdaderos o falsos. Justifique en
cada caso.
a. "Segn Thomas Kuhn, la verdad
constituye una meta inalcanzable a la
que nos acercamos en un proceso de
creacin de teoras tentativas y
eliminacin de errores."
b. "Imre Lakatos es un epistemlogo
identificado con el Positivismo lgico."
c. "Para Karl Popper, en el desarrollo
de una ciencia hay que distinguir entre
dos perodos: uno de desarrollo normal

y otro de ruptura no acumulativa."


d. "Un rasgo comn entre los
inductivistas y los falsacionistas es el
esfuerzo por encontrar un criterio
universal y ahistrico para juzgar los
mritos entre teoras rivales."
e. "El epistemlogo Rudolf Carnap se
identifica con el falsacionismo
sofisticado."
f. "Para Thomas Kuhn, el cambio de
paradigma implica progreso en el
sentido de mejoramiento de la teora."
g. "Segn Karl Popper, la observacin
precede a la teora y ofrece una base
concluyente para obtener conocimiento
cientfico."
h. "En la epistemologa de Thomas
Kuhn, los cientficos normales se ocupan

de falsar la teora principal o


paradigma."
i. "El epistemlogo Thomas Kuhn fue
miembro del Crculo de Viena".

Actividad de
integracin de las
Unidades 4, 5 y 6
En esta actividad presentamos un caso
muy analizado en el campo de la
Epistemologa y revisaremos contenidos
de las Unidades 4 y 5. Lea el siguiente
artculo y, luego, responda las
preguntas:

Ciencias Fcticas. Invencin y


contrastacin de teoras en la
investigacin cientfica y modelos
de explicacin
Un caso de investigacin y
elaboracin de teoras cientficas
clsico de la espistemologa de las
ciencias mdicas es el de
Semmelweis en relacin con la fiebre
puerperal. Presentemos el caso:
Entre 1844 y 1848 en el Hospital
General de Viena un mdico hngaro,
Ignaz Semmelweis, realiz una serie
de trabajos vinculados a las mujeres
en estado de postparto. En ese
Hospital haba dos Divisiones de

Maternidad. Semmelweis era mdico


de la Primera Divisin de
Maternidad, y en ella un hecho estaba
causando conmocin: una gran
proporcin de mujeres que haban
dado a luz, contraan una seria
enfermedad, la cual con frecuencia
era mortal. Esta enfermedad era
conocida con el nombre de Fiebre
Puerperal o Fiebre Postparto.
En 1844, en la Divisin Primera de
Maternidad del Hospital General de
Viena, sobre un total de 3.157
mujeres en estado de postparto,
murieron de esa enfermedad 260, un
8,2%. En 1845 el ndice de muertes
en esa Divisin y por esa enfermedad
descendi al 6,8%, pero en 1846

ascendi al 11,4%. La preocupacin


mayor estaba dada porque en la
Divisin Segunda de ese Hospital, en
los mismos aos los ndices fueron
del 2,3; 2,0 y 2,7 respectivamente. Es
decir, mientras que en la Primera
Divisin de Maternidad los nmeros
de muertes por esta enfermedad eran
significativamente altos, en la
Divisin Segunda, en cambio, eran
significativamente bajos en
comparacin.
Semmelweis, posteriormente a esta
experiencia, da cuenta en un libro de
las causas de la fiebre puerperal y las
prevenciones que deben tomarse para
evitarla, relatando sus esfuerzos para
resolver este rompecabezas. Hasta

ese momento, cuando se pensaba en


las posibles causas de la fiebre
puerperal, una opinin ampliamente
aceptada sostena que se deba a
"influencias epidmicas" las cuales
se describan como "cambios
atmosfrico-csmico-telricos" que
se expandan por ciudades y pueblos
enteros produciendo la fiebre
puerperal en mujeres en estado de
posparto. Respecto de esta posible
causa, Semmelweis sin embargo se
preguntaba: cmo era posible que
una epidemia semejante pueda haber
infestado durante aos la Divisin
Primera y haber eludido a la Divisin
Segunda? Y cmo era posible que
mientras esta epidemia acosara al

Hospital, casi no se registrasen otros


casos en la ciudad de Viena y sus
alrededores? Porque una epidemia de
verdad, como el clera por ejemplo,
no sera tan selectiva.
Semmelweis en principio indica que
una causa posible podra darse
porque algunas de las mujeres
internadas en la Divisin Primera que
vivan lejos del hospital, sufran
repentinamente de dolores de parto y
daban a luz en la calle. Sin embargo,
el porcentaje por muerte de fiebre
puerperal en "partos callejeros" era
menor que en la Divisin Primera.
Otra opinin atribua la causa de
mortalidad en la Divisin Primera al
hacinamiento; sin embargo,

Semmelweis indicaba que el


hacinamiento era mucho mayor en la
Divisin Segunda. Luego
Semmelweis descart otras
conjeturas similares. En 1846 se
design una comisin especial para
investigar este problema. Esta
comisin atribuy la frecuencia de
esta enfermedad en la Divisin
Primera a las lesiones que se
producan en las pacientes por
revisiones poco cuidadosas
efectuadas por los estudiantes de
medicina, todos los cuales realizaban
sus prcticas de obstetricia en esta
Divisin. Con el objetivo de refutar
esta ltima opinin, Semmelweis
seala que: a) las lesiones que se

producen naturalmente en un parto


son mucho mayores que las que
podra haber producido un examen
poco cuidadoso de la paciente; b) las
parteras que reciban enseanzas en
la Divisin Segunda hacan los
reconocimientos de sus pacientes de
una manera muy similar a los
estudiantes de obstetricia, sin
embargo, no se producan los mismos
efectos; c) cuando, a partir de un
informe elevado por esta comisin, se
redujo a la mitad el nmero de
estudiantes y se limit los
reconocimientos hechos a estas
pacientes por parte de ellos, la
mortalidad, luego de un breve
descenso, lleg a sus ndices ms

altos.
Tambin se acudi a varias
explicaciones psicolgicas. Una de
stas indicaba que la Divisin
Primera estaba organizada de tal
manera que, cuando el sacerdote
deba dar la extremauncin a una
moribunda, antes de llegar a la
enfermera deba pasar por cinco
salas. Se sostena que la presencia
del sacerdote quien pasaba precedido
por un aclito que haca sonar una
campanilla, causaba un terror tal en
las mujeres de las salas por las que
pasaba que las debilitaba y las haca
ms propicias a contraer la
enfermedad. En cambio, en la
Divisin Segunda esto no suceda

porque el sacerdote tena entrada


directa a la enfermera. Semmelweis
decidi probar esta suposicin. Le
solicit al sacerdote que suprimiera
el toque de la campanilla y evitara
pasar por las salas precedentes a la
enfermera para que llegara a la
habitacin de la enferma terminal en
silencio y sin ser visto. A pesar de
todo esto, la mortalidad en la
Divisin Primera no descendi. A
Semmelweis se le ocurri una nueva
idea: en la Divisin Primera, las
mujeres estaban acostadas de
espaldas, mientras que en la Segunda
de costado. Decidi probar si la
diferencia de posicin resultaba
significativa. Sin embargo, la

mortalidad continu.
En 1847 finalmente por casualidad
Semmelweis dio con la clave para la
solucin del problema. Un colega
suyo, Kolletschka, sufri una herida
cortante en el dedo con un escalpelo
que estaba usando un estudiante suyo
en una autopsia. Luego de ese hecho,
Kolletschka muri presentando los
mismos sntomas que las vctimas de
fiebre puerperal. Por ese entonces
an no se haba descubierto el papel
que cumplan los microorganismos en
ese tipo de infecciones. Semmelweis
por su parte comprendi que la
materia cadavrica que el escalpelo
del estudiante haba introducido en la
corriente sangunea de Kolletschka

era la causa de la enfermedad y


muerte de su colega. Las semejanzas
entre las dolencias de Kolletschka y
las mujeres en estado de postparto le
permitieron a Semmelweis llegar a la
conclusin de que sus pacientes
haban muerto por un envenenamiento
de la sangre del mismo tipo. l, sus
colegas y los estudiantes de medicina
haban sido los portadores de la
materia infecciosa, ya que todos ellos
solan llegar a las salas de parto para
realizar el reconocimiento de las
parturientas inmediatamente despus
de realizar disecciones de cadveres
en la sala de autopsias y slo
habindose lavado las manos de un
modo muy superficial, de tal manera

que incluso stas conservaban a


menudo un caracterstico olor a
suciedad.
Nuevamente, Semmelweis puso a
prueba esta posibilidad. Si la
suposicin era correcta, entonces se
podra prevenir la fiebre puerperal
destruyendo con medios qumicos el
material infeccioso que portaban los
mdicos y estudiantes en sus manos
por el contacto con la materia
cadavrica. Para probar esto dict
una orden que exiga a todos los
estudiantes de medicina que se
lavaran las manos con una solucin
de cal clorurada antes de realizar
cualquier reconocimiento a una
enferma. Luego de esto, la mortalidad

por fiebre puerperal comenz a


decrecer, y en el ao 1848 descendi
hasta un 1,27% en la Divisin
Primera frente a un 1,33% en la
Divisin Segunda.
Esta hiptesis es apoyada adems por
el hecho de que en la Divisin
Segunda la mortalidad fuera ms
baja, ya que all las parturientas eran
atendidas por parteras quienes no
tenan entre sus prcticas realizar
disecciones de cadveres. Asimismo
esta hiptesis explicaba tambin el
caso de que en los "partos callejeros"
la mortalidad fuera tambin menor
respecto a la Divisin Primera, ya
que al ingresar en el hospital
habiendo ya dado a luz, casi nunca se

haca un reconocimiento de la
paciente evitando de este modo la
posibilidad de infectarse. Tambin
esta hiptesis daba cuenta del hecho
de que todos los recin nacidos que
contrajeron la fiebre puerperal lo
haban hecho porque sus madres
haban contrado la enfermedad
durante el parto; porque en ese caso
la infeccin se le poda transmitir al
nio antes de su nacimiento a travs
de la corriente sangunea comn de
madre a hijo, mientras que resultaba
imposible esto cuando la madre
estaba sana.
Posteriormente, a partir de una serie
de experiencias realizadas,
Semmelweis pudo incluso ampliar su

hiptesis: en una oportunidad, l y sus


colaboradores, despus de haberse
desinfectado cuidadosamente las
manos, examinaron primero a una
mujer en estado de parto que sufra de
cncer cervical ulcerado y, a
continuacin, y slo habindose
lavado superficialmente las manos,
examinaron a otras doce mujeres de
la misma sala. Once de las doce
pacientes murieron de fiebre
puerperal. De esta manera
Semmelweis lleg a la conclusin de
que la fiebre puerperal poda ser
producida no slo por materia
cadavrica, sino tambin por materia
en estado de putrefaccin proveniente
de organismos vivos.

[32]

a. Cul es el problema?
b. Cules son las seis hiptesis que han
sido descartadas?
c. Reconstruya cules son los
argumentos y contraargumentos de esas
hiptesis.
d. Cules son sus consecuencias
observacionales?
e. Cul es la primera hiptesis
confirmada y qu argumentos la
apoyaron?
f. Es concluyente la verdad de esa
hiptesis? Justifique.
g. Cul es la segunda hiptesis
confirmada que extrae Semmelweis?
Qu argumentos la apoyaron?
h. Utiliz algn mtodo para la

elaboracin/descubrimiento de las
hiptesis?
i. Qu mtodo utiliz para la
justificacin de las hiptesis tanto de las
refutadas como de la aceptada?
j. Cmo se valora el caso confirmatorio
de una hiptesis en una posicin
refutacionista como la de Popper o en
una posicin confirmacionista como la
de Carnap y la de Hempel?
k. Qu es posible explicar mediante la
primera hiptesis confirmada?, y
mediante la segunda hiptesis
confirmada? Construya el esquema de
explicacin explanans/explanandum
determinando si se trata de una
explicacin nomolgico-deductiva o
estadstico-inductiva.

l. Podra predecir otros hechos con esta


teora? Por qu?

Temas de la Unidad
7.1. Poltica cientfica
7.1.1 Caractersticas generales
7.1.2. La planificacin de la poltica
cientfica. Debates contemporneos
7.1.3. La formacin del investigador
7.1.4. Etapas de la poltica cientfica
argentina 7.1.4. Institutos de
investigacin y Universidad
7.2. Ciencia, tecnologa y sociedad
7.2.1. Ciencia, tecnologa y tica: la
responsabilidad social del cientfico y
el tecnlogo
7.2.2. Ciencia y tica: debates en torno a
la neutralidad valorativa de la ciencia

7.2.3. La tica de la investigacin


cientfica
7.2.4. Ciencia bsica, ciencia aplicada y
tecnologa. Cientificismo y
anticientificismo
Bibliografa obligatoria
Unidad 7 en la Gua de Estudio de IPC,
producida por UBA XXI y editada por
Eudeba, a partir de la edicin de 2010.
Contratti, M. B. (2010). Documento de
Ctedra: Poltica cientfica: problemas
y perspectivas.
En esta Gua de Estudio de IPC, Buenos
Aires: Eudeba; y en el Campus virtual
de UBA XXI.
Contratti, M. B. (2010). Documento de
Ctedra: tica y ciencia.
En esta Gua de Estudio de ipc, Buenos

Aires: Eudeba; y en el Campus virtual


de UBA XXI.
Presentacin de la Unidad
En esta Unidad 7 enfocaremos el
problema de la ciencia desde un punto
de vista distinto al adoptado en las
unidades anteriores, en las que se
consideraba a la ciencia exclusivamente
como una forma de conocimiento, la ms
relevante, por cierto, de nuestro mundo
occidental. En esta unidad analizaremos
a la ciencia como un fenmeno social.
Es preciso tener en cuenta que la
consideracin de la ciencia como un
modo de conocimiento y la ciencia como
fenmeno social sealan aspectos
distintos pero interdependientes, de
modo que en el desarrollo de los temas

se mostrarn vnculos entre uno y otro


modo de enfocar el estudio de la
ciencia.
En tanto institucin pblica, la ciencia
guarda varios tipos de relaciones con el
resto de la sociedad: poltica,
econmica, cultural, etc. En esta parte
del programa veremos a la ciencia
desde dos perspectivas: una es la
perspectiva poltica desde la cual el
fenmeno tecno-cientfico se muestra
como un asunto de inters pblico, por
lo tanto, un objeto ms de las polticas
pblicas que un gobierno debe
implementar en todas las reas de su
incumbencia. El segundo enfoque sobre
la ciencia que desarrollaremos hace
hincapi en el aspecto tico que presenta

la ciencia, que es un aspecto ineludible


de todo fenmeno social.
En relacin con el primer punto se
trabaja el texto poltica cientfica:
problemas y perspectivas, Documento
de Ctedra de Mara Beatriz Contratti.
Como se dijo ms arriba, la ciencia y la
tecnologa constituyen un rea de inters
de los Estados, por eso los gobiernos
respectivos deben disear polticas
destinadas a organizar las actividades
que giran alrededor de esos dos
procesos. No pueden llevar a cabo esta
tarea unilateralmente pues su
complejidad e importancia para la
organizacin y crecimiento de un pas
excede en mucho los propsitos de un
grupo poltico. Por lo general, la

planificacin de una poltica cientfica y


tecnolgica, que es el documento ms
general y exhaustivo de los principios,
propsitos y actividades que se llevarn
a cabo en todo lo que concierna al
establecimiento, ejecucin y promocin
de dicha poltica, se realiza en forma
conjunta con la comunidad educativa,
los distintos sectores econmicos y las
restantes fuerzas polticas, aunque sea
de modo indirecto, adems de atender a
las lneas que en la materia se ofrecen
en otros pases con los cuales mantiene
relaciones de cooperacin. Es por esto
que cada una de las cuestiones
involucradas en la planificacin genera
debates tericos y posiciones
encontradas. En el texto mencionado se

intenta destacar este hecho, pues as se


pone de relieve que el diseo y
ejecucin de una poltica cientfica no es
producto de decisiones errticas o
azarosas; por el contrario, requiere un
fundamento terico afianzado y un arduo
trabajo de coordinacin y centralizacin
entre actores y sectores.
De acuerdo con este espritu, las
cuestiones que se trabajarn en los
apartados del Documento son las
siguientes:
- Introduccin y conceptos
fundamentales. Aqu se desarrollan las
nociones bsicas y generales de lo que
es una poltica cientfica. Se plantea el
carcter histrico que sta tiene y la
opinin de Mario Albornoz, profesor de

filosofa, investigador y encargado de la


gestin en organismos pblicos y
privados en el pas y el extranjero,
sobre los diferentes criterios para
establecer el momento en que dicha
poltica surge.
- La planificacin de la poltica
cientfica. En este punto se analizan los
factores que concurren en el
planeamiento de la poltica de la ciencia
y la tecnologa y las discusiones
centrales en torno a la naturaleza,
alcance e importancia de cada uno. La
distincin (o no distincin) de la ciencia
bsica y la tecnologa y sus
implicaciones en la asignacin de
recursos destaca una diferencia entre las
posiciones del fsico y filsofo

argentino Mario Bunge y el investigador


espaol en agroqumica Eduardo Primo
Yfera. Asimismo, se mencionan los
planteos en torno a la polmica cuestin
de la importacin de patentes del doctor
en economa, Daniel Chudnosvsky, y del
economista Andrs Lpez.
- La formacin del investigador. El tema
de este apartado recae sobre la
formacin de los recursos humanos, el
presupuesto que debe asignarse a este
fin, la dotacin de becarios, etc.,
cuestin central en la planificacin de
una poltica cientfica. Se considerarn
las posturas de Hctor Ciapuscio, doctor
en filosofa, y de Carlos R. Abeledo,
doctor en qumica y ex presidente del
CONICET, sobre la evaluacin que

merece el sistema cientfico argentino en


relacin con el punto en anlisis.
- Caractersticas de la poltica cientfica
argentina. Siguiendo el pensamiento de
Albornoz, se mencionan las distintas
etapas que se han seguido en la
Argentina en materia de poltica
cientfica, las caractersticas de cada
una y principales representantes. Se
examina si en nuestro pas se ha
alcanzado un nivel acorde con los
estndares internacionales actuales en
relacin con el papel desempeado por
las nuevas tecnologas en la
implementacin de polticas. Se
considera la informacin aportada al
tema por el ya nombrado Abeledo y del
economista, ex secretario de Ciencia y

Tcnica y actual rector de la


Universidad de Ro Negro, Juan Carlos
Del Bello.
- La investigacin cientfico-tecnolgica
en la Universidad. En este punto, el
anlisis se centra en la cuestin del
papel que debe cumplir la universidad
dentro del sistema de investigacin.
Aunque en las ltimas dcadas se han
formulado proyectos de avanzada en el
campo de la investigacin en los que se
contempla, entre otras cosas, la
articulacin entre investigacin y
empresa, lo cual involucra una
reformulacin completa de bases y
objetivos en las polticas de la ciencia y
la tecnologa; la universidad no ha
acompaado estos cambios en forma

regular y orgnica. Las razones sobre


ste y otros problemas de la universidad
son examinados en este apartado por
Ciapuscio, Abeledo y Del Bello.
- Conclusiones. Se sintetiza la
problemtica que se ha ido
desmenuzando a lo largo del artculo y
se cierra el mismo con una reflexin de
Albornoz sobre la situacin argentina
actual en materia de poltica cientfica y
tecnolgica.
El segundo aspecto que se considerar
sobre la ciencia como fenmeno social
se refiere a la relacin entre ciencia y
tica. El texto que trabaja el tema es
tica y Ciencia. La responsabilidad
social del cientfico. Crtica de la
teora sobre la neutralidad valorativa

de la ciencia y de la actividad
cientfica. Polmica entre cientificismo
y anticientificismo, Documento de
Ctedra de Contratti.
La relacin entre ciencia y tica tiene
una historia y esta historia se ha ido
articulando a travs de los cambios
sociales, las distintas concepciones
filosficas de la ciencia y los proyectos
polticos, entre otras cosas. Es por esto
que se han generado entre los estudiosos
del tema posiciones encontradas sobre
la naturaleza y alcances de la relacin;
incluso dentro de cada una de las
posturas se puede encontrar una gran
diversidad de matices, lo cual hace
dificultosa una presentacin unitaria de
la cuestin. Pero quiz el factor

fundamental en la dispora de
perspectivas sobre la relacin entre
ciencia y tica, sean las situaciones
paradjicas que crea la cultura cientfica
y tecnolgica a la sociedad actual. El
rostro dual de la ciencia y la tecnologa
siempre ha sido reconocido: por un
lado, stas traen progreso y bienestar en
casi todos los rdenes de la vida, pero
su marcha, desde su surgimiento en la
modernidad, es tan arrolladora que
impide reestructurar a tiempo la
integridad tica necesaria para
acompaar esos cambios. Es as que se
ven cuestionadas desde la perspectiva
tica, en general, por los daos que
ocasionan a su paso, o en particular
cuando se produce un descubrimiento o

una innovacin por la inestabilidad y


conflictos que generan en la vida social.
A partir de este planteo general, el
Documento se organiza del siguiente
modo:
- Introduccin. Se anticipan los temas y
el enfoque de los mismos que se van a
desarrollar en el texto.
- Caracteres de la sociedad
contempornea. Se realiza una
semblanza de la sociedad actual cuyos
rasgos ms sobresalientes llevan la
impronta de la ciencia y la tecnologa.
Esta caracterizacin muestra la
conflictividad generada por estos
factores en la vida social que hacen
imperativa la intervencin de la
reflexin tica para en algunos casos

detener, y en otros contener, el poder


adquirido por la ciencia y la tecnologa
sobre el resto de los sectores sociales.
- La tica y sus relaciones con la ciencia
y la tecnologa. Se ofrecen algunas
nociones bsicas sobre las que se basa
la tica y se examina el sentido preciso
del concepto de responsabilidad en el
marco de la relacin entre tica y
ciencia. Tambin se desarrolla la nocin
de tica aplicada y se hace referencia a
sus diversas ramas. Se plantea la
discusin sobre si se puede adjudicar al
conocimiento mismo involucrado en las
teoras cientficas y tecnolgicas la
responsabilidad por los efectos nocivos
ocasionados por la ejecucin de los
proyectos cientficos y tecnolgicos.

Otra cuestin relacionada al enunciado


del apartado es la que se refiere a las
evaluaciones de riesgo; se pone en duda
el valor que tienen las evaluaciones
obtenidas por procedimientos
exclusivamente estadsticos al tiempo
que se proponen otros modos ms
democrticos y, por lo tanto, ms
cercanos a una perspectiva tica.
- La tica de la investigacin cientfica.
Aqu se trabaja la cuestin de la
conducta del cientfico en el proceso de
investigacin. Este proceso se lleva
adelante mediante la aplicacin del
prestigiado mtodo cientfico. El planteo
central aqu es: el escrupuloso
cumplimiento de los pasos del mtodo
basta para que la investigacin resulte

ticamente correcta? Algunos dan una


respuesta afirmativa a esta cuestin,
otros una respuesta negativa. Estas
respuestas estn condicionadas por la
distincin entre valores cognitivos y
valores morales, si slo los primeros
cuentan en el accionar del cientfico o
tambin juegan los segundos en el curso
de su trabajo. La autora del texto toma
partido por la segunda opcin, y con el
fin de especificar los problemas ticos
que pueden surgir en el transcurso de la
investigacin se analiza un documento
de la National Academy of Science de
Estados Unidos: On Being a Scientist:
Responsible Conduct in Research.
- El cientificismo, el anticientificismo y
la tica. Se ha denominado cientificismo

y anticientificismo a dos posturas


opuestas respecto de la naturaleza de la
ciencia, su valor epistmico y su rol
social. Para el cientificista, la ciencia es
la nica empresa intelectual que
proporciona conocimiento universal y
objetivo; al satisfacer los estndares
cognitivos ms altos tiene un rol
preponderante sobre las dems
instituciones sociales. El
anticientificista, por el contrario, piensa
que la ciencia es una empresa humana
entre otras, y por lo tanto, est sujeta a
los mismos vaivenes polticos y sociales
a que estn sometidas el resto de las
instituciones. Mario Bunge es
considerado uno de los ms notables
representantes de la postura cientificista.

Pero el cientificismo estara de algn


modo respaldado si la posicin
epistemolgica a la que se considera
asociado, el Positivismo lgico, est a
su vez totalmente justificada. A travs
del texto de Marta I. Gonzlez Garca,
Jos A. Lpez Cerezo y Lujn Lpez,
Ciencia, tecnologa y sociedad (1996),
se pone en duda esa posibilidad. Estos
autores enumeran una serie de
objeciones epistmicas al Positivismo
que parecen desautorizar su pretendida
preeminencia sobre todo otro
conocimiento.
- Ciencia bsica, ciencia aplicada y
tecnologa. Otro de los pilares sobre los
que se asienta la posicin cientificista es
la consideracin de la ciencia bsica

como un proceso separado y diferente en


sus objetivos y resultados de la
tecnologa: aqulla slo se ocupa de
conocer el mundo, sta de transformarlo.
La distincin lleva a considerar a la
ciencia ticamente neutral y a la
tecnologa como ticamente responsable
por las consecuencias de las
aplicaciones de las teoras cientficas.
Nuevamente, Bunge se presenta como
uno de los defensores de esta idea junto
al filsofo alemn Nicholas Rescher.
Evandro Agazzi (filsofo italiano), por
su parte, ofrece una concepcin ms
matizada del punto en cuestin: ve una
vinculacin tan estrecha entre ciencia y
tecnologa que las considera como un
fenmeno unitario: la tecnociencia, y

esta forma de entender la relacin da


cabida al enfoque tico. Por ltimo, se
expone el pensamiento del doctor en
filosofa e investigador mexicano Len
Oliv, quien mantiene una postura sobre
la responsabilidad tica de la ciencia
que es afn con la sustentada por la
autora del Documento.
- Reflexiones generales sobre el tema.
Aqu se hace hincapi en una
concepcin de la ciencia como una
institucin integrada al cuerpo social y
solidaria con l. Se remarcan los efectos
negativos de la ciencia que generan
incompatibilidad con la posicin
cientificista, al tiempo que se muestran
las dificultades conceptuales que tiene
que enfrentar una tica de la ciencia por

la diversidad de aspectos que el tema


conlleva.

Actividades de
aprendizaje
ACTIVIDAD 1. LA PLANIFICACIN
DE LA POLTICA CIENTFICA
1.1. En el Documento Poltica
cientfica: problemas y perspectivas se
analizan algunas cuestiones que surgen
en el proceso de armado de la
planificacin de la poltica cientfica.
a. Lea nuevamente los puntos del texto
que tratan sobre las siguientes
cuestiones:

- La distribucin del presupuesto en


las reas consideradas prioritarias:
investigacin bsica, investigacin
aplicada y desarrollo tecnolgico.
- La poltica cientfica orientada al
desarrollo, en el que se seleccionan
las actividades que permiten que el
pas alcance niveles de
competitividad.
b. Cul es el problema en discusin
correspondiente a cada uno de los
aspectos sealados en a.?
c. Cules son las posiciones
respectivas? (Si se mencionan autores
que las sostengan, nmbrelos; si no,
exprese slo la idea)
d. Esquematice los argumentos que
respaldan a cada una.

1.2. Qu problemtica trae aparejada la


transferencia de los resultados de la
investigacin a los sectores
productivos?

ACTIVIDAD 2. LA FORMACIN DEL


INVESTIGADOR
a. A qu aspecto de la planificacin de
la I+D alude el ttulo del apartado?
b. Relacione las nociones de
conocimiento e innovacin.
c. Explique en qu consisten el modelo
lineal de innovacin y las crticas que l
mismo ha recibido.
d. Cules son las ventajas del modelo
interactivo?
ACTIVIDAD 3. CARACTERSTICAS
DE LA POLTICA CIENTFICA
ARGENTINA

a. Complete el siguiente cuadro sobre


las etapas cumplidas por la poltica
cientfica argentina de acuerdo con los
conceptos vertidos por Albornoz:
Caracterssticas
Etapas
Problemas
principales

b. Por qu, segn Albornoz, la


Argentina se estanca en la etapa de
institucionalizacin de la ciencia en
lugar de progresar en lnea con los
pases desarrollados?
c. Cul es la situacin actual del
problema mencionado en b.?
ACTIVIDAD 4. INVESTIGACIN E

INNOVACIN EN LA ARGENTINA
Lea el caso sobre la produccin de
alimentos y luego responda las preguntas
que estn a continuacin.
Lunes 3 de febrero de 2003

ALTA TECNOLOGA: "MADE IN


ARGENTINA" RA EL SECADO
DE ALIMENTOS

EN CHUBUT, UNA PLANTA NICA


EN EL MUNDO
Permite tratar frutas y hortalizas, y

conservarlas indefinidamente sin


necesidad de la cadena de fro
- La liofilizacin posibilita aumentar
12 veces el valor de los alimentos
- La tcnica fue inventada por incas y
vikingos
- Invap desarroll una ingeniera
original
En el gris de la estepa patagnica, el
valle inferior del ro Chubut y su
corredor de chacras y cipresales
esconden un as de espadas para el
campo y la industria criollos. La
novedad es una planta de liofilizacin
(secado por sublimacin) para
alimentos. Pertenece a la empresa

Nutripac SA, y su ingeniera, por


ahora nica en el planeta, fue
desarrollada por la firma nuclear y
espacial Invap SE de Ro Negro
gracias a crditos del Fondo
Nacional de Tecnologa (Fontar) y
avales de la provincia de Chubut.
La liofilizacin, poco conocida en la
mesa local, no altera la estructura
fsico-qumica del material original
crudo, pero permite su conservacin
indefinida sin cadena de fro. A
diferencia de lo que pasa en el
secado por calor, con la liofilizacin
el aspecto, la textura, el sabor y el
aroma del alimento crudo no se
pierden. Por el contrario, se
intensifican. El proyecto tuvo

prioridad nacional y provincial


porque supone dos apuestas
simultneas: el pas puede mejorar
mucho su performance de exportador
frutihortcola, pero adems podra
vender fbricas de liofilizados "llave
en mano" aprovechando que tiene la
planta ms avanzada del mundo, una
que vale la mitad, consume la mitad
de energa y prcticamente no se
rompe. Si para un tecnlogo en
alimentos liofilizar es extraer ms del
95 por ciento de agua, para un
transportista significa llevar diez
veces ms mercanca, pero sin unidad
frigorfica a cuestas; para un
supermercadista, stocks de frutas y
hortalizas sin gastos de conservacin,

y para un economista, valor agregado


local para nuestras exportaciones.
Pero mucho valor agregado: el
proceso permite transformar un
kilogramo de frutillas frescas, que
vale 20 centavos de dlar, en 70
gramos. Slo que cuando se junta un
kilogramo de liofilizado (a partir de
13 kilogramos de material crudo), el
valor del producto terminado no es,
como indican las razones y
proporciones, de 2,6 dlares. Es de
30 dlares. Casi 12 veces ms.
Aunque la liofilizacin es compleja,
su fsica es simple: previamente
congelado y trozado, el alimento va a
una cmara hermtica donde soporta
un alto vaco como slo lo conocen

los astronautas: de los 1025 milibares


tpicos a nivel del mar, la presin de
aire cae debajo de 1 milibar, lo que
supone una zambullida igualmente
brutal de la temperatura. El agua est
petrificada como hielo, que en alto
vaco "sublima", es decir, pasa
directamente de slido a gas,
evaporndose despacio de los
tejidos, que siguen intactos. Por eso,
una frutilla liofilizada parece
seriamente una frutilla y no una pasa,
aunque pesa menos que el telgopor.
El gusto, sin embargo, no es de
telgopor. El tecnlogo logra que la
sublimacin no arrastre los aceites
aromticos livianos del alimento. Por
ende, sabor y olor no slo

permanecen intactos, sino que se


concentran en la superficie del
material. El resultado es que una
frutilla liofilizada tiene, aunque
parezca raro, ms gusto a frutilla que
una fresca. Y es crocante. Y lo mismo
vale para la carne, la papa, la pera, la
zanahoria... "Para poner en cifras el
potencial del producto -explica el
doctor Jorge Yanovsky, de Nutripac,
rarsima mezcla de bilogo molecular
y empresario-, digamos que si la
Argentina dedicara no las actuales
800 mil hectreas de tierra a frutas y
verduras, sino dos millones, y adems
exportara todo ese producto
liofilizado, estara generando un
milln de puestos nuevos de trabajo.

Y nuevos ingresos por 30 mil


millones de dlares." La
especulacin de Yanovsky
impresiona cuando se piensa que todo
lo que genera en la Argentina el
poroto de soja anda por los seis mil
millones de dlares por ao. Con
frutas y verduras liofilizadas se
lograra el mismo resultado
econmico... pero en slo 400.000
hectreas y con 400.000 nuevos
puestos de trabajo, mitad en las
chacras y mitad en el segmento
industrial. Son nmeros que
apabullan.
El liofilizado surgi de la necesidad,
inventado por incas y vikingos que
necesitaban comida hipercalrica,

ultraliviana e imputrescible para sus


raids militares. Los incas
aprovechaban el altiplano, con sus
noches glaciales y su insolacin
diurna, para transformar la papa en
chuo y la carne de llama en charqui,
los primeros liofilizados de la
historia. Los vikingos, con montaas
ms bajas y sol ms oblicuo,
liofilizaron el arenque con menos
perfeccin. Pero aunque hoy los
sobres con sopas, guisos y otros
liofilizados son ms cosa de
astronautas, montaistas y comandos
militares que del comensal comn,
hay otro mercado ms interesante: son
las firmas alimentarias que
descubrieron los liofilizados por su

sabor intenso, su consistencia


crocante y su carcter novedoso. Una
constante en la antropologa
alimentaria muestra que lo que
empieza como tecnologa de
conservacin la cultura lo transforma
en capricho del paladar. Nadie piensa
hoy en pimientas, ahumados o
fermentacin como modos de evitar
la prdida de carnes o lcteos, pero
hace milenios su origen fue se. "Ms
que en el terreno de la necesidad, en
el del gusto hay un mercado de
lmites desconocidos, todo a crear y
conquistar", resume Yanovsky.
Invap: del uranio a las frutillas

La tecnologa de liofilizacin deriva


de otra francamente vetusta: la de
refrigeracin. Al hacer vaco en una
masa de gas, una bomba genera fro.
En el caso de una heladera, el fro es
lo importante, y en el caso de un
liofilizador, un subproducto, pero la
idea de base es la misma desde ms o
menos 1870. En Gaiman hay otra idea
mejor: en lugar de bombear
trabajosamente el aire hacia afuera de
la cmara de vaco se lo puede
aspirar con un chorro hipersnico de
vapor. ste pasa rugiendo a cinco
veces la velocidad del sonido por una
tobera anexa conectada con la cmara
mediante un empalme en T. El
teorema de Bernouilli garantiza que

en el vapor hipersnico habr zonas


de presin bajsima que "chuparn" el
aire de la cmara hasta expulsarlo
casi enteramente. Slo que sin
enormes, costosas y frgiles bombas
llenas de piezas mviles y sellos, y
con la mitad del gasto de energa.
Sencillo, cuesta la mitad, a la medida
de un pas rico en gas (sirve para
generar vapor), y funciona.
La diferencia conceptual entre el
liofilizador de Invap y los que hoy
funcionan en otros pases es que el
aparato nacional no usa bombas de
vaco (carsimas y frgiles). Para
crear el vaco y las temperaturas
subcongelantes necesarias, el sistema
de Invap usa chorros de vapor

hipersnicos, una idea que los


ingleses probaron sin mayor xito en
la dcada del 60, considerada desde
entonces una va muerta. Y por qu
funcion aqu y no all? Contesta el
ingeniero Horacio Bccoli, de Invap:
"En Pilcaniyeu aprendimos mucho de
ambientes de alto vaco, sustancias
que subliman sin atravesar una fase
lquida y difusin de gases a travs
de superficies porosas. Y lo que
aprendimos funciona tanto para el
uranio como para las frutillas. La
historia mundial de la tecnologa
abunda en estos cambios de
escenario. Lo bueno es que sucedan
tambin aqu".
Opina al respecto el ingeniero

Horacio Bccoli: "Si el Invap no


hubiera hecho en 1983 la instalacin
de enriquecimiento de uranio de
Pilcaniyeu, en Ro Negro, lo de
Gaiman habra sido difcil. Pero
aunque Nutripac y Pilcaniyeu son
plantas con finalidades muy distintas,
tienen tres asuntos en comn:
ambientes de alto vaco, sustancias
que subliman sin atravesar una fase
lquida y difusin de gases a travs
de superficies porosas. Lo que Invap
aprendi con el uranio tal vez ahora
le permita ganar plata al pas con
tomates, frutillas, manzanas... Las
vueltas de la tecnologa son notables.
Tenemos la mejor herramienta. La
ingeniera de Gaiman resulta tan

barata, econmica y durable,


comparada con todo lo actual, que en
medio ao de funcionamiento esta
planta ya gener ms de 30 pedidos
de informes en tres continentes".
Muchos vienen de gigantes mundiales
de la alimentacin.
Daniel E. Arias (2003, febrero 3).
En Chubut, una planta nica en el
mundo. La Nacin.
Lunes 4 de agosto de 2003
Materia gris de exportacin: se
venden dos plantas de secado de
alimentos

MXICO COMPRA ALTA


TECNOLOGA ARGENTINA
- Se utiliza para extraer hasta el 98%
del agua de productos frescos sin
alterar su estructura.
- La primera de las unidades
vendidas se est levantando a 350
Km de Mxico DF.
- Dentro de poco se exportar otra
ms. Ambas para liofilizar frutillas.
Entre 2000 y 2001, la firma nuclear y
espacial argentina Invap construy en
Gaiman, Chubut, una planta para

secar alimentos por sublimacin, o


liofilizarlos, que gasta la mitad de
energa y es ms robusta que
cualquier otra en el mundo. Ahora,
Mxico acaba de comprar dos de
esas unidades, pero adems aparecen
clientes locales. El pas est por
comenzar dos nuevos negocios que
renen casi emblemticamente sus
capitales de siempre, tierra negra y
materia gris... y por bastantes
millones de dlares. La primera de
las unidades exportadas se est
levantando ya en Quertaro, Mxico,
zona industrial situada en el valle
Central, unos 350 kilmetros al norte
del Distrito Federal. En poco tiempo
ms, el mismo cliente (Exportadora

de Hortalizas SD, gigante mexicano


de la alimentacin) pedir una
segunda planta, y dedicar ambas a
liofilizar frutillas para la empresa
Kellogg's de Estados Unidos. Ambas
plantas son de considerable tamao,
capaces de rendir hasta 80 toneladas
anuales de producto seco. Por otra
parte, la empresa rionegrina est a
punto de firmar la construccin de
una cuarta unidad, esta vez para una
cooperativa frutihortcola neuquina.
Tras dcadas de vender slo
productos frescos, o a lo sumo
secados al calor, ahora algunos
productores del Alto valle se
aprestan a imitar a los mexicanos y
poner un pie en el hemisferio norte

con un producto bastante ms


exclusivo.
Fsica, qumica y matemtica
Ms all de la fsica y la qumica,
liofilizar es multiplicar dlares. Al
eliminar el 98 por ciento o ms del
agua del producto fresco sin alterar
su estructura, sabor o calidad
nutricional, la liofilizacin permite
transformar un kilo de frutillas
frescas (que vale 20 centavos de
dlar) en slo 70 gramos de lo
mismo. Slo que cuando se junta un
kilo de frutillas liofilizadas (a partir
de 13 kilos de material crudo) el
valor del producto terminado no es,

como indican las razones y


proporciones, de 2,6 dlares, sino de
30. Esto explica que, hasta que se
conoci la planta de Gaiman, en
2002, hubiera "frutilleros" tucumanos
dispuestos a liofilizar su producto
en... China. Y para entrar al mercado
ingls!
Semejantes triangulaciones se
originan, ante todo, en que hoy la
demanda mundial de liofilizado
supera largamente la oferta. Mientras
esto siga as, en el rubro frutillas
quien hoy factura un milln de
dlares vendiendo producto fresco
podra facturar 6,5 millones
liofilizando slo la mitad de su

produccin, y 12 millones
procesndola toda. Tanta
multiplicacin de valor surge de las
propiedades del producto terminado:
su sabor natural se potencia, se puede
almacenar indefinidamente sin cadena
de fro, suele pesar entre el 5 y el 10
por ciento de lo que pesa la materia
prima fresca, pasa fcilmente las
barreras sanitarias y ofrece una gama
poco conocida de consistencias y
sabores que est originando muchos
productos emergentes de consumo
masivo, asuntos casi de "cocina
ficcin". Por ejemplo, esa barrita de
carne tipo snack que uno puede
comprar en un quiosco neoyorquino
para comer en la calle y que hace

"crunch" en la boca como un turrn. o


papas crocantes idnticas a las papas
fritas... pero crudas, nutritivamente
superiores y totalmente libres de
algunos inevitables subproductos
cancergenos que originan las frituras.
El dueo actual de la planta de
Gaiman, Jorge Yanovsky, de Nutripac
SA, cuyo prototipo ya ha funcionado
tres aos sin problemas, hace el
siguiente clculo: "Si la Argentina
dedicara, no las actuales 800 mil
hectreas a frutas y verduras, sino dos
millones, y adems exportara todo
ese producto liofilizado, estara
generando un milln de puestos
nuevos de trabajo, mitad en las
chacras y mitad en el segmento

industrial. Y, de yapa, nuevos


ingresos por 30 mil millones de
dlares". Todo lo que genera en la
Argentina el poroto de soja anda por
los 6 mil millones de dlares por
ao. Con frutas y verduras
liofilizadas se lograra el mismo
resultado econmico... pero en
apenas 400.000 hectreas y con
400.000 nuevos puestos de trabajo.
Son nmeros que apabullan: permiten
imaginar un futuro agroindustrial ms
integrador que el actual, basado casi
nicamente en las oleaginosas.
Daniel E. Arias (2003, agosto 4).
Mxico compra alta tecnologa

argentina. La Nacin.
a. Caracterice los conceptos de I+D,
innovacin y competitividad (utilice a
este fin el texto completo de Poltica
cientfica: problemas y perspectivas,
especialmente el apartado: "La
investigacin cientfico-tecnolgica en
la Universidad").
b. Cmo cree que operan los factores
mencionados en 1.1.a. en el caso de la
liofilizacin de los alimentos? (Aclare
si encuentra un nuevo concepto, adems
de los mencionados en el Documento)
c. Por qu lo narrado en el caso de la
liofilizacin de los alimentos difiere de
las conclusiones presentadas en el texto
sobre poltica cientfica?
ACTIVIDAD 5. TICA Y CIENCIA

En el Documento tica y ciencia... de


Contratti, se han considerado
centralmente dos aspectos de la relacin
entre tica y ciencia: uno est referido a
la actuacin del cientfico en tanto
investigador y a ciertas normas de
conducta que el mismo tiene que
satisfacer desde el punto de vista
deontolgico de su profesin. El
segundo aspecto apunta a la
responsabilidad del cientfico como
productor de conocimientos en la
medida en que tales conocimientos son
susceptibles de portar los valores que el
cientfico o la comunidad de cientficos,
en tanto seres humanos que viven en una
sociedad determinada y en una poca
histrica dada, introducen en ellos. La

presente actividad requiere que aplique


los conceptos desarrollados sobre estas
dos cuestiones al relato sobre los
experimentos nucleares franceses en
Argelia.
Lea el artculo periodstico y luego
resuelva los puntos que estn a
continuacin.
infobae.com 16/2/10

FRANCIA US CIENTOS DE
SOLDADOS EN
EXPERIMENTOS NUCLEARES
El ejrcito francs expuso

deliberadamente a sus soldados a


radiaciones durante una serie de
ensayos nucleares atmosfricos
realizados en los aos 60 en Argelia,
segn un informe militar.
Al situar soldados cerca del punto de
explosin, el ejrcito buscaba
"estudiar los efectos psicolgicos
producidos por el arma atmica sobre
el hombre", segn indica el primer
tomo del "Informe sobre los ensayos
nucleares franceses de 1960-1965",
clasificado como secreto.
El Ministro de la Defensa Herv
Morin asegur que se informara "con
total transparencia" sobre el nivel de
exposicin a las radiaciones despus
de cada ensayo.

El informe, escrito por oficiales


franceses seguramente en 1998, es
decir despus de que Francia dejara
de realizar pruebas nucleares (en
1996), hace referencia especialmente
al ltimo ensayo atmosfrico
realizado en el Sahara argelino, que
se llev a cabo el 25 de abril de
1961, cuando Argelia an era una
colonia francesa. Poco despus del
lanzamiento, se organizaron
maniobras para enviar una treintena
de soldados al rea contaminada.
Algunos soldados se protegieron en
trincheras individuales cavadas a
unos 800 metros del lugar de impacto
mientras que otros se acercaban a
dicho punto en camiones 4x4. Estaban

equipados con botas, guantes y


mscaras de combate. El objetivo era
"estudiar los efectos fisiolgicos y
psicolgicos producidos sobre el
hombre por el arma atmica, con el
fin de obtener los elementos
necesarios para la preparacin fsica
y la formacin moral del combatiente
moderno", explica el informe. Las
maniobras deban permitir tambin
"realizar un programa de instruccin
sobre las medidas prcticas que
deben tomar los combatientes para
ponerse en guardia, protegerse y
descontaminarse", aade el texto.
Despus de la explosin, los
resultados constatados por el informe
sealan "la ausencia de quemaduras

visibles, efectos mecnicos


prcticamente nulos, nivel de
radiactividad elevado en maniques".
"Pareca, segn estos resultados, que
a 800 metros del punto cero y afuera
de la zona de cada de polvo
radioactivo, los combatientes estaran
fsicamente aptos para continuar en
combate", aade el documento. El
texto reconoce sin embargo que
"estando a la ofensiva, si la infantera
fuese llamada a combatir en una zona
contaminada (... ) la vestimenta
especial no les proporcionara sino
una proteccin relativa y habra que
reducir el tiempo de permanencia".
Morin afirm que "una sntesis de ese
informe fue elaborada para el

Ministerio de la Defensa en enero de


2007", relatando "las
experimentaciones tcticas durante"
dos pruebas nucleares. El ministro
record adems que una ley de
indemnizacin de las vctimas de los
ensayos fue adoptada en 2009,
indicando que haba exigido "un
estudio complementario sobre cada
tiro y el nivel de exposicin
correspondiente".
"Las conclusiones sern pblicas",
anunci Morin. Francia ha realizado
210 ensayos nucleares desde el
primero en el Sahara en 1960 hasta el
ltimo de 1996 en Polinesia francesa.
Miles de veteranos de los ensayos
nucleares, convencidos de haber sido

contaminados por la radiactividad,


luchan porque se les reconozcan los
perjuicios ocasionados. En otras
publicaciones se dan otras
precisiones sobre el operativo
francs:
"Participaron 300 personas, en su
mayora reclutas que 20 minutos
despus de la explosin salieron de
los refugios y a los 35 minutos
empezaron la aproximacin a pie al
punto cero y llegaron a 700 metros de
distancia. Se hizo llegar una patrulla
de vehculos todoterreno a 275
metros. Los autores del informe de
1998 reconocen que las autoridades
militares de los aos 60 hicieron

manipular sustancias a los miembros


de la tropa, pese a conocer los
peligros que eso poda entraar. As,
por ejemplo autorizaron a los
trabajadores a no llevar mscara, lo
que significaba que podan inhalar en
un da el polvo radiactivo equivalente
al "normalmente autorizado en tres
meses". En otro artculo se informa
sobre la reaccin de los afectados
por la contaminacin radiactiva y la
respuesta de los Estados
responsables. "Existen imgenes del
acontecimiento, aunque el montaje y
el relato propagandstico d una
versin de los hechos completamente
diferente. Pierre Messmer lleg vivo
a los 91 aos, pero el ministro Gastn

Palewski muri en 1984 como


consecuencia de una leucemia,
supuestamente provocada por el
incidente. Se desconoce el nmero
exacto de afectados por este y otros
incidentes, y por los ensayos de
Reggane, entre soldados y habitantes
de la regin, incluyendo tuaregs que
no pasaron por un proceso de
descontaminacin (unas duchas no
muy fiables). Los primeros se
organizaron hace unos aos para
reclamar una investigacin de las
consecuencias de los accidentes, as
como indemnizaciones. Las
asociaciones de los afectados por los
ensayos nucleares, Aven y Moruroa e
Tatou (esta ltima rene a las

vctimas de los 193 ensayos que se


realizaron en Polinesia tras la
finalizacin de los experimentos
argelinos en 1967), llevan tiempo
reclamando, adems de
indemnizaciones, cosas tan
elementales como un censo de
potenciales afectados, la creacin de
un comit de seguimiento de las
consecuencias sanitarias y
medioambientales de los ensayos
nucleares. El pasado mes de julio se
anunci la creacin de un comit
"verdad y justicia" de apoyo a dichas
asociaciones. Pero ni el Estado
francs ni el argelino, cmplice de la
antigua potencia colonial, estn
dispuestos a dar pasos que impliquen

asumir algn tipo de responsabilidad


jurdica".
Francia us cientos de soldados en
experimentos nucleares. (2010,
febrero 16). Disponible en
<http://www.infobae.com/mundo/5009
0-0-francia-uso-cientos-soldadosexperimentos-nucleares>
a. Cules son los objetivos del
experimento descrito en el caso?
b. Mencione tres objeciones al
procedimiento experimental narrado en
el caso, desde el punto de vista del
concepto de responsabilidad analizado
en el apartado "La tica y sus relaciones
con la ciencia y la tecnologa", del
Documento de Ctedra.

c. Qu norma o normas de la
investigacin se ha incumplido
especficamente en este caso?
d. i) Determine cul o cules de los
siguientes agentes deberan haber
evaluado la implementacin de
experimentos como el descrito antes de
su ejecucin:
- El gobierno.
- La comunidad cientfica.
- El o los cientficos que disearon e
intervinieron en el experimento
exclusivamente.
- La sociedad en cuyo pas se realiz la
experiencia
- Todos los nombrados.
ii) Explique la razn de su eleccin.
e. identifique en el caso el aspecto

cientfico y el tecnolgico. Suponiendo


cul de los aspectos mencionados se
encontraran incorporados valores
polticos, econmicos y sociales para el
cientificista y para el anticientificista?
Fundamente su respuesta.
f. Qu opinaran sobre lo ocurrido en
Argelia, en el supuesto de que lo
hicieran, los siguientes autores: Mario
Bunge, Nicols Rescher, Evandro
Agazzi y Len Oliv?
g. Cules son los mecanismos de
reparacin con los soldados "cobayos"
utilizados por el ejrcito y gobierno
francs? Cree usted que son suficientes
y adecuados? Por qu?

Introduccin y conceptos
fundamentales
Se denomina poltica cientfica al
conjunto de disposiciones
gubernamentales destinadas a organizar

el potencial investigador de un pas y


orientarlo en su crecimiento y su
aplicacin en otras reas donde los
resultados de la investigacin son
necesarios para su desarrollo. Mario
Albornoz (investigador argentino)
caracteriza este proceso como "un
mbito de polticas pblicas referido a
la produccin de conocimiento, su
comercializacin, adaptacin e
incorporacin al aparato productivo
[...] configura un vasto espacio en el
que confluyen aspectos de diversas
polticas, como la educativa, la
econmica y la industrial. La poltica
cientfica y tecnolgica est entramada
con todas ellas, lo cual le confiere
centralidad, en el marco de las

polticas pblicas, y refuerza su


naturaleza poltica" (Albornoz, 1990.
171).
A travs de las distintas etapas
histricas la poltica cientfica se fue
complejizando ms y ms. Mientras a
fines del siglo pasado apenas se poda
hablar de administracin de las
actividades cientficas, hoy tenemos un
gran nmero de categoras distintas
interviniendo en la conceptualizacin
del proceso. Por eso haremos continuas
referencias a fenmenos que se han
generado en su carcter peculiar
especialmente a partir de la segunda
mitad del siglo XX, como el "proceso
de innovacin", el "cambio tecnolgico"
y su impacto en la economa y la vida

social, a las actividades de


investigacin y desarrollo
(representadas habitualmente por la
sigla I+D) que son objeto de la poltica
cientfica y a la competitividad, uno de
los fines fundamentales de esta poltica
dada la existencia de un mundo
globalizado.
La vigencia institucional de la poltica
de la ciencia tiene una historia muy
breve; la mayora de los autores que se
ocupan del tema acuerdan en que la
Segunda Guerra Mundial marca la
consolidacin de tal institucin. Adems
de las razones de ndole blica que
impulsaron el avance de la investigacin
en ese momento (entre los productos del
inters militar se encuentra el proyecto

Manhattan de la bomba atmica, el radar


en la defensa area de Inglaterra, los
cohetes balsticos alemanes, etc.),
existieron otras razones procedentes de
las caractersticas adquiridas en su
expansin por la ciencia y la tecnologa:
el papel que juegan en el desarrollo
econmico de un pas, la creciente
tendencia a la aplicacin de las teoras
cientficas, los efectos beneficiosos o
nocivos de la tecnologa que provocaron
el consiguiente inters social por ella y
por la ciencia a la cual est asociada y
la aparicin del cientfico como un
profesional independiente.
Albornoz no acuerda con esta
consideracin histrica, al menos en el
caso latinoamericano, especialmente el

argentino. Segn este autor es "[...]


hasta cierto punto, un lugar comn en
la bibliografa sobre el tema dar por
sentado que la poltica cientfica y
tecnolgica constituye un fenmeno
que emerge despus de la Segunda
Guerra Mundial. Pero esta suposicin
no toma en cuenta importantes
decisiones adoptadas en pocas muy
anteriores, ni la centralidad que lleg
a tener el problema cientfico en el
discurso poltico, en determinados
perodos de la historia argentina [... ]
una perspectiva histrica que no se
restrinja a las dcadas recientes,
permitir ver que la ciencia y la
tecnologa se han acercado al centro
de la escena poltica, o se han alejado

a zonas perifricas, por razones que


han obedecido tanto a los proyectos
polticos hegemnicos como a la
percepcin de los cientficos respecto
de sus propios intereses" (Albornoz,
1990. 172).
La planificacin de la poltica
cientfica
La articulacin de todas las
disposiciones decididas por los
gobiernos en el rea cientfica y
tecnolgica se plasma en planificaciones
de largo alcance. stas deben ser el
resultado del consenso de todas las
fuerzas polticas de un pas, nico modo
de alcanzar la estabilidad necesaria para
el logro de los objetivos propuestos.
En el diseo de los planes de poltica

cientfica se toman en cuenta distintas


cuestiones que van de lo general a lo
especfico. Dada la diferente
generalidad de los aspectos
considerados en los planes, los actores
intervinientes en su confeccin son
tambin distintos, operan en diferentes
niveles. En este trabajo comentaremos
algunas de esas cuestiones, aquellas que
a nuestro juicio muestran en toda su
riqueza el debate terico sobre la
cuestin de la poltica de la ciencia y
que son tambin, frecuentemente, objeto
de debate pblico y de inters inmediato
para el especialista en este tema, y para
el ciudadano.
Algunos de los factores a tomar en
cuenta en los planes de polticas

cientficas son, esquemticamente, las


siguientes: a) el monto total de inversin
en I+D; b) la seleccin y valoracin de
los objetivos mayores: ciencia bsica,
desarrollo industrial, servicios, etc.; c)
la seleccin de reas prioritarias
cientficas y tecnolgicas, donde no slo
se debe tener en cuenta el nivel de
desarrollo a alcanzar, sino tambin la
formulacin de prospectivas, es decir la
previsin de los avances cientficos y
tecnolgicos y los cambios sociales; d)
la organizacin del sistema de
investigacin y la coordinacin
interinstitucional e interdisciplinar; e) el
control y la evaluacin de los planes
implementados en relacin con los
resultados de la produccin cientfica y

tecnolgica; f) la aplicacin de los


resultados de la investigacin, es decir,
la transferencia de informacin hacia los
sectores productivos (innovacin); g) la
promocin y organizacin de las
relaciones cientficas internacionales
(Yfera, 1994).[33]
Como hemos sealado ms arriba, no
vamos a analizar puntualmente todas
estas cuestiones si vamos a considerar
globalmente ciertos problemas que se
presentan al momento de hacer
explcitos los planes de poltica
cientfica. De todos modos, los aspectos
enumerados no slo no funcionan en
forma puramente secuencial, sino que
adems interaccionan creando vas de
ida y vuelta y generando mecanismos de

condicionamiento mutuo.
Veamos la primera cuestin referida a la
inversin en Investigacin y Desarrollo.
Es una tendencia general que todos los
gastos de investigacin estn unificados
en el rubro I+D. El peso otorgado a la
I+D dentro del presupuesto total de un
pas corresponde a una decisin poltica
cuyo criterio de decisin se basa en las
necesidades relativas de todas las reas
que debe cubrir el presupuesto y a la
funcin que cumplen la ciencia y la
tecnologa como variables del
desarrollo econmico-social para un
Estado determinado.
Pero el criterio mencionado no se aplica
de modo uniforme en todos los pases.
En algunos se suele fijar el monto en

I+D adecundolo al 3% del PBI. Este


criterio es menos racional que el
mencionado en el prrafo anterior,
porque al considerar slo la cantidad de
recursos que se van a volcar en
investigacin cientfica y tecnolgica se
corre el riesgo de que dichos recursos
sean malgastados, al no estar guiados
por una planificacin ordenada y
compatible con otros criterios de
crecimiento a nivel nacional.
Los comentarios que haremos a
continuacin estarn, por lo tanto,
orientados por el primero de los
criterios mencionados.
Con el fin de realizar la distribucin
global del monto asignado en ciencia
bsica o aplicada y tecnologa, el

estudio del presupuesto debe


contemplar, en primer lugar, los gastos
de estructura y funcionamiento. Esta
estructura est constituida por los
centros de investigacin y las
universidades. En el presupuesto debe
tenerse en cuenta el mantenimiento de
los institutos de investigacin, el
crecimiento del potencial investigador
en la creacin de nuevas entidades y
aumento del personal, la dotacin de la
investigacin universitaria y la
renovacin del instrumental de alto
costo.
Es criterio corriente que la cantidad
asignada para mantenimiento del
personal no debe superar el 70% del
presupuesto total. En el caso de nuestro

pas, resulta ilustrativo el documento del


mdico argentino Bernardo Houssay de
1921, en el cual este investigador se
quejaba de que aqu se gastaba un 90%
del presupuesto en sueldos y un 10% en
laboratorio, mientras en Berln suceda
casi lo contrario: 61% en los institutos y
2l% en sueldos. Con el correr de los
aos, la situacin no ha variado
sustancialmente.
En segundo lugar, otro aspecto que debe
caer bajo la consideracin
presupuestaria est ligado a los
objetivos considerados prioritarios. Es
decir, las cantidades necesarias para la
investigacin bsica, la investigacin
aplicada y el desarrollo tecnolgico y
los mbitos especficos dentro de cada

una de stas, que se deben calcular en


funcin de la importancia que cada una
de estas reas representa en el
desarrollo del pas.
Consideremos con un poco ms de
detalle los problemas que surgen en este
punto. En la seleccin de prioridades
entran en juego diversas cuestiones.
Segn lo define Eduardo Primo Yfera
(qumico espaol, 1918-2007): "La
poltica de I+D debe decidirse en
funcin de la poltica socioeconmica
general y nunca desconectada de sta,
lo que supone que la mayor parte de la
comunidad cientfica deber adaptar
sus objetivos a los grandes problemas
tecnolgicos, econmicos y sociales de
su pueblo" (Yfera, 1994. 333). Este

autor compara el modo en que se da este


proceso de seleccin en los pases
desarrollados y en los de menor
desarrollo a la luz de lo que se
considera en los pases avanzados el
principio fundamental: el objetivo
prioritario de la poltica cientfica de un
pas debe subordinarse a sus
necesidades sociales y econmicas y no
perseguir el incremento de investigacin
cientfica como un fin en s mismo.
Es posible ver la diferencia de enfoque
entre pases en desarrollo y
desarrollados a travs de la declaracin
de principios de las polticas cientficas
de Francia y Alemania. Las
declaraciones dicen, en el caso alemn,
lo siguiente: "El gobierno federal

alemn ha tomado conciencia, y as lo


expresa pblicamente, de que el
fomento y perfeccionamiento de la
capacidad de rendimiento cientfico no
apunta tan slo a los fines de la
poltica cientfica, sino que es premisa
fundamental para las realizacin de
todos los dems fines nacionales [...]
El gobierno federal actuar de modo
que la poltica de investiga cin no
incida slo sobre el desarrollo de la
ciencia, sino que tendr en cuenta las
necesidades nacionales relacionadas
con ciertas parcelas de la ciencia y la
tecnologa y sus consecuencias en la
sociedad" (Yfera, 1994. 334).
Por su parte, en el llamado plan
Giscard, que preside las investigaciones

de Francia desde los aos setenta, de


acuerdo con Yfera, se afirma lo
siguiente: "Sern prioritarias las
investigaciones de fuerte impacto
colectivo y las que, en el sector
industrial y agronmico, favorecen el
crecimiento de las exportaciones y la
independencia nacional en el
aprovisionamiento de energa y
materias primas" (Yfera, 1994. 335).
En cambio, en los pases sin potencial
econmico y poltico independiente,
segn Yfera, "las prioridades
sectoriales son vagas, difuminadas y
poco fundamentadas en estudios
socioeconmicos serios, y la
trascendencia de la actividad de I+D
no se valora a la hora de

seleccionarlas" (Yfera, 1994. 334). Y


lo que es quiz peor, el resultado de las
investigaciones en ciencia bsica
practicadas en estos lugares se publica
en revistas cientficas internacionales
con la triste consecuencia de que es
aprovechado por otros pases
poseedores de un desarrollo econmico
afianzado.
En realidad, hace ya bastante tiempo se
ha instalado en los medios conectados
con las polticas de la ciencia una
discusin sobre el monto que se debe
destinar a la investigacin bsica en
relacin con otras reas.
Reproducimos la versin de Yfera de
esa polmica segn la cual hay dos
posiciones extremas:

Una de esas posiciones expresa lo


siguiente: "Hay que dar preferencia a
la ciencia bsica libre porque lo dems
viene por su cauce natural; la
investigacin bsica es la causa
primaria de la riqueza".
La otra, en cambio, dice: "La ciencia
bsica necesaria para el desarrollo
tecnolgico est en las revistas a
disposicin de todos; los pases menos
ricos deben aprovecharla para crear
riqueza; los pases ricos son los que
pueden estar en cabeza de la
investigacin bsica" (Yfera, 1994.
339).
Los ejemplos de Estados Unidos y Japn
apoyaran la ltima posicin. El primero
se sirvi de la ciencia bsica europea

para lograr su potencial industrial y ms


tarde logr su liderazgo en materia
cientfica. Japn tom tanto la ciencia
bsica como import la tecnologa que
necesitaba, y ello le sirvi de
plataforma de lanzamiento, en primer
lugar, de su desarrollo tecnolgico y
luego, en los ltimos aos, de su nivel
cientfico.
El fsico y filsofo argentino Mario
Bunge (1997), en cambio, denomina
practicismo o anticientificismo a la
posicin segn la cual los intereses
nacionales, considerados de naturaleza
econmica y social pero no cultural y
poltica, deben orientar la poltica
cientfica de un pas. Rechaza esta
perspectiva porque desde ella se abona

el desconocimiento sobre la funcin


cultural que tiene la ciencia bsica, esto
es la de satisfacer la necesidad de saber
y la de promover la independencia
intelectual de la poblacin,
subordinando la ciencia a los
imperativos del desarrollo econmico y
la produccin.
Adems, con ese desprecio a la ciencia
bsica -afirma el autor- se olvida al
mismo tiempo que la ciencia es
precondicin del desarrollo tcnico, "La
ciencia til favorecida en nuestros
pases en desmedro de la investigacin
bsica, ha resultado intil" (Bunge,
1997. 106). El autor proporciona
numerosos ejemplos que muestran el
valor de la ciencia para el desarrollo

tecnolgico: "A quienes dudan de los


beneficios prcticos a largo alcance de
la investigacin bsica, habra que
recomendarles la lectura de Pasteur, o
de una buena historia de la ciencia o
de la tcnica y recordarles los
siguientes ejemplos. La industria
mecnica no puede prescindir de la
mecnica terica, que a su vez naci de
la astronoma, ciencia pura si la hay,
aunque necesaria para la navegacin
de altura y la confeccin de
calendarios [...]" (Bunge, 1997. 106).
En consecuencia, en los pases
subdesarrollados siempre es ventajoso
practicar la investigacin bsica, aunque
deben darse ciertas condiciones de
orden cultural para que ello sea posible.

Tales condiciones, sin embargo, no


constituyen una barrera insalvable.
Menciona a modo de ilustracin, entre
otros, el caso de Florentino
Ameghino,[34] quien desarroll una rama
de ciencia bsica en un contexto cultural
que no era el ms favorable para ese
tipo de actividades (Bunge, 1997. 114).
No slo se puede hacer buena ciencia
bsica en el subdesarrollo, sino que es
ms fcil hacerla que hacer buena
tcnica. sta exige inversiones en
produccin que un pas pobre no tiene.
Por otro lado, la innovacin tcnica
depende de la demanda, y "A quin se
le puede ocurrir disear un nuevo
proceso metalrgico en un pas sin
industria metalrgica, o una nueva

mquina de escribir en una nacin de


analfabetos?" (Bunge, 1997. 117). En
cambio, lo nico que se necesita para
hacer investigacin terica, del tipo de
la matemtica pura, fsica y qumica
tericas y ciencias sociales tericas, es
"papel, lpiz y bibliografa". Incluso
pueden realizarse estudios
experimentales siempre y cuando no
signifique la utilizacin de equipos
costosos.
Para Bunge entonces, la libertad de
investigacin debe estar necesariamente
contemplada en la planificacin de las
polticas cientficas. Aunque la mayora
de los especialistas estn de acuerdo
con el postulado de la libre
investigacin, el peso que le otorga

Bunge y los argumentos que ofrece sobre


su primaca no son compartidos, en
general, en la actualidad. Yfera
entiende que en el presupuesto de I+D
debe destinarse una parte a la actividad
libre de los cientficos y otra al
desarrollo tecnolgico. Cul sea la
proporcin que le corresponde a la
primera depende de criterios
valorativos y pragmticos: la
creatividad cientfica es intrnsecamente
libre; pero hay tambin involucrada una
cuestin de decisin poltica: dicha
actividad responde a la necesidad de
integrar grupos internacionales de
prestigio en la materia. En tal caso seala este autor- el nivel profesional
alcanzado por los cientficos debe ser el

adecuado al compromiso emprendido,


cuestin que conviene cuidar
especialmente en los pases en
desarrollo donde muchas veces este
requisito no se cumple.
Veamos ahora otro punto de vista
necesario a tener en cuenta en las
planificaciones de I+D. Es el caso de la
poltica cientfica orientada al
desarrollo, en el que se prioriza el
fomento de las actividades que permitan
al pas alcanzar o sostener los niveles
de competitividad necesarios para su
sustentabilidad y tambin, como
condicin necesaria del desarrollo
interno, su participacin en el juego
competitivo internacional. La cuestin
central que aqu se plantea es la relacin

que debe establecerse entre la


investigacin tecnolgica y las lneas de
desarrollo econmico y comercial del
pas.
La atencin en esta etapa se dirige
fundamentalmente al potencial
investigador de las empresas, a quienes
el Estado debe apoyar y complementar,
por ejemplo, facilitndoles estudios
prospectivos cuando sea necesario.
En los pases desarrollados, ms del
60% de la actividad de I+D se realiza
en las empresas. En los pases menos
competitivos, en cambio, el Estado sigue
concentrando las polticas de desarrollo
tecnolgico. En estos lugares, el
problema radica en la forma en que se
obtiene la tecnologa producida en los

centros desarrollados, esto es, por


medio de la compra de patentes o bien
por acuerdos de cooperacin, aunque
estos ltimos en general son de
naturaleza cientfica y no tecnolgica.
La utilidad que reviste la compra de
patentes para lograr un nivel de
desarrollo adecuado es otro asunto
siempre discutido. Como dice Yfera,
en s mismo no es bueno ni malo que
haya un dficit en la balanza de pagos
por la compra de licencias de
tecnologa, pues la calificacin de la
compra depende de si est integrada o
no a planes de I+D.
El problema de la importacin de
tecnologa es analizado por el doctor en
economa Daniel Chudnovsky y el

economista Andrs Lpez (1996) en el


artculo "Poltica tecnolgica en la
Argentina: hay algo ms que laissez
faire?"
La expresin laissez faire hace
referencia en general a la poltica del
"dejar hacer" en un asunto; en el caso
del desarrollo tecnolgico, se refiere a
la falta de regulacin por el Estado de la
adquisicin de patentes y su
consiguiente ausencia de coordinacin
con los planes generales de desarrollo.
Para los autores mencionados, la
presencia del laissez faire en nuestro
pas en materia tecnolgica se ha dado
en casi todas las etapas histricopolticas y est estrechamente vinculado
a la poltica econmica vigente y a la

desatencin crnica de los marcos


regulatorios en muchos asuntos
nacionales.
En la dcada de los 90, no obstante, se
adopt la modalidad del laissez faire de
una manera ms deliberada en relacin
con la poltica de ortodoxia liberal de la
economa que, en la mayor parte de los
casos, considera a la tecnologa como
una variable exgena (es decir, que no
juega en forma directa en el proceso de
equilibrio econmico) y con otros
supuestos vinculados a procesos de
modernizacin de la economa. Desde
esa perspectiva poltica se pensaba que:
La apertura de la economa a las importaciones
estimulara una mejora en la eficiencia del sector
productivo a travs de la mayor competencia en el
mercado local; al mismo tiempo facilitara el acceso

a maquinaria y equipo de ltima generacin. A su


vez, la liberalizacin del rgimen de IED (inversin
extranjera directa) promovera la incorporacin y
difusin de los conocimientos tcnicos y gerenciales
de los inversores extranjeros. As mismo, la
desregulacin de los acuerdos de transferencia de
tecnologa sera otro estmulo al proceso de
modernizacin. (Chudnovsky y Lpez, 1996. 34)

Estos presupuestos por s solos no


garantizan el cumplimiento de las metas
esperadas. Es preciso poner en marcha
polticas de innovacin complejas y
continuadas en respuesta a la demanda
empresarial de modernizacin
tecnolgica. La mayora de estas
condiciones de una poltica de
desarrollo tecnolgico efectivo a partir
de la transferencia de tecnologa no se
han cumplido por cuanto "las iniciativas
implementadas estn desarticuladas,

carecen de rumbo definido y no


generan las sinergias que deberan
constituir el propsito central de la
poltica tecnolgica" (Chudnovsky y
Lpez, 1996. 36). A consecuencia de
ello, a pesar de este intento ms
coherente y fundamentado, el laissez
faire tecnolgico no se diferenci de las
polticas errticas del pasado.
En un artculo posterior de uno de los
autores citados, Daniel Chudnovsky
(1999), se da cuenta de la introduccin
de cambios en esta poltica. Esos
cambios estn contemplados en el Plan
Plurianual de Ciencia y Tecnologa
1998-2000. All ya comienza a
instrumentarse un concepto, el llamado
Sistema Nacional de Innovacin (SNI)

que tendr mucho peso en lo sucesivo en


las polticas cientfico-tecnolgicas de
la Argentina (y en otros pases tambin).
Haremos una referencia a las
caractersticas especficas del SNI ms
adelante. Por ahora, basta decir que es
un concepto que involucra a una gran
cantidad de actores de diversa ndole
(Estado, empresas, mercado, sistema de
investigacin, universidades, mbitos
sociales y otros). El plan mencionado
inspirado en polticas afines con el SNI
deja atrs el fracasado laissez faire. Lo
importante a rescatar en la presentacin
de este plan es que, a pesar de que se
trata slo de la formulacin del plan y
considera las condiciones concretas de
la ejecucin del mismo, pone en

evidencia la necesidad de desarrollar


internamente la investigacin cientficotecnolgica (no depender solamente de
la inversin extranjera) y su vinculacin
con el sector productivo, para lo cual la
articulacin por parte del Estado de los
diferentes sectores con polticas
adecuadas es un factor imprescindible.
Esto, por s mismo, represent en su
momento un avance importante.
Otro objetivo que debe ser considerado
en una planificacin de I+D es la
transferencia de los resultados de estos
planes a los sectores productivos. Dado
que el nivel econmico de las naciones
est basado, de una forma cada vez ms
decisiva, en el desarrollo de las
explotaciones agrcolas y la industria, es

obligacin del Estado aumentar el nivel


tecnolgico de dichas reas.
Esta cuestin se articula, no slo en
torno a la transferencias de
conocimientos del Estado al sector
productivo privado, sino tambin a la
distribucin del financiamiento total de
I+D, o sea a la parte que le corresponde
realizar al Estado y a la parte que le
corresponde a la empresa privada. En
general, en los pases desarrollados el
Estado participa en la financiacin de la
I+D realizado en las industrias. Algunos
datos ilustran bien la situacin: en
EE.UU. el 70% de la investigacin se
realiza en las empresas, pero del gasto
global de I+D, el Estado aporta el 50%;
en Japn el 75-80% de los gastos totales

de investigacin lo invierte el sector


privado y en Suiza casi toda la
investigacin la hace la empresa.
En los pases de menor desarrollo
corresponde al Estado liderar los
emprendimientos de I+D. Puede
formalizar programas conjuntos entre
empresas y universidades o centros de
investigacin, o proporcionar incentivos
fiscales a la investigacin, subsidios,
etc. Un problema aqu es que las
empresas transnacionales no realizan
labor de I+D en los pases poco
avanzados. Por consiguiente, el pas
debe importar todas las tecnologas
necesarias cuyas consecuencias son,
entre otras, el atraso en materia de
innovacin y el desequilibrio de la

balanza de pagos.
El ltimo aspecto del proceso de
planificacin de I+D, estrechamente
ligado al anterior, que vamos a
considerar aqu, es la produccin
comercial de los resultados de la
investigacin, es decir la transferencia
de la labor de los institutos de
investigacin aplicada a la industria. En
este nivel se consolida el punto final del
avance tecnolgico: la "innovacin"
tecnolgica. Aqu se pone en juego otro
aspecto de la financiacin que
representa grandes inversiones de
capital sin garanta de rentabilidad, pues
la innovacin producida est sujeta a
muchos factores de incertidumbre, como
cambios de ciclos econmicos,

desequilibrios financieros, variaciones


de la demanda del mercado
internacional, etc. El "capital de riesgo"
que se requiere, en general, procede de
inversores especficos, es decir, de
entidades financieras ad hoc. Un caso es
la Agence National de Valorisation de
la Recherche de Francia, creada para
promover el aprovechamiento de la
tecnologa producida en los organismos
pblicos de investigacin.
La formacin del investigador
Un aspecto crtico del proceso de I+D
es la cantidad y la formacin de
cientficos que lo lleva adelante. Lo
ideal es que el nmero de investigadores
no implique un gasto mayor del 65% del
gasto global, porque de lo contrario no

queda nada para invertir en


mantenimiento y materiales. Algunas
cifras, nuevamente, ilustran la diferencia
entre los distintos pases en esta
cuestin: en EE.UU. la cantidad de
personal entre cientficos e ingenieros
asciende a 750.000; en Japn, 370.000;
en Francia, 135.000 y en Gran Bretaa,
92.000. Con referencia a la cantidad de
habitantes: en Espaa hay 2,21
investigadores por cada 1.000 habitantes
activos, en Francia, 5/1.000 y en
Alemania, 5,9/1.000.
Hay que tener en cuenta, como observa
Yfera y es importante para interpretar
las cifras, que la escasez de
investigadores es a la vez causa y efecto
del bajo desarrollo. Al mismo tiempo, el

nmero del personal condiciona la


poltica cientfica y sta a su vez
determina la cantidad de cientficos que
se requieren de acuerdo con los planes
de I+D, condicionados, por su parte, a la
situacin econmica general. Como
vemos, se producen espirales de las
cuales slo es posible salir mediante
decisiones de alta poltica cuyos
principios y planeamientos trascienden
los intereses sectoriales hacia el
crecimiento de la nacin.
Una perspectiva sobre el modo en que
es posible encontrar un punto de partida
para tomar decisiones es la formulada
por Hctor Ciapuscio. Este autor afirma:
"Dado que la revolucin cientficotecnolgica actual tiene al

'conocimiento' como su factor


principal, la formacin intensiva de los
recursos humanos es una estrategia
hbil para mejorar la situacin
relativa de los pases menos
desarrollados" (Ciapuscio, 1990. 257).
Ciapuscio apoya su tesis en
afirmaciones como la siguiente: "La
materia prima estratgica de la
industria no es ms el carbn, el
petrleo o algn metal raro, es la
materia gris. sta, si se la sabe
movilizar, transforma todo; no hay ms
sector de industria o de punta del
pasado, sino empresas que giran sobre
la inteligencia y otras que giran sobre
los resortes menos poderosos,
financieros y materiales" (Ciapuscio,

1994. 257).
Una consecuencia de la adopcin de
polticas basadas en la premisa de que
el saber es el factor fundamental del
desarrollo de un pas es la necesidad de
estudiar el problema de los recursos
humanos (formacin de profesionales,
investigadores y becarios) relacionados
con las polticas de la ciencia y la
tecnologa y los mecanismos que
permitan implementar eficazmente los
resultados de dichos estudios. En este
aspecto, es importante la fecha en que
Ciapuscio escribe, pues los datos que
ofrece al respecto hoy han perdido
vigencia, aunque no podamos decir
todava que se hayan revertido
totalmente las condiciones de atraso.

Por ejemplo, el autor se refiere al


sistema de becarios, y pone a modo de
ilustracin del rezago argentino en este
campo: en 1989 Brasil, de acuerdo con
su Programa de becarios, esperaba
contar con 10.000 becarios de este tipo;
en tanto la Argentina tena, en ese mismo
ao, nada ms que 100 becarios
externos. sta y otras cuestiones que
hacen al tema de los recursos humanos
han cambiado en la actualidad. Existen
ejecucin de polticas y proyectos
pergeados durante las casi dos dcadas
posteriores al texto de Ciapuscio que
permiten visualizar mejoras importantes:
en 2006 haba ya 3700 becarios slo del
CONICET (Abeledo, 2006b).
Ciapuscio analiza la opinin segn la

cual la escasez de investigadores en


formacin se justifica por nuestra
deficiente situacin econmica y la
estrecha estructura del sector productivo
y de servicios. Bajo estas condiciones,
mayor cantidad de becarios significara
un gasto intil y gente muy preparada sin
lugares de insercin para su realizacin
profesional.
El autor califica esta perspectiva de
"malthusiana"[35], Venezuela, que
contaba en 1974 con poderosos recursos
derivados del petrleo, implement un
ambicioso programa de becas en el
exterior. Cuando los beneficios del
petrleo se acabaron, el pas qued con
una buena infraestructura industrial,
tecnolgica y universitaria.

Nosotros, en cambio, afirma, quedamos


en total condicin de inferioridad frente
a un vecino como Brasil, por ejemplo,
que cuenta con una gran poblacin
capacitada cientfica y tcnicamente. El
problema es que, pese a los datos en
contrario, "en nuestro pas se tiene la
arraigada ilusin de que disponemos
de recursos humanos de alta calidad y
relativamente abundantes" (Ciapuscio,
1990. 258). Esto se debe a "la
supervivencia del mito de la riqueza
nacional y la autocomplacencia de
algunos de nuestros dirigentes cuando
se refieren al hombre argentino." En
realidad, segn el autor, esta idea del
hombre argentino proviene de ciertas
condiciones del pasado (segunda mitad

del siglo XIX y principios del XX) en


las que se destacan la conducta de
ciertos sectores sociales que
contribuyeron en su momento al
crecimiento general en materia poltica,
social y econmica del pas, a saber: "el
legado de la inmigracin, la vieja
escuela pblica y el perodo
'cientificista' de la Universidad"
(Ciapuscio, 1990. 259).
El autor sostiene tambin que,
comparados con otros pases donde se
ha producido una verdadera revolucin
acadmica, consistente en incluir la
investigacin cientfica en el nivel de
grado universitario, nosotros estamos en
el nivel de pregrado, ciclo de enseanza
caracterizado por la enseanza

repetitiva, el libro de texto y la clase


"magistral". El nuevo modelo, en
cambio, pone el acento en la actitud
crtica e independiente del aprendiente,
favoreciendo su capacidad de
creatividad y cuestionamiento.
Especificando y dando forma concreta a
la nocin de conocimiento, mencionada
por Ciapuscio, como motor de las
decisiones de poltica tecnolgica,
Carlos Abeledo (doctor en qumica)
parte de la nocin de un sistema
nacional de innovacin, surgida de los
encargados de formular polticas
cientficas en los pases
industrializados. Dicha nocin, afirma,
"proporciona un marco til para la
formulacin de polticas

tecnolgicas[36] puesto que hace


explcitas las muchas y variadas
formas de aportes necesarios para
producir una economa innovadora y
por lo tanto competitiva en los
mercados actuales, cada vez ms
globales" (Abeledo, 2006b).
Antes de continuar ahondando en las
caractersticas de este sistema nacional
de innovacin, es preciso hacer algunas
aclaraciones respecto del trmino
"innovacin". En otro texto del autor que
estamos viendo (Abeledo, 2004a), ste
dice que el trmino tiene dos
significados: uno responde al modelo
lineal, segn el cual la relacin entre
investigacin y desarrollo tecnolgico
sigue la secuencia:

Investigacin bsica investigacin


aplicada desarrollo tecnolgico
introduccin de nuevos productos al
mercado (Abeledo, 2004a)
El autor sostiene que este modelo lineal
ya no responde a la realidad.
Originalmente surgi a partir de
innovaciones excepcionales que se
gestaron a raz de descubrimientos
cientficos de gran importancia y
notoriedad, como por ejemplo los casos
de la energa nuclear, el caucho y los
textiles sintticos, el radar, la
computacin digital, los transistores y
circuitos integrados, el lser, los
antibiticos y la ingeniera gentica.
Pero la mayora de las innovaciones no
obedece a la secuencia establecida por

el modelo. La inadecuacin del mismo


se acenta cuando se intenta aplicar a
los procesos de investigacin,
innovacin y desarrollo tecnolgico al
mbito latinoamericano. Como explica
Abeledo: "La mayora de las
innovaciones que impulsan el
desarrollo tecnolgico no son
necesariamente invenciones de gran
alcance. Algunas innovaciones son
'radicales' y conducen a la creacin de
productos o procesos totalmente
nuevos, en tanto que la mayora son
'incrementales' y representan una serie
de pequeas mejoras en la forma de
hacer las cosas. Las empresas
introducen constantemente
innovaciones incrementales en sus

productos o procesos para poder


mantener su competitividad" (Abeledo,
2004a).
A raz de estas comprobaciones, y dada
la diversificacin de las economas
actuales, se ha formulado otro concepto
de innovacin denominado modelo
interactivo, no causal, que responde a
las variadas relaciones que se
establecen entre los principales actores
que intervienen en el proceso de
innovacin. En este modelo se muestran
las interacciones entre los mbitos de
innovacin de las empresas y a su vez
stas con las fuentes del conocimiento,
es decir, las universidades y los
organismos de investigacin, que en el
caso de nuestro pas, son: CONICET

(Consejo Nacional de Investigaciones


Cientficas y Tcnicas), INTA (Instituto
Nacional de Tecnologa Agropecuaria),
INTI (Instituto Nacional de Tecnologa
Industrial), CNEA (Comisin Nacional
de Energa Atmica), etc. Siendo un
modelo no causal las interacciones
pueden darse en paralelo, no
linealmente. Es preciso notar tambin
que el modelo descrito pone de relieve
el papel de las empresas y las demandas
del mercado en el proceso de
innovacin.
En el concepto de "sistema nacional de
innovacin" se piensa la innovacin en
el segundo de los sentidos, tal como lo
muestra la definicin que da Abeledo:
"un sistema de interaccin de empresas

(pequeas o grandes) del sector


pblico y privado, de universidades y
organismos estatales orientado a la
produccin de ciencia y tecnologa
dentro de fronteras nacionales. La
interaccin entre estas unidades puede
ser tcnica, comercial, jurdica, social
o financiera siempre y cuando el
objetivo de la interaccin sea el
desarrollo, la proteccin, el
financiamiento o la reglamentacin de
ciencia y tecnologa nuevas" (Abeledo,
2006b).
Este ambicioso programa de innovacin,
plasmado en proyectos y polticas de las
instituciones correspondientes, exigi
prestar mucha atencin al tema de los
becarios y personal de investigacin.

Desde 1990, fecha del artculo de


Ciapuscio, hasta la actualidad, en todos
los mbitos conectados con la
implementacin de las polticas de I+D
e innovacin se ha tomado conciencia de
los cambios que era necesario
introducir, tanto en la formulacin de
polticas como en el nivel operativo,
para superar el atraso en materia de
recursos humanos denunciado por
Ciapuscio. Como vimos, el dato del
nmero de becarios proporcionado por
Abeledo en 2006, contrasta fuertemente
con el ofrecido por Ciapuscio. Veremos
ms adelante si la enseanza
universitaria tambin se ha puesto a tono
frente a los desafos planteados a la
educacin por la ciencia y la tecnologa

mundiales.
Caractersticas de la poltica
cientfica argentina
Albornoz sostiene respecto de las
caractersticas de la poltica cientfica
en la Argentina "la hiptesis de que, a
lo largo del proceso de organizacin
nacional y durante el presente siglo,
hasta nuestros das, la poltica
cientfica se fue desplazando desde el
centro hacia la periferia de la escena
poltica pero, inversamente, se
especific 'hacia adentro' del sistema,
lo cual alent tendencias
autonomizantes y corporativas."
(Albornoz, 1990a. 173). Entonces, segn
esta afirmacin, la poltica cientfica fue
perdiendo esa centralidad no por

integrarse a la accin de otros factores


importantes para el desarrollo nacional,
sino ms bien por encerrarse en s
misma y separarse de esos otros mbitos
del progreso econmico y social. No
obstante, desde su ciuda-dela continu
ejerciendo su papel rector en los
procesos polticos de la ciencia y la
tecnologa. La distincin de las
diferentes etapas de la historia de la
poltica cientfica argentina que realiza
Albornoz explica claramente la
afirmacin antedicha, la ciencia tuvo un
lugar central al comienzo del proceso de
institucionalizacin de la poltica
cientfica.
En la segunda mitad del siglo XIX, en el
perodo de organizacin nacional, se

despleg un gran inters por la ciencia


expresado en el marco de un proyecto
poltico articulador del sistema
econmico, industrial, educativo y de la
consolidacin de la institucin cientfica
y su insercin en la escena internacional.
La figura que se destaca en este
emprendimiento es la de Domingo
Faustino Sarmiento, entusiasta
admirador del modelo tecnolgico e
industrialista norteamericano en materia
de educacin e investigacin.
A pesar de este entusiasmo e inters
poltico por la ciencia en este perodo,
no se logr alcanzar un contenido
cientfico alto. Es decir, hubo grandes
dificultades para concretar las
aspiraciones enunciadas en discursos y

proyectos. Inversamente, en el perodo


siguiente, o sea en el siglo XX, en la
etapa que Albornoz denomina de
institucionalizacin de la ciencia, "La
actividad cientfica se intensific,
mejor su calidad y se expandi a un
conjunto ms amplio de actores. Las
polticas para la ciencia se hicieron
ms especficas y apropiadas. De este
modo el proceso adquiri un fuerte
tono en lo cientfico pero perdi
paulatinamente su centralidad poltica.
La poltica cientfica se fue
convirtiendo en un asunto de
cientficos" (Albornoz, 1990a. 185).
En esta etapa la figura sobresaliente es
la de Bernardo Houssay, un cientfico
con inquietudes por los temas de

poltica cientfica y esa preocupacin se


reflejaba en todos sus escritos. Conoca
bien todas las variables y problemas del
proceso de institucionalizacin de la
ciencia, salarios, institutos, becas, etc.,
por eso pudo desarrollar normativas
respecto de esas cuestiones: los
requisitos que deben cumplir los
institutos para funcionar adecuadamente,
cantidad y carcter de las becas, etc.
Albornoz considera a Houssay uno de
los representantes ms notorios del
positivismo argentino. De todas las citas
que el primero toma del segundo, la que
mejor refleja esa tendencia del ilustre
bilogo, a nuestro entender, es la
siguiente: "[...] Por lo tanto la ciencia
es bienhechora. Acorta o suprime las

distancias, permite distribuir la


riqueza, disminuye el esfuerzo
material, facilita la supresin de todas
las esclavitudes, asegura una vida sana
y ms larga, con ms confort; da las
bases para mejorar el bienestar
general, distribuir las ventajas y
disminuir las desigualdades excesivas"
(Albornoz, 1990a. 187). Todo este
optimismo respecto del poder de la
ciencia es trasladado por Houssay a sus
planteos de poltica cientfica.
En la segunda posguerra se consolida
definitivamente la institucionalizacin
de la ciencia, ya considerada en
concomitancia con la tecnologa y el
desarrollo econmico. Marc el
comienzo de un segundo ciclo dentro de

este perodo de institucionalizacin


caracterizado por la preponderancia del
paradigma cientfico, de la "gran
ciencia", especialmente de la ciencia
fsica. En el plano tecnolgico, en los
aos 60 aproximadamente, en el resto de
Amrica Latina tambin comienzan a
tomar fuerza las teoras del desarrollo
centradas en la investigacin aplicada y
en la problemtica de la transferencia de
tecnologa.
Estas cuestiones vistas
preponderantemente desde el lado
poltico, se analizaron bajo la oposicin
"centro-periferia" en la que se puso el
acento en las relaciones polticas de
dominacin ejercidas por los pases
centrales en detrimento de los

perifricos en virtud de su poder


econmico, poltico y militar
posibilitado por el desarrollo
tecnolgico.
Un rasgo propio de este perodo es la
falta de claridad respecto de los
vnculos entre ciencia y cambio
tecnolgico, explicable en razn de la
estrecha relacin que el desarrollo
tecnolgico tiene con la vida econmica.
Este problema no resuelto se present
tanto en el plano conceptual como en el
organizativo.
El modelo descrito comienza a mostrar
su agotamiento en la dcada que va de
los 70 a los 80. El centro de este cambio
est constituido por las llamadas
"nuevas tecnologas", es decir, la

tecnologa de las comunicaciones, de la


informacin y la ingeniera gentica,
cuya caracterstica principal es el
acercamiento que establecen entre la
investigacin bsica, el desarrollo
tecnolgico y el aparato productivo. En
nuestro pas el reacomodamiento que se
debe operar en el plano de las polticas
a raz de las nuevas formas de
conocimiento y difusin de estas
tecnologas sofisticadas, esto es, la
revisin de la racionalidad de las
prioridades, la distribucin de recursos,
el tipo de institucin requerida y el
perfil adecuado del profesional, tard
mucho en producirse. Recin avanzada
la dcada del 90 y estos primeros aos
del siglo XXI, se vieron los frutos del

trabajo de polticos, tcnicos e


investigadores plasmados en proyectos y
prospectivas que buscan armonizar el
sistema argentino de I+D al concierto
internacional y propiciar de ese modo el
tan ansiado despegue poltico, social y
econmico del pas que le otorgue al
mismo tiempo un lugar destacado en el
progreso de las naciones. El ejemplo
ms destacado en este sentido es el Plan
Estratgico Nacional de Ciencia,
Tecnologa e Innovacin "Bicentenario" 2006/2010. La lectura de sus
metas y objetivos muestra a las claras
que en el pas se ha asumido la
necesidad de actualizar los planes de
acuerdo con los conceptos vertidos en
este Documento:

(i) alcanzar al final del perodo una inversin


consolidada en CTI del 1% del PIB, con una
participacin paritaria estatal y privada, y un total
de investigadores equivalentes a tiempo completo
de 2,3%o de la PEA; (ii) desarrollar capacidades,
recursos humanos y acciones de investigacin e
innovacin en nueve "reas-problema- oportunidad"
y en diecinueve "reas temticas prioritarias"
(disciplinarias y tecnolgicas), en ambos casos en
coordinacin con los ministerios sectoriales; (iii)
desarrollar el Sistema de Informacin de Ciencia y
Tecnologa Argentino (SICyTAR); (iv) evaluar
externamente las instituciones nacionales del
sistema nacional de CTI e impulsar programas
estratgicos de fortalecimiento institucional y
cambio estructural; (v) desarrollar y consolidar la
federalizacin de la CTI a nivel de provincias y
regiones; (vi) promover actividades de divulgacin
y alfabetizacin en ciencia y tecnologa y, (vii)
fortalecer los mecanismos de coordinacin del
sistema y de programacin a corto, mediano y largo
plazo. (Del Bello y Abeledo, 2007)

La investigacin cientfico-tecnolgica

en la Universidad
En relacin con las exigencias que
plantea la nueva orientacin en materia
de investigacin, Ciapuscio, en su
trabajo comentado ms arriba, seala
las deficiencias que padece el sistema
de investigacin en nuestro pas, y
recomienda una rpida reconversin del
mismo si queremos evitar una situacin
de atraso irrecuperable frente a la
velocidad del cambio tecnolgico. Entre
muchas carencias, seala que "ramas
enteras de las nuevas ingenieras y de
las tecnologas estn desiertas"
(Ciapuscio, 1990. 259).
Podra esperarse que hoy las cosas
hayan mejorado notablemente si se
llevaran a la prctica los proyectos

establecidos, los cuales, como el


denominado "Bicentenario", estn
orientados a promover la innovacin.
Para que esto ocurra, las instituciones de
investigacin tendran que presentar un
nmero alto de investigaciones
dedicadas a solucionar los problemas
cientfico-tcnicos ms urgentes del
pas, es decir, las que ponen en marcha a
las nuevas tecnologas. Pero las
conclusiones de los analistas de
polticas pblicas en materia de I+D e
innovacin no son positivas: el
CONICET ha alterado sus objetivos
iniciales que apuntaban a apoyar las
investigaciones de los docentes que
trabajaban en las universidades y otras
instituciones de investigacin cientfica

y tcnica. En la dcada del 70 se


modific el estatuto de la carrera de
investigador y los investigadores
pasaron a tener una relacin directa con
el CONICET, de modo que el organismo
dedic ms a atender la carrera de
investigador de su propio personal
abandonando el objetivo de formacin
de investigadores en relacin con las
necesidades del pas. Abeledo afirma
que hay una diferencia sustancial con lo
que sucede en los pases
industrializados:
La comparacin de los datos estadsticos con los de
pases de mayor desarrollo muestra, adems de las
diferencias cuantitativas de la cantidad de
investigadores en relacin con la PEA (poblacin
econmicamente activa), una distribucin con una
muy baja proporcin de investigadores en
empresas. En los sistemas de innovacin de mejor

desempeo, la proporcin de investigadores en


empresas es comparable a la que se desempea en
universidades y en institutos pblicos. Un aumento
en las tasas de formacin de nuevos investigadores
debera estar acompaado por un aumento en la
apertura de nuevas plazas, especialmente en las
empresas privadas. (Abeledo, 2006b)

Albornoz, por su parte, en su artculo


"Universidad y nuevas tecnologas"
atribuye a la Universidad la funcin de
"crear, atesorar y difundir el
conocimiento cientfico, tanto a travs
de la formacin de profesionales, como
de la prestacin de servicios a la
sociedad" (Albornoz, 1990b. 263).
Dicho en otras palabras, debe asumir en
todos los niveles el fenmeno de las
nuevas tecnologas.
Pero este ideal es por el momento difcil
de alcanzar para los pases

latinoamericanos: el cambio tecnolgico


y la respectiva adaptacin del sistema
educativo comenzaron en los pases
industrializados y desde all se siguen
produciendo ondas expansivas de
innovaciones que abren con los pases
de la periferia brechas tecnolgicas y
problemas en la divisin internacional
del trabajo.
Otra consecuencia para los pases en
vas de desarrollo por esta falta de
protagonismo en la produccin de
tecnologas es la dificultad para
constituirse en culturas tecnolgicas;
existe ms bien un dualismo tecnolgico
provocado por la presencia de empresas
de tecnologa de punta, por lo general
pertenecientes a empresas

multinacionales, y otras retrasadas y


obsoletas. Tambin en este orden
pueden mencionarse la infrautilizacin
de la capacidad productiva, la falta de
incentivos en la inversin local y la
ausencia de alternativas tecnolgicas
viables.
La Universidad refleja estas tensiones
polticas, econmicas y sociales dando
lugar a falsas dicotomas como
"masividad" versus "excelencia", o a
una situacin de incapacidad para
adaptar sus estructuras a la
incertidumbre del cambio tecnolgico
(qu carreras deben priorizarse, cul es
la duracin real de las carreras?).
La Universidad de Buenos Aires es la
institucin argentina que, pese a sus

deficiencias estructurales, cuenta con


una importante tradicin de
investigacin creativa en ciencia, con la
mayor cantidad de investigadores y con
el prestigio conferido por los Premios
Nobel a sus graduados (B. Houssay,
medicina, 1947; L. F. Leloir, qumica,
1970 y C. Milstein, medicina, 1984).
No ha avanzado, sin embargo, en
relacin con las urgencias planteadas
por las nuevas tecnologas. Para
alcanzar el nivel adecuado, Albornoz
entiende que es necesario entender la
investigacin bsica como investigacin
estratgica, de acuerdo con la
proximidad que hoy tiene la ciencia
bsica con la industria. En el momento
en que Albornoz publica su texto, existe

una escasa cantidad de investigadores


dedicados a disciplinas estratgicas
bsicas (biotecnologa, biologa celular
y molecular, microbiologa y gentica) o
a las aplicadas (microbiologa aplicada,
inmunologa, etc.).
Ha cambiado esta situacin en la
actualidad? Como dijimos antes, ha
cambiado en los proyectos pero no en
los hechos, o no lo suficiente. Un caso
ilustrativo: en 2007 el Ministerio de
Economa lanz el Plan Productivo
Nacional. En l se destaca un concepto
clave para la cuestin del desarrollo
nacional que estamos examinando: el
concepto de competitividad.
Transcribimos las afirmaciones de Juan
C. Del Bello y C. Abeledo sobre el

significado de este concepto:


La competitividad es entendida como el incremento
sistemtico del nivel de productividad de la
economa y la transformacin de la estructura
productiva, a nivel de sectores, cadenas de valor y
conglomerados productivos, que combinen un alto
grado de innovacin y que no sean islas de
modernidad. El Plan afirma: "El reto de la poltica
productiva para la Argentina es, entonces,
promover un cambio que transforme la estructura
productiva a favor de actividades que incorporen
ms innovacin y que contribuyan al desarrollo de
un tejido productivo ms denso, en forma
consistente con los ejes rectores de aumento de las
exportaciones, la inversin y la competitividad."
En cuanto a los lineamientos sectoriales se
promueven las actividades de "alto impacto
potencial": biotecnologa, nanotecnologa y energas
renovables, los "sectores innovadores":
agroalimentos, farmo y agroqumica, TICs,
tecnologa nuclear, aeroespacial y satelital, bienes
de capital y servicios de alto valor agregado
(turismo, industrias culturales, diseo, etc.) y los

"sectores tradicionales". El documento explicita la


necesidad de la "coordinacin interministerial",
especficamente del Plan Estratgico Nacional de
Ciencia, Tecnologa e Innovacin, el Plan
Estratgico Territorial y el Plan Nacional de
Inversin Pblica. (Del Bello y Abeledo, 2007)

Lamentablemente, si bien la finalidad


que el plan enuncia es la correcta, falla
en la especificacin de cuestiones
bsicas que debera contemplar. No
establece, por ejemplo, "la
institucionalidad necesaria para
asegurar una buena gobernanza del
Plan", es decir, no indica mediante qu
mecanismos se van a relacionar las
actividades de investigacin con el
sector productivo y con el de
financiamiento.
Como hemos sealado a propsito del
funcionamiento del CONICET, los

institutos de investigacin no se han


adecuado a los lineamientos
establecidos por los planes y proyectos
gubernamentales. Dicen los autores:
[Los organismos estatales] Padecen de baja
autonoma respecto de la administracin central,
cuentan con rgidos sistemas de personal e
inflexibles regmenes de remuneraciones, no tienen
flexibilidad y agilidad para generar recursos propios
y disponer de ellos, as como de su patrimonio. En
fin, son instituciones pensadas para el enfoque
superado de la "transferencia de tecnologa", que
supone una situacin de generacin y oferta de
tecnologas por parte del Estado, y un sector
empresarial pasivo "demandante". El enfoque de los
sistemas nacionales de innovacin ha superado esa
visin, pero las instituciones continan funcionando
con el viejo enfoque. Los "involucrados/actores"
(stakeholders) no tiene participacin en los
gobiernos de las instituciones, o la participacin es
de naturaleza corporativa (representantes del
sector privado en el Directorio del CONICET sin

empresas, por ejemplo). Mientras que otros pases


de la regin han reestructurado instituciones o
generado nuevas, con un enfoque basado en la
asociatividad y en el desarrollo de redes, los
institutos estatales argentinos permanecen
congelados en el tiempo. (Del Bello y Abeledo,
2007)

Otra condicin que puede favorecer el


crecimiento en materia de investigacin,
segn Albornoz y los dems autores que
estamos viendo, es la integracin de la
investigacin con la estructura docente,
es decir, integrar la investigacin en la
Universidad y proyectarla realmente en
los planes de estudio.
Hacer esto significara adecuar la
Universidad, institucin rgida por
naturaleza, al dinamismo del cambio
tecnolgico, es decir, tornarla flexible y
cambiante, significa pedirle que "no

ensee tanto conocimientos objetivos,


como que entrene para la discusin de
alternativas, opciones, riesgos y
ganancias potenciales", que reconozca
que "el cmo que transfiere el
conocimiento se ha convertido en algo
ms importante que el propio
conocimiento transferido" (Albornoz,
1990b. 271).
Conclusiones
Hasta 1990, la Argentina, y Amrica
Latina en general, presentaban un atraso
radical respecto de los pases
industrializados en materia de polticas
educativas orientadas al desarrollo y el
crecimiento socio-econmico. Durante
la dcada de los 90 se comienza el
intento de recuperar el terreno perdido

pero con polticas equivocadas, tal


como lo sealan Chudnovsky y Lpez
(1996), por eso no se produjo el cambio
estructural esperado. En la dcada
siguiente, que llega a nuestros das, se
intenta dar nuevo impulso a la
investigacin cientfica y tecnolgica
orientada a la innovacin, ya no
dependiendo de la inversin extranjera
como en la dcada anterior, sino
fortaleciendo la infraestructura existente
mediante la creacin de organismos
adecuados y planes conceptual y
tcnicamente a la altura de las
exigencias internacionales. El contenido
del Proyecto de Plan Nacional de
Ciencia, Tecnologa e Innovacin
Productiva de 2003, 2004 y 2005,

elaborado por la Secretara de Ciencia,


Tecnologa e Innovacin Productiva,
corrobora lo que acabamos de decir. Sin
embargo, los cambios que tales
propuestas de avanzada introducen no se
han concretado en las acciones
correspondientes.
Como se ha expresado a lo largo del
presente documento, la inercia de los
organismos de investigacin, el
academicismo universitario nacional, la
desvinculacin de ambos con las
necesidades sociales y las empresas, a
lo que se suma la no intervencin del
Estado en estas cuestiones, impiden
realizar las modificaciones necesarias.
Para finalizar, se transcribe una
reflexin de Mario Albornoz que reitera

todo lo dicho hasta aqu de forma muy


contundente:
Hoy, en cierta medida, en ciencia y tecnologa
vivimos del capital acumulado en las dcadas de los
cincuenta y los sesenta, ya que ellas fueron
prdigas en esfuerzos y resultados. En aquellos
aos se consolid la capacidad cientfica del pas,
se crearon sus principales instituciones (como el
CONICET, el INTA, el INTI, la CNEA) y las
universidades nacionales alcanzaron un alto nivel en
materia de investigacin y se constituyeron en los
principales ncleos impulsores de la investigacin
cientfica.

El panorama actual es un reflejo


empobrecido del pasado esplendor.
Buenos ejemplos actuales, como los de
la empresa INVAP (que es una sociedad
del Estado perteneciente a Ro Negro) y
la Comisin Nacional de Actividades
Espaciales (CONAE), son excelentes
casos de capacidades tecnolgicas,

aunque lamentablemente aisladas, en un


escenario con un perfil productivo de
escaso valor agregado.
El modelo neoliberal, pero tambin la
incapacidad puesta de manifiesto por los
sectores ms progresistas para generar
un nuevo pensamiento movilizador,
carece de motivos para formular
polticas que tiendan a aprovechar al
mximo las capacidades disponibles. De
este modo, un pas que dcadas atrs
pudo producir premios Nobel y
desarrollar tecnologa propia, hoy no
puede sostener el esfuerzo cientficotecnolgico que requiere el mundo de fin
de siglo (Albornoz, 2003. 2).
Bibliografa
Abeledo, C. (2004a). La investigacin

en la Universidad: Creacin de
conocimiento o desarrollo tecnolgico?
En Universidad, Sociedad y
Produccin. (J. C. Pugliese, Ed.).
Buenos Aires.
Abeledo, C. (2006b). Las necesidades
de recursos humanos para el desarrollo
del sistema nacional de innovacin.
Albornoz, M. (1990a). Consideraciones
histricas sobre la poltica cientfica y
tecnolgica en la Argentina. En
Albornoz, M. y Kreimer, P. Ciencia y
tecnologa: estrategias y polticas de
largo plazo. Buenos Aires: Eudeba.
Albornoz, M. (1990b). Universidad y
nuevas tecnologas. En Albornoz, M. y
Kreimer, P.
Albornoz, M. Ciencia y tecnologa en

la Argentina: capacidades sin sustento,


2003.
<http://www.escenariosalternativos.org>
Bunge, M. (1997). Ciencia, tcnica y
desarrollo. Buenos Aires:
Sudamericana.
Chudnovsky, D. (1999). Polticas de
ciencia y tecnologa y el Sistema
Nacional de Innovacin en la Argentina.
Revista de la CEPAL. N 67.
Chudnovsky, D. y Lpez, A. (1996).
Poltica tecnolgica en la Argentina: hay
algo ms que laissez faire? Redes,
revista de estudios sociales de la
ciencia (vol. III). Universidad Nacional
de Quilmes.
Ciapuscio, H. (1990). Formacin
intensiva de recursos humanos: una

necesidad imperiosa. En Albornoz, M. y


Kraimer, P.
Ciapuscio, H. (1994). El fuego de
Prometeo: tecnologa y sociedad.
Buenos Aires: Eudeba.
Del Bello, J. C. y Abeledo, C. (julio de
2007). Reflexiones sobre cuestiones
pendientes de la Agenda de Poltica
Pblica en Ciencia, Tecnologa e
Innovacin de Argentina. Primer
Congreso Argentino sobre Estudios
Sociales de la Ciencia y la Tecnologa.
Universidad Nacional de Quilmes.
Yfera, E. P. (1994). Introduccin a la
investigacin cientfica y tencnolgica.
Madrid: Alianza.

LA RESPONSABILIDAD SOCIAL
DEL CIENTFICO. CRTICA DE LA
TEORA SOBRE LA
NATURALIDAD VALORATIVA DE
LA CIENCIA Y DE LA ACTIVIDAD
CIENTFICA. POLMICA
ENTRE CIENTIFICISMO Y
ANTICIENTIFICISMO
Mara Beatriz Contratti

Introduccin
La relacin entre tica y ciencia deriva
principalmente de la imperiosa
necesidad de la sociedad contempornea
de poner en orden o contener los
resultados de las investigaciones

cientficas que se hacen tangibles para la


mayor parte de los habitantes del planeta
a travs de la tecnologa. Desde luego,
la tica no es el nico modo a travs del
cual puede llevarse a cabo esa tarea de
ordenamiento. Tambin lo hacen la
jurisprudencia, la poltica y la religin,
por ejemplo. Pero el fenmeno de la
ciencia y tecnologa ha desbordado las
posibilidades de esos tres mbitos que
se han ocupado tradicionalmente de
generar o mantener orden social. Por
qu esto parece ser as, se ir viendo a
medida que se avance en el desarrollo
de los temas.
Una de las primeras tareas que se
abordarn aqu, entonces, ser la de
esbozar brevemente los rasgos de la

sociedad actual que, precisamente al


estar caracterizada por la impronta de la
ciencia y la tecnologa, se suele
denominar sociedad del conocimiento o
sociedad de la informacin.
En segundo lugar, se har una breve
sntesis de lo que es la tica, en tanto
marco normativo de la conducta humana
y en tanto disciplina filosfica, para
referirnos despus a la tica aplicada,
terreno al cual pertenece en parte el
tema de este texto. Estas nociones
permitirn incursionar en la cuestin
especfica que nos ocupa: los aspectos
ticos de la ciencia y/o de la tecnologa.
Sobre todo, se intentar hacer hincapi
en el modo de reflexin que opera en
este campo, con el propsito de poner

en evidencia los problemas o


cuestionamientos que ms comnmente
se formulan a la ciencia y la tecnologa
desde diversos sectores sociales. Se
ver que es el concepto de
responsabilidad el que predomina en
los juicios ticos que tienen por objeto a
la tecnociencia.
Una vez aclaradas estas cuestiones
bsicas preliminares, es posible
examinar uno de los aspectos de la tica
de la ciencia: la investigacin cientfica.
En sta se pone de relieve la conducta
del investigador a la luz de la
correccin o incorreccin de sus
acciones en relacin con la produccin
de conocimiento. Como todo trabajo o
profesin, la tarea del investigador debe

ajustarse a ciertos estndares


establecidos, algunos generales y otros
peculiares a su campo, para que el
resultado de su tarea sea el adecuado.
Se analizarn algunos de esos estndares
o reglas de conducta.
Pronto se ver que la tica de la ciencia
no puede reducirse a juzgar la tarea del
cientfico como si de eso slo
dependiera la obtencin de resultados
confiables de las investigaciones,
entendiendo por resultado confiable
teoras ciertas y objetivas o altamente
probables. La conducta del cientfico
ajustada a normas ticas es
imprescindible en una investigacin
seria, pero el problema de la tica de la
ciencia es ms complejo y difcil de

tratar. Incluye el anlisis y crtica del


conocimiento mismo surgido de las
investigaciones, aun de aquellas que
respetan todos los protocolos requeridos
por la labor cientfica. El anlisis
epistmico de la investigacin y las
teoras cientficas mostrar que la
pretensin de obtener conocimiento
altamente confiable, objetivo, es una
empresa mal encaminada. El
conocimiento, desde este anlisis,
resulta ser ms bien "construido" que
"obtenido". Las conclusiones de esta
forma de ver el conocimiento de la
ciencia tienen importantes implicancias
en el tema que nos ocupa.
Esto lleva, finalmente, a la distincin
entre cientificismo y anticientificismo,

trminos estos desde los que se ha


planteado la polmica entre dos posturas
opuestas: quienes piensan que la ciencia
es neutral respecto a valores, es decir,
es conocimiento "obtenido" acerca del
mundo por procedimientos confiables,
no contaminado con intereses o
tendencias personales, polticas o
ideolgicas (cientificismo), y quienes
creen que la ciencia es conocimiento
"construido", lo que hace inevitable que
todos esos sesgos e intereses confluyan
en su produccin (anticientificismo).
Es preciso aclarar que la postura
cientificista, que proporciona una
imagen de la ciencia como conocimiento
imparcial y objetivo, tiene an, a pesar
de que los recientes estudios sobre la

ciencia han mostrado que esa imagen es


inadecuada, una influencia muy grande
en las capas dirigentes de la sociedad,
lo que conlleva una unin ntima entre
ciencia y poder, o conocimiento y poder.
Como cierre, se intentar brindar un
panorama del mundo actual que recoja
todos los aspectos de la relacin
ciencia-tica que se han ido
comentando. Se dejar claro que cuando
se habla de ciencia se quiere decir en
realidad "tecnociencia" en todos los
casos, pues una y otra, a diferencia de lo
que piensa el cientificista, son, en la
prctica, difciles de distinguir y, a los
efectos de considerar las consecuencias
de la aplicacin del conocimiento
cientfico y tecnolgico en el mundo

social y natural, son indistinguibles.


Caracteres de la sociedad
contempornea
"Saber es poder", el famoso aforismo
enunciado por el filsofo ingls Francis
Bacon[37] en el siglo XVII, parece
encontrar en la sociedad actual un
significado pleno. Efectivamente, en la
segunda mitad del siglo pasado se
caracteriz a la sociedad contempornea
como la "sociedad de la informacin" y
"sociedad del conocimiento" justamente
porque la informacin y el conocimiento
constituyen los recursos fundamentales
que nutren su cultura y resultan
condicin de su mantenimiento y
desarrollo. El alto valor adquirido por
el saber en el presente se relaciona con

la difusin de las llamadas "tecnologas


de la informacin", por un lado, y, por
otro, con la preponderancia adquirida
por el conocimiento cientfico sobre
cualquier otro tipo de conocimiento. Las
tecnologas de la informacin presiden
el desarrollo tecnolgico en general
puesto que toda la tcnica actual
depende para su funcionamiento en
forma directa o indirecta de la primera.
A su vez, la tecnologa de la
informacin ha sido posible gracias a la
comprensin por parte de la ciencia de
los mecanismos que permiten a los seres
vivos en general interactuar con su
medio, de los procesos cognitivos
estudiados por la psicologa, de los
conocimientos matemticos avanzados y

del inters en transformar los


dispositivos mecnicos en automticos
en beneficio de la produccin
industrial.[38]
Es fcil advertir que la complejidad y
profusin del panorama que presenta la
cultura cientfica y tecnolgica
contempornea hace que los fenmenos
como la ciencia y la tecnologa no se
dejen apresar en definiciones o
caracterizaciones simples. Una
afirmacin como la siguiente: "a la
ciencia slo le interesa ofrecer
conocimiento del mundo", resulta
demasiado estrecha si tomamos en
cuenta las diversas motivaciones que
impulsan a los cientficos a realizar su
trabajo y no considerar que el

conocimiento sobre el mundo es algo tan


problemtico y discutido que hace poco
plausible dicho ofrecimiento. De igual
modo, sostener, como es habitual, que la
tecnologa tiene como objetivo no
conocer el mundo, sino "slo
transformarlo", es realizar un recorte
sumamente arbitrario en la compleja red
formada por propsitos, saberes,
historia, relaciones sociales y otras
cuestiones que son inherentes al proceso
tecnolgico.
Se hace necesario, pues, proporcionar
una caracterizacin de la ciencia y la
tecnologa ms ajustada a la diversidad
y complejidad de su papel en el mundo
contemporneo, y esta necesidad no se
debe slo a razones de ndole

conceptual, sino sobre todo a esclarecer


las relaciones entre la ciencia y la
tecnologa con la sociedad. Es obvio
que ellas han sido creadas y
desarrolladas por el hombre para hacer
cada vez ms confortable y satisfactoria
su vida, pero este propsito natural y
original parece desvirtuarse en la
actualidad cuando la ciencia o la
tecnologa, cada una por razones a veces
muy diferentes y otras veces semejantes,
se independizan de la sociedad a la que
pertenecen y cobran un poder autnomo
mediante el cual dejan de servir a la
humanidad para convertirse ms bien en
sus amos despticos. Cmo y por qu ha
sucedido esto est ya insinuado en la
Introduccin y se profundizar en el

resto del documento.


Esta cuestin del poder adquirido por la
ciencia y la tecnologa sobre el resto de
las fuerzas sociales, poder que en ltima
instancia tiene una raz poltica, ha sido
denunciada y analizada por muchos
especialistas en cuestiones sociopolticas, filsofos de la ciencia y de la
tecnologa, cientficos, tecnlogos y
analistas provenientes de muchas otras
reas. En este artculo se destacar el
aspecto tico de esas relaciones. Es sta
una cuestin que requiere realizar
distinciones y precisiones porque el
rtulo "los problemas sociales y ticos
de la ciencia y la tecnologa" admite
diversas interpretaciones y enfoques que
son muy discutidas o, al menos,

discutibles.
La tica y sus relaciones con la ciencia
y la tecnologa
Antes de hablar sobre las relaciones
entre ciencia, tecnologa y sociedad es
preciso aclarar qu se entiende por tica
en general y especialmente en el marco
de los estudios sociales de la ciencia.
En trminos muy abarcadores, la tica es
una disciplina filosfica que se ocupa de
la justificacin racional de las normas
morales que regulan la conducta humana
individual y social. Las normas morales
nos dicen qu es correcto hacer y qu no
lo es, tanto en relacin con nosotros
mismos como en relacin con los dems.
Dichas normas se expresan en juicios
morales que, si se consideran en

conjunto, suelen revelar aspectos


importantes de la idiosincrasia de
grupos o pueblos, como podra ser el
ideal de vida buena que una comunidad
comparte. En relacin con la accin
humana, entonces, en dichos juicios se
articulan conceptos como bueno, malo,
correcto, incorrecto, justo, injusto,
deber, obligacin, prohibicin y otros
que aluden a las creencias que tiene la
gente sobre la conducta propia y ajena y
lo que est corporizado en las
instituciones sobre lo que es moralmente
aceptable. El concepto vinculado con
las cuestiones ticas de la ciencia y la
tecnologa que ms peso tiene
actualmente, dada la influencia que
tienen la ciencia y la tecnologa sobre el

destino de las sociedades y los hombres,


es el concepto de responsabilidad.
Como sucede con la gran mayora de los
conceptos filosficos, el significado de
la nocin de responsabilidad es
problemtico. Aqu se tomarn en cuenta
slo aquellos sentidos que sean
relevantes a las cuestiones de la tica de
la ciencia. En primer lugar, habra que
distinguir un sentido causal y un sentido
normativo de responsabilidad: la falta
de lluvias es causa de la sequa y, por lo
tanto, de la ruina de las cosechas, pero
la falta de lluvias no es moralmente
responsable de esa catstrofe. Pero si
alguien arroj alguna sustancia
perjudicial a los cultivos, entonces ese
alguien s es responsable moral y/o

legalmente de su ruina.[39]
Parece, pues, que un elemento de
intencionalidad es necesario para
adjudicar responsabilidad por la
consecuencia de una accin, por eso en
el lenguaje del derecho se dice que
quien causa dao a otro tiene la
obligacin de "responder" o es
"responsable" o tiene "responsabilidad"
por el dao sufrido por la otra persona.
Desde luego, es preciso realizar
posteriores especificaciones respecto de
las condiciones psicofsicas que debe
reunir una persona para ser considerada
responsable de sus actos, pero sta es
una cuestin que urge dilucidar ms
desde el sentido jurdico que desde el
moral. Es necesario adems aclarar que

estos dos tipos de evaluaciones del


comportamiento humano no siempre
coinciden sobre las distinciones de los
actos (correctos o incorrectos) y su
correspondiente nocin de
responsabilidad. Un ejemplo: desde el
punto de vista jurdico no es imputable
mentir a un allegado por piedad o
conmiseracin, pero desde un punto de
vista moral puede crear un dilema, y
desde la perspectiva tico-filosfica, es
posible plantearse si la regla que dice
"no mentir" admite excepciones.
De acuerdo con estas consideraciones,
cabra preguntarse por qu y en qu
medida se puede hablar de la
responsabilidad moral de la ciencia y la
tcnica. Si, como se ha dicho, los

sujetos de la responsabilidad son las


personas o los individuos, son los
cientficos o tecnlogos quienes pueden
ser imputados en ese sentido. Pero la
responsabilidad moral no es slo
atribuible a los individuos, tambin es
lcito plantear la existencia de la
responsabilidad colectiva. Justamente,
los problemas socio-ambientales
ocasionados por la ciencia y la
tecnologa actual han llevado a
considerar el concepto de
responsabilidad ms all de los
individuos que intervienen circunstancialmente en la produccin de un
fenmeno dado; los desastres ecolgicos
producidos por derrame de petrleo, las
transfusiones realizadas con sangre

contaminada con el VIH y muchos otros


casos conocidos ilustran la cuestin de
la responsabilidad colectiva. Es
indudable que el concepto plantea
muchos interrogantes difciles de
solucionar,[40] pero su empleo viene
exigido por las peculiares situaciones
conflictivas de la vida contempornea.
Por otra parte, a raz de los diversos
perjuicios en la salud y el
medioambiente derivados de la
implementacin de las teoras cientficas
y tecnolgicas, es que ha nacido en los
ltimos tiempos una disciplina filosfica
denominada "tica aplicada". En
principio, la tica aplicada sera, valga
la redundancia, la aplicacin de la tica
terica, es decir, aquella que analiza

filosficamente las normas morales, a


casos particulares. Cmo se realiza tal
aplicacin, o sea, los mtodos y
procedimientos, si los hay, que
permitiran "bajar" al campo de los
hechos concretos los principios
tericos, es materia de ardua discusin.
Lo que interesa en este texto es mostrar
cmo el uso generalizado de la tica
aplicada est sealando la ntima
relacin que hay entre ciencia,
tecnologa y tica, puesto que son esos
dos primeros fenmenos tpicamente
contemporneos los que han elevado la
necesidad de apelar a la tica para
resguardar los derechos a la vida de los
habitantes de la Tierra expuestos a
diversos peligros por el accionar de

aquellas. Precisamente, el tema tratado


en este artculo se encuadrara en una
reflexin general sobre cuestiones de
tica aplicada a la ciencia y la
tecnologa en tanto productores de
conocimiento y las prcticas que ello
implica. Existen dentro del campo de la
tica aplicada otras disciplinas que se
ocupan de los problemas especficos
ocasionados por la ciencia y la
tecnologa en ciertos sectores del mundo
natural y social. As se puede hablar de
"tica del medioambiente", "tica
mdica" o "biotica", "tica aplicada al
tratamiento de los animales (y plantas)",
etc. Todos estos mbitos tienen a la
investigacin cientfica o al
conocimiento cientfico y tecnolgico

como marco de referencia, pero no


cuestionan directamente el conocimiento
mismo o el accionar de la ciencia y la
tecnologa como instituciones
generadoras de saber, como se hace
aqu, sino sobre todo a los conflictos
sociales y morales que provoca su
aplicacin. Es preciso aclarar, por otro
lado, que ambas perspectivas, la que
considera las consecuencias de la
aplicacin del conocimiento y la que se
interesa por el conocimiento mismo,
interaccionan en forma continua en los
estudios de tica aplicada.
Otra distincin aplicable a la cuestin
de la responsabilidad moral de la
ciencia y la tecnologa (a las personas o
al colectivo) es la establecida entre

responsabilidad retrospectiva y
responsabilidad prospectiva. La
primera se refiere a los resultados de las
acciones ya realizadas, por ejemplo, el
haber permitido que se arrojaran los
residuos industriales en el Riachuelo, y
la segunda, a la exigencia de prever los
posibles resultados de las acciones. El
segundo sentido, del que se pueden
encontrar infinidad de ejemplos, tiene un
inters especial pues plantea a la ciencia
y la tecnologa la exigencia de conservar
el planeta en buenas condiciones para
las generaciones venideras.
Sin embargo, no todos acuerdan en que
el "conocimiento" cientfico y
tecnolgico pueda ser en parte
responsable de los daos

medioambientales, mdicos, etc. que


ocurren en el mundo actual. Desde un
punto de vista por dems optimista, se
suele afirmar que la ciencia y la
tecnologa estn ms all de posibles
objeciones de naturaleza tica porque
son buenas en s mismas, lo cual
quedara probado por las mltiples
formas de progreso con que han
beneficiado a la humanidad.
Efectivamente, da a da sorprenden al
mundo nuevos artefactos y procesos
innovadores que salvan vidas, evitan los
efectos, o al menos los atenan, de las
catstrofes naturales, incrementan la
posibilidad de la comunicacin entre las
personas y, en fin, proporcionan todos
los medios para lograr el bienestar

material, psicolgico y espiritual que


necesitan los seres humanos. Y esto es,
sin duda, cierto. Pero desde otras
perspectivas se muestran los efectos
adversos e irreversibles que acompaan
a los beneficios. Muchas veces las
mismas tcnicas y productos que
permiten salvar vidas ocasionan otros
malestares y dolencias, y no pocas veces
esos malestares y dolencias terminan en
daos irreversibles: la represa que
proporciona la energa elctrica tan
necesaria en un mundo superpoblado
tambin provoca la prdida de valiosas
especies naturales y altera el microclima
de la regin donde se construy,
perturbando de ese modo el sabiamente
aceptado modo de vida de las

poblaciones cercanas, al punto de causar


su extincin por la ruptura del equilibrio
ecolgico, la comodidad que brindan los
medios de transporte modernos se ve
disminuida en su valor por la
contaminacin que produce, y como
stos hay gran cantidad de otros
ejemplos que muestran los efectos
nocivos que en forma directa o potencial
puede acarrear la implementacin de las
distintas tecnologas.
Un caso que ha resultado paradigmtico
en aos anteriores en los estudios de
ciencia y tecnologa es el de la energa
nuclear. En este mbito se ve claramente
el rostro dual de la ciencia y la
tecnologa. Si fuera posible dominar los
procesos que rigen la desintegracin o

fisin del tomo de modo que su


utilizacin fuese segura, los beneficios
seran cuantiosos. Pero la leccin que
dej la catstrofe de Chernobyl[41] fue
suficiente para saber que est lejos de
alcanzarse este ideal, y que los peligros
que entraa la manipulacin de la
energa nuclear por ahora se consideran
inevitables, como tambin lo atestigua el
problema de los residuos nucleares. La
decisin de poner en marcha una central
nuclear exige una evaluacin rigurosa de
los riesgos que implica. Pero, es
posible realizar una evaluacin tal?
Hay algn procedimiento establecido
desde el cual se pueda determinar con
una precisin aceptable el riesgo
potencial que supone el funcionamiento

de una planta nuclear? Las evaluaciones


de riesgo se realizan en general por
procedimientos estadsticos, pero como
la estadstica opera sobre variables
preestablecidas es bueno preguntarse
hasta qu punto ese mtodo resulta
totalmente satisfactorio para los
habitantes de una regin donde se
encuentra una planta nuclear. La
estadstica nos da alguna informacin
sobre la correlacin entre ciertas
variables seleccionadas por el
evaluador, pero bien analizadas las
cuestiones referidas a las evaluaciones
de riesgo van mucho ms all de los
porcentajes que proporciona la
estadstica. Lo que la gente quiere saber
para aceptar o tener cierta seguridad

sobre una determinada tecnologa, tiene


menos relacin con nmeros que con
modos de vida, esperanzas, temores,
creencias enraizadas en la tradicin y
perspectivas sobre el futuro.
La consideracin de estos factores por
parte de quienes dirigen el aspecto
poltico y social de la ciencia y la
tecnologa significara adoptar un
modelo de evaluacin en el que se
considere el riesgo, no como algo
abstracto y determinable en forma
cuantitativa o tcnica solamente, sino
pensndolo desde una dimensin
integralmente humana. En otras palabras,
este modo de encarar el control de la
ciencia y la tecnologa implica asumir
sobre todo una perspectiva tica y no

meramente tcnica en dicho control.


Silvio O. Funtowics y Jerome R. Ravetz,
filsofos de la ciencia contemporneos,
argentino e ingls respectivamente,
hablan en este sentido de una
"comunidad de evaluadores extendida"
que es requerida cuando la evaluacin,
como en el caso de la complejidad del
proyecto de construccin de una represa,
"no depende de manera esencial de la
diversidad de disciplinas cientficas
relevantes, sino que, ms bien, consiste
en la multiplicidad de perspectivas
legtimas desde las que es posible
contemplar el problema en su
totalidad" (1997. 156). La posibilidad
de concretar este tipo de evaluaciones
se sostiene, segn los autores, en lo

siguiente: "En las sociedades


modernas, incluyendo tanto las ricas
como las pobres, hay un gran nmero
de gente comn que puede leer,
escribir, votar y debatir. La
democratizacin de la vida poltica es
algo usual hoy por hoy; sus riesgos se
aceptan como un pequeo precio a
pagar a cambio de sus beneficios"
(1997. 159). Estos beneficios resultan
de "el logro de un sistema que, a pesar
de sus ineficacias, es el medio ms
eficaz de evitar desastres originados
por el prolongado silenciamiento de
las crticas. Experiencias recientes han
mostrado que tal presencia crtica es
tan importante para nuestros
problemas ambientales y tecnolgicos

como lo es para la sociedad" (1997.


159).
La tica de la investigacin cientfica
Es preciso convenir, por lo pronto, que
el proceso de produccin de
conocimiento cientfico y tecnolgico
involucra cuestiones ticas que
comprometen al cientfico en tanto ser
humano que tiene intereses, porta
valores personales, incurre en
parcialidades y errores y otros rasgos
conductuales comunes a todos. Algunas
normas de conducta exigidas a los
cientficos, en tanto individuos, son las
exigibles a cualquier profesional o
trabajador, como la honestidad, la
veracidad, etctera, y otras estn
asociadas a la puntillosa observacin de

los pasos del mtodo cientfico y las


condiciones de su aplicacin. A raz de
este nexo entre regla metodolgica y
regla moral, que sera exclusivo del
mbito de la ciencia, muchos piensan
que ms que hablar de cdigos de
conducta o normas a las que el cientfico
debe ajustarse, es suficiente poner de
relieve los valores que son propios de
las actividades inherentes a la
investigacin. Estos valores seran de
diversos tipos: cognitivos,
metodolgicos y prcticos o morales, y
el tenerlos en cuenta y darles
cumplimiento en el proceso de
investigacin dara forma por s mismo
a la responsabilidad tica del
investigador. Otros, en cambio, piensan

que slo los valores o reglas cognitivas


y metodolgicas son de posible
consideracin en la ciencia, puesto que
sta se ocupa de conocer el mundo
nicamente y no realiza acciones que
apunten a cambiar ese mundo.[42] Los
valores prcticos o morales quedaran
fuera de toda consideracin. Pero esta
perspectiva resulta bastante
simplificadora y supone la aceptacin
de criterios que han sido muy objetados,
como la distincin tajante entre ciencia y
tecnologa. Es, pues, pertinente hablar
de una tica de la investigacin
cientfica en la que se entienda al
investigador cientfico como una
"persona" que investiga, sujeto por lo
tanto a una constelacin de restricciones

de orden moral que quedara formulada


en cdigos deontolgicos como en
cualquier otra actividad profesional.
En un documento de la National
Academy of Science de Estados Unidos:
On Being A Scientist: Responsible
Conduct in Research,[43] documento
redactado y supervisado por academias
y comits cientficos norteamericanos,
se examina el problema de la
investigacin cientfica en la actualidad
y los errores que se pueden cometer en
la investigacin daando gravemente no
slo el resultado de la misma, sino
tambin la confianza que la sociedad
puede otorgar a la ciencia.
Una primera cosa que los autores del
documento aclaran es que cuando se

habla del cientfico, no slo se alude a


un individuo particular, sino tambin a
una comunidad de cientficos, sin la cual
la ciencia actual en su complejidad y
entrecruzamientos de campos y
disciplinas no sera posible. Esta
comunidad cientfica tiene que enfrentar
en forma constante numerosos
problemas: el crecimiento de la ciencia
ha sido muy grande y rpido, y esto ha
creado dificultades con el
financiamiento, con los medios para
alcanzar las metas y nuevas y
problemticas relaciones con la
sociedad de la que forma parte. El peso
que la ciencia y la tecnologa tienen
sobre la sociedad contempornea ha
despertado el inters de los no

cientficos en evaluar las afirmaciones


de los cientficos, ya que su bienestar
hoy depende en alto grado de la
actividad de los ltimos.
Entre las cuestiones que ataen a la
responsabilidad personal del
investigador relacionadas con el
cumplimiento de las normas
institucionales explcitas o implcitas, o
de las que se desprenden de las
prescripciones metodolgicas, se
encuentran las siguientes:
Errores cometidos en el transcurso
de la investigacin: pueden ser
"errores honestos" que se producen
inevitablemente por razones
circunstan-ciales (falta de tiempo o

recursos financieros); en la medida


en que sean detectados en
publicaciones o artculos, deben
ser reconocidos por su autor, lo
cual seguramente ser considerado
con tolerancia por la comunidad
cientfica. Otro tipo de error
posible se puede deber a la
"negligencia" del investigador; el
error surge por precipitacin, falta
de cuidado, inatencin, etc. En este
caso, la reaccin de los pares ser
ms severa, y las consecuencias
que pueden tener los errores
debidos a la negligencia son ms
difciles de revertir que los
primeros. Pero los errores ms
graves son los que involucran

"engao" por parte del cientfico:


inventar datos o resultados o
falsificarlos y el plagio constituyen
los ms conocidos. Segn los
autores del documento On Being a
Scientist, stos no slo socavan el
progreso de la ciencia, sino el
conjunto completo de valores sobre
los cuales se sustenta la misma. Las
consecuencias de tales acciones no
es asunto interno solamente, pues
su correccin no se dirime
exclusivamente en el mbito de la
comunidad cientfica, sino tambin
en tribunales externos.
La adjudicacin de crditos o
recompensas:[44] segn este
documento de la National Academy

of Science, en el sistema estndar


de la ciencia la adjudicacin del
crdito por el mrito de los
trabajos realizados es reconocido
en tres lugares: en la lista de
autores, en el reconocimiento por
las contribuciones de otros y en la
lista o referencias de las citas. En
estos lugares pueden surgir
conflictos por una inadecuada
atribucin del crdito o
reconocimiento. Varias son las
consecuencias cuando las citas no
se realizan como es debido:
perjudican tanto al investigador en
su prestigio y su carrera como al
sistema de publicaciones
cientficas. Los nombres de los

autores de un proyecto de
investigacin, el primero de los
lugares mencionados, debe
preestablecerse al desarrollo de la
investigacin, para no crear
conflictos posteriores. La
contribucin de cada miembro
suele determinarse de acuerdo con
el rango que ostenta: graduado,
estudiante, etc., pero una cuestin
importante es que estn
mencionados "todos los
colaboradores".
El tratamiento de las tcnicas
experimentales: las observaciones
realizadas por los cientficos a
travs de las tcnicas
experimentales estandarizadas

permite la verificacin
independiente de los datos. En la
medida en que el cientfico usa
esas tcnicas, los resultados de su
trabajo pueden ser reproducidos
por otros cientficos y esto
favorece su confiabilidad. Pero
tanto los mtodos como el
conocimiento obtenido mediante
ellos no son infalibles, deben
someterse a continua revisin y
control, de lo contrario se puede
caer en fatales errores. El
escepticismo es por esta razn una
caracterstica de la ciencia
presente en todos los momentos de
la investigacin.
La publicacin y la apertura: la

ciencia no es una experiencia


individual sino conocimiento
compartido basado en la
comprensin comn de algunos
aspectos del mundo fsico o social.
Para garantizar la confiabilidad de
este conocimiento la ciencia cuenta
con ciertas convenciones sobre la
forma en que deben difundirse los
resultados de la investigacin. La
principal es la publicacin de esos
resultados. Se pueden originar
conflictos sobre la prioridad en los
descubrimientos, ya que el primero
que publica es el que obtiene el
crdito, no el primero en descubrir
algo nuevo. Quien publica es
reconocido por sus pares a travs

de la cita en otras publicaciones


que usan sus hallazgos, pero debe
cumplir una condicin bsica: su
trabajo tiene que haber pasado el
control de calidad efectuado por
sus colegas. De lo contrario, puede
caer en errores o dar lugar a
interpretaciones equivocadas.
Conflictos de intereses: el
cientfico tiene que emplear su
propio juicio para interpretar
datos, para encontrar problemas de
investigacin y cundo concluir un
experimento. Para ello se necesita
desarrollar ciertas habilidades que
se pueden aprender, pero no
totalmente. Cuenta mucho la
creatividad y la intuicin, que son

condiciones personales. En estos


casos los juicios empleados por los
cientficos comportan valores.
Algunos de estos son de naturaleza
epistmica, como la consistencia,
la precisin en las predicciones, la
capacidad de unificacin de
observaciones diferentes, la
simplicidad y la elegancia. Otros
son valores personales, religiosos,
filosficos, culturales, polticos o
econmicos, que forman el juicio
del cientfico de manera
fundamental. Un punto importante
desarrollado por los autores del
Documento es el de la influencia
positiva o negativa de los valores
en el desempeo del cientfico en

tanto tal. Piensan que a veces


favorecen la investigacin y otras
la entorpecen; un ejemplo de lo
primero es el deseo de realizar una
buena investigacin, as como
aceptar los estndares de
honestidad y objetividad propios
de la ciencia y un ejemplo de lo
segundo es cuando desde el campo
de la eugenesia (aplicacin de las
leyes biolgicas de la herencia al
perfeccionamiento de la especie
humana), se usan las tcnicas de la
ciencia para intentar mostrar la
inferioridad de ciertas razas. Los
autores creen que cuando la
investigacin corre peligro de
distorsin por la influencia de los

valores de los cientficos, los


mecanismos correctivos de la
ciencia, como el control emprico
de las afirmaciones y el consenso
de la comunidad cientfica, ponen
lmites a esa posibilidad. Adems,
la formacin del cientfico debe ser
lo suficientemente amplia a fin de
que pueda tomar conciencia de las
suposiciones y creencias que
subyacen a las elecciones y
decisiones que debe enfrentar en su
trabajo y as, en lo posible,
neutralizarlos cuando afectan
negativamente los objetivos de la
ciencia. Esa formacin debe incluir
el aprendizaje de cuestiones no
cientficas, como las que

proporcionan las ciencias sociales,


la religin y la tica.
Las consideraciones que se acaban de
formular se relacionan con dos
cuestiones que posiblemente sean las
que ms influyen en los problemas
ticos de la ciencia: el aislamiento del
cientfico del resto de la sociedad y la
percepcin exclusivamente tcnica de
los conflictos que suscita su actividad.
Pero por s solas, estas consideraciones
no agotan la cuestin de la
responsabilidad de la ciencia, razn por
la cual deben ser ajustadas y
profundizadas.
Si bien la tica de la investigacin tal
como la expone la National Academy of

Science destaca aspectos importantes de


la conducta responsable de los
cientficos, es de lejos insuficiente para
comprender los aspectos ticos de la
ciencia. La razn de esta insuficiencia es
que mide la adecuacin moral de la
conducta del cientfico sobre el
trasfondo de "la ciencia" como una
institucin autosuficiente e
indiscutiblemente certera en todos sus
presupuestos. Bastara entonces con dar
al cientfico una formacin amplia que
le permita reflexionar sobre sus
inclinaciones e intereses, como
recomendaba antes el documento
mencionado, para lograr un
conocimiento objetivo y neutral respecto
de valores ticos. Sin embargo, en

primera instancia, se puede decir que


este enfoque es muy defectuoso porque
supone que tal conocimiento es posible,
logrando solamente la
"deshumanizacin" del investigador. Si
bien es cierto que es necesario despojar
a la investigacin de elementos
espurios, como ciertos intereses o
inclinaciones, no es algo indiscutido que
todos los intereses o inclinaciones sean
negativos en la labor cientfica.
Quienes sostienen a ultranza la idea de
que el conocimiento cientfico es, y debe
ser, formalmente racional, objetivo y
ticamente neutral, es decir, no
atravesado por intereses o valoraciones
de ningn tipo, han sido denominados
"cientificistas". Una de las estrategias

del anticientificista para desactivar las


bases de la posicin cientificista es
apelar al argumento de la naturaleza
problemtica del conocimiento. Se ver
a continuacin cules son las
caractersticas de estos dos enfoques,
sus proponentes, los problemas
conceptuales, histricos y polticos que
entraan, as como las alternativas de la
disputa en la que estn involucrados.
El cientificismo, el anticientificismo y
la tica
Se considera al cientificismo como un
punto de vista sobre la naturaleza de la
ciencia, la tecnologa y sus relaciones
con el resto de la sociedad
caracterizado por considerar a la
ciencia, no slo independiente del resto

de la sociedad, sino tambin su rectora.


La preeminencia de la misma sobre los
dems mbitos sociales se supone
derivada de su capacidad de obtener y
formular un conocimiento de la realidad
absolutamente racional y objetivo. Las
teoras cientficas, como resultado de
ese proceso altamente preciso y seguro,
proporcionaran una descripcin y
explicacin de los fenmenos reales
totalmente cierta y confiable.
En los estudios epistemolgicos sobre la
ciencia, fueron las corrientes
denominadas positivismo y empirismo
lgico[45] las que trasmitieron esta
imagen del conocimiento cientfico. Sin
embargo, desde otras perspectivas
epistemolgicas se fueron planteando

objeciones a esa imagen y sus


implicancias, de modo que el
positivismo o empirismo lgico fueron
perdiendo vigencia y terminaron siendo
superados por otras perspectivas que,
lejos de idealizar el conocimiento
cientfico, mostraron sus races
histricas y humanas. Estos cambios en
el enfoque sobre la naturaleza de la
ciencia provinieron tambin desde otros
estudios filosficos, desde la
epistemologa, tales como la historia de
la ciencia y la filosofa de la tecnologa,
y desde otros mbitos no filosficos
como la sociologa y la antropologa. Se
puede decir que hoy da una
caracterizacin ajustada de lo que es la
ciencia no puede prescindir de alguno

de estos distintos puntos de vista. Los


estudios sobre ciencia, tecnologa y
sociedad, justamente, representan un
enfoque integral del fenmeno
tecnocientfico.[46]
Es preciso aclarar, no obstante, que si
bien el positivismo lgico ha sido
reemplazado en su presentacin
primitiva por otras concepciones que
son opuestas en sus presupuestos
fundamentales, los aportes que aquellos
realizaron al anlisis de la ciencia
fueron tan importantes y fecundos que
constituyen la condicin necesaria de
toda la reflexin posterior. El control
emprico de las hiptesis y las
cuestiones lgicas que ste implica,
entre muchas otras, que fueron muy

elaboradas y estudiadas por los


miembros de aquellas corrientes, deben
ser tenidos en cuenta en cualquier
enfoque epistemolgico o metodolgico
que se precie de tal, aunque se discrepe
con los positivistas sobre su rol y
alcance.
Continuando con el tema del
cientificismo, se puede decir que uno de
sus ms famosos exponentes es el fsico
y filsofo argentino Mario Bunge (n.
1919) que se encuentra trabajando en
Canad desde 1966. Desde su punto de
vista, la ciencia es el nico modo de
saber que proporciona una comprensin
fundamentada de la realidad, por eso
est totalmente justificado su papel
central en la cultura contempornea.

Pero no solamente la ciencia predomina


sobre cualquier otro aspecto del sistema
cultural sino que la ciencia misma es un
modelo de tica. Afirma: "La ciencia es
un modelo de produccin con una
modalidad tica bien precisa: no puede
haber ciencia deshonesta, ciencia en
bsqueda deliberada del error, o que
eluda la crtica, o que suprima la
verdad. La bsqueda de la verdad
objetiva impone una recta conducta
[...]" (Bunge, 1996. 54). Y ms an:
"De aqu la posibilidad de moralizar
por la ciencia; esta posibilidad puede
realizarse si se adapta el cdigo de
conducta deseable en la vida diaria al
cdigo de conducta deseable en el
campo de la ciencia" (Bunge, 1996.

56). Las palabras de Bunge son


suficientemente elocuentes como para no
dejar dudas sobre el modo de
pensamiento caratulado como
cientificista.
Profundizando las crticas y objeciones
ya sealadas recibidas por el
cientificismo, se podra decir, en primer
lugar, que la influencia que ha tenido la
imagen de la ciencia construida por el
Positivismo y sus defensores ha sido y
es muy grande. Como sera absurdo
pensar que los tericos fundadores del
Positivismo, y otros filsofos que
adhieren a esa corriente, hayan obrado
de acuerdo con oscuros propsitos,
resulta claro que es en el terreno
poltico desde donde se ha logrado

instalar la idea de autonoma y


"santidad" de la ciencia11 de modo de
sacar partido de su poder potencial. Es
preciso aclarar que se entiende aqu el
trmino "poltica" en sentido amplio,
referido descripcin de los lazos
polticos, econmicos y culturales y las
teoras cientficas. Ver Gonzlez Garca,
M., Lpez Cerezo, J. y Lujn Lpez, J.
(1996). 11 La expresin es de Vandana
Shiva (1993). al modo en que una
sociedad y sus instituciones gestionan
sus recursos, proyectos, mbitos de
poder, etc., y no significando las
actividades partidarias del sistema
poltico formal. Al hacer de la ciencia
no una institucin ms en interaccin con
otros sistemas sociales, sino un

santuario encerrado en una torre de


marfil desde la cual slo se emite
verdades indiscutidas, se ha impedido al
resto de la sociedad participar en un
dilogo con la comunidad cientfica
sobre la conveniencia de proseguir
investigaciones propuestas o de
conservar teoras que se han mostrado
perjudiciales, aun cuando las
actividades de esta comunidad ponen en
riesgo sus intereses y hasta su
supervivencia.
Habra que preguntarse, entonces, si el
conocimiento cientfico contiene los
caracteres que el cientificista dice que
tiene y que justificaran su posicin. Si
se encuentra que no los tiene, o son
imposibles de definir claramente,

entonces la posicin anticientificista,


cuya tesis es opuesta a la del
cientificista en la medida en que
considera a la ciencia como
conocimiento construido, provisorio y
negociable, histrica y culturalmente
contextualizada y ticamente
comprometida junto con la tecnologa,
encontrara razones de peso para
rechazar la posicin cientificista, que,
como se ha dicho, pretende que el
conocimiento cientfico tiene
preponderancia sobre todo tipo de
conocimiento y por eso es ajena a todo
requerimiento de orden social y tico.
En Ciencia, tecnologa y sociedad
(Gonzlez Garca, Lpez Cerezo y Lujn
Lpez. 1996) se ha sistematizado el

conjunto de objeciones epistmicas que


comnmente se formula contra la idea de
ciencia como conocimiento universalmente vlido, racional y objetivo que
es la que sustenta la postura cientificista;
esas objeciones son las siguientes:
La fragilidad del conocimiento
inductivo: remite al problema ya
planteado por David Hume
(filsofo britnico, 1711-1776) en
el siglo XVIII, segn el cual ningn
nmero finito de enunciados
singulares puede justificar
concluyentemente un enunciado
universal. Ninguna de las dos
versiones de la induccin, ni la
verificacionista ni la confirmatoria,

se salva de la condena de este


argumento. Las observaciones son
limitadas y las enunciados de la
ciencia son generales, abiertos, de
modo que el conocimiento que
brindan no es totalmente racional
(en el sentido de tener un
fundamento absoluto), sino slo
"razonable", dependiente ms del
acuerdo intersubjetivo que de la
certeza objetiva. Tampoco la
refutacin resulta un proceso
concluyente, aunque el esquema
lgico correspondiente lo sea, pues
en ciencias fcticas las hiptesis no
se ponen a prueba aisladamente
sino en "red" con otras creencias e
hiptesis auxiliares, lo cual hace

imposible determinar su falsedad.


Tomas Kuhn (1922-1996),
socilogo e historiador de la
ciencia, asest un golpe definitivo
a la concepcin del conocimiento
emprico tradicional al introducir
nociones impensables hasta ese
momento en el hallazgo y
evaluacin de ese conocimiento: a
travs del concepto de paradigma
postul que una amalgama de
factores tecnolgicos, pedaggicos,
psicolgicos, sociolgicos,
considerados no epistmicos por la
tradicin empirista, era la
responsable de las teoras
aceptadas. La historia interna de la
ciencia se muestra as insuficiente

para explicar el modo en que la


ciencia debe resolver sus
problemas.
Relativismos: a partir de Kuhn se
desarrollan con intensidad los
estudios sociolgicos de la ciencia.
Con la entrada de la sociologa, la
frontera entre ciencia y sociedad
parece desdibujarse. La ciencia no
es ya un dominio privilegiado
donde se produce el conocimiento,
sino que ste surge de las mismas
fuentes que alimentan cualquier
creencia comn. La calificacin de
buena o mala ciencia ya no
depende slo de criterios
epistmicos sino tambin de
intereses y convenciones sociales.

Michael Mulkay (n. 1936),


socilogo ingls que trabaja
actualmente en la Universidad de
York, por ejemplo sostiene que la
ciencia toma muchas veces del
mbito cultural externo ciertos
trminos o conceptos cuando sus
recursos interpretativos fracasan o
que tambin es receptora de las
demandas de intereses de grupo, y
esto le permite afirmar que "el
contenido de la ciencia se afecta
por factores sociales y culturales
originados fuera de la ciencia"
(Mulkay, 1994. 347). Por lo tanto,
el valor cognitivo de una teora
cientfica resultara en aras de su
justificacin relativa a los

contextos particulares, o sea, a los


criterios correspondientes a
aquellos sectores externos, o bien
carecera por completo de
justificacin.
La carga terica de la observacin:
esta cuestin alude a que lo que se
ve o percibe depende tanto de las
impresiones sensibles como del
conocimiento previo, las
expectativas, los prejuicios y el
estado interno general del
observador. Este modo de concebir
la observacin complica la
explicacin del cambio terico
(cuando una teora es sustituida por
otra que le es incompatible) o bien
la eleccin entre teoras rivales

incompatibles. El concepto de
paradigma de Kuhn expresa en
forma radical este problema, pues
cientficos que trabajan bajo
paradigmas diferentes tambin ven
el mundo de modo diferente.[47] La
cuestin de la carga terica
profundiza el problema de la
relatividad.
Infradeterminacin: es un problema
ocasionado por la posibilidad de
establecer teoras alternativas
incompatibles para explicar un
mismo conjunto de fenmenos. Otra
manera de expresar esto es afirmar
que la evidencia emprica es
insuficiente para determinar la
solucin de un problema dado. La

infradeterminacin se relaciona con


el problema de la carga terica y la
inconmensurabilidad.
Clausura metodolgica: a raz de
los problemas anteriormente
mencionados se produce un vaco
epistmico entre la evidencia
emprica y las teoras respectivas.
Como no hay recursos de ese orden
para llenar esa brecha se recurre a
factores no epistmicos, que, segn
los autores, pueden ser"-Factores
tcnico-instrumentales, como
tradiciones tcnicas y
disponibilidades instrumentales,
que canalizan el tipo de
resultados obtenidos y la
interpretacin de los mismos; -

Factores sociales, es decir,


factores econmicos,
profesionales, polticos o
ideolgicos (el 'contexto social',
en pocas palabras), que tienen un
papel crucial en la interpretacin
de los resultados tericos y la
promocin selectiva de ciertas
tecnologas" (Gonzlez Garca,
Lpez Cerezo y Lujn Lpez, 1996.
46).
De este modo, parece clara la relacin
del contenido de las teoras con los
factores externos considerados no
epistmicos por la perspectiva
tradicional. En lugar de considerar este
modo de clausura como un

procedimiento espurio, se piensa que


favorece la consecucin de los objetivos
de la ciencia y disminuye la
incertidumbre ocasionada por la
infradeterminacin. Son importantes en
la medida en que permiten tomar
decisiones sobre asuntos de inters
general.
Este conjunto de restricciones al alcance
epistmico de las afirmaciones del
conocimiento cientfico es objeto de
amplio consenso entre los estudiosos de
la ciencia y ofrece, ms all de lo que
pueda implicar desde el punto de vista
estrictamente lgico o epistemolgico
de las teoras, una imagen de la ciencia
como una actividad humana ms, lejos
de la idealizada perspectiva del

cientificista. Y esto no es un rasgo


secundario, externo a la ciencia misma,
sino una condicin necesaria en la
produccin de su saber.
Las cuestiones puntualizadas ms arriba
estn apuntando a una concepcin del
conocimiento cientfico como algo
construido, no dado, sus resultados son
consensuados, no impuestos por "la
realidad"; los cientficos, que son seres
humanos, de algn modo deciden con
qu cosas han de tratar (ya sean tericas
o empricas), no las "encuentran" aqu y
all. Este modo de concebir el mundo de
la ciencia no significa que sus conceptos
y objetos sean libres invenciones. Hay
lmites respecto de la posibilidad de
construccin del conocimiento. Como

dicen los autores: "La naturaleza,


realidad, mundo externo, o como
queramos llamarlo, es interpretable de
diversas maneras (en este aspecto
reside el componente social del
conocimiento) pero no de cualquier
manera (de aqu su componente lgico
y emprico).[...] No decidimos, en un
sentido fuerte, cuales son los hechos
del mundo, aunque s asumamos o
alcancemos un consenso acerca de
cmo describirlo o acerca de cmo
manipularlos dados ciertos
presupuestos y ciertos fines
"extracientficos" (Gonzlez Garca,
Lpez Cerezo y Lujn Lpez, 1996. 51).
Esto significa decirle "no" al
mantenimiento de una postura positivista

ya sin fundamento, pero tambin "no" a


la anticiencia. La idea es desenmascarar
a la ciencia en su pretensin hegemnica
injustificada para acercarla a la
sociedad y ponerla a su servicio, es
decir, invertir la direccin que
actualmente se da entre ambas.
Ciencia bsica, ciencia aplicada,
tcnica y tecnologa
En los tpicos que hacen a la tica de la
ciencia, tratados hasta el momento, no se
ha hecho ninguna alusin a las posibles
diferencias entre ciencia y tecnologa.
Ahora es preciso referirse al tema
porque otro de los factores que
favorecen el mantenimiento de una idea
de ciencia desvinculada de la sociedad
y productora de conocimiento universal

y necesario, tal como la ve el


cientificista, es justamente la
diferenciacin que se suele establecer
entre ciencia o ciencia aplicada y
tecnologa.[48] La distincin da pie
muchas veces a la creencia de que la
ciencia es valorativa o ticamente
neutral porque slo se ocupa de
producir conocimiento, el cual es
indiferente respecto a valores o normas
morales, y que es la tecnologa, en tanto
aplicacin de ese conocimiento, la que
tiene propsitos o fines que pueden ser
intrnsecamente malos. Este argumento
general del cientificista suele tener
mucho peso en la sociedad por la propia
consideracin de los cientficos
respecto de su actividad; segn Stewart

Richards, cientfico ingls


contemporneo dedicado al estudio de
la epistemologa y la historia de la
ciencia, "Muchos cientficos
practicantes, que ignoran o desprecian
la filosofa de la ciencia, se afe-rran a
una concepcin idealizada de su
profesin y propagan una opinin de la
'verdad cientfica' que implica absoluta
certeza, objetividad y desprendimiento.
Tal punto de vista podra sostenerse
con conocimiento total de que muchos
tipos de ciencia pueden practicarse
solamente en virtud del apoyo
financiero proporcionado por los
gobiernos o las compaas industriales
con fines que frecuentemente son poco
claros, y casi siempre dirigidos por

intereses polticos o econmicos"


(Richards, 1987. 172). La distincin
entre ciencia y tecnologa suele deberse,
adems, a que la gente tiene mayormente
contacto directo con tcnicos o
tecnlogos y no con los cientficos,
quienes permanecen a salvo aislados en
gabinetes o laboratorios, y es a aquellos
a quienes atribuyen toda la
responsabilidad (causal y moral) por los
daos y perjuicios en el medioambiente
social y natural.
Es momento de preguntarse qu
razonabilidad tiene la afirmacin sobre
la distincin tajante entre ciencia y
tecnologa, como si fueran dos empresas
con propsitos cognitivos, sociales,
polticos y econmicos tan diferentes

que, en tanto objetos de una evaluacin


tica, segn los cientificistas, deben ser
tratadas como mbitos independientes.
Uno de los autores internacionalmente
conocido que apoya la distincin es
Mario Bunge, filsofo al que ya se ha
hecho referencia. Bunge distingue entre
ciencia bsica, ciencia aplicada y
tecnologa. Las dos primeras tienen
como objetivo la produccin de
conocimiento aplicando el mtodo
cientfico, pero mientras la primera
"trabaja en los problemas que le
interesan (por motivos puramente
cognoscitivos), el investigador
aplicado estudia solamente problemas
de posible inters social" (Bunge, 1997.
42). Por eso, mientras la investigacin

bsica no se puede planear, la aplicada


s. En la ciencia bsica el cientfico
debe ser absolutamente libre en la
eleccin de los temas a investigar, y en
el tiempo y los procedimientos que
utiliza. Si no se entorpece este modo de
trabajo del cientfico bsico, el mismo
producir resultados beneficiosos a la
ciencia aplicada, a la tcnica y a la
sociedad en forma automtica.
La tcnica se diferencia de las dos
formas de ciencia porque mientras stas
buscan las leyes que explican la
realidad, el propsito de la primera es
controlar la realidad empleando
principalmente el conocimiento y el
mtodo cientfico, pero tambin otros
tipos de conocimientos. Se encarga de

solucionar problemas prcticos, no


cognoscitivos. La diferencia en los
resultados de la investigacin
bsica/aplicada y la tcnica tambin es
inequvoca: en la primera el producto
final es conocimiento y en la segunda es
un artefacto o plan de accin con valor
prctico.
El cuadro trazado por Bunge no deja
duda acerca de la concepcin del autor
sobre la ciencia, la tcnica y sus
relaciones. Y explica por qu sostiene la
neutralidad valorativa de la ciencia y no
de la tecnologa: la primera es
totalmente independiente de cualquier
inters externo u objetivo que trascienda
el puro conocimiento, en cambio la
segunda produce efectos reales en el

mundo. Una de sus conclusiones sobre el


tema de la tica, la ciencia y la tcnica,
es la siguiente: "A diferencia de la
ciencia bsica o pura, que es
intrnsecamente valiosa o, en el peor
de los casos, carente de valor, la
tecnologa puede ser valiosa o
disvaliosa, segn sean los fines a los
cuales sirve. Por consiguiente es
preciso someter la tecnologa a
controles morales y sociales" (Bunge,
1996. 125).[49]
En la misma vena que Bunge, Nicolas
Rescher (n. 1928), filsofo alemn
radicado en EE.UU., en su obra Razn y
valores en la Era cientficotecnolgica afirma que "Las ciencias de
la Naturaleza y la tecnologa van

juntas como piezas de un mismo


cuerpo. Ninguna puede ir lejos sin la
otra" (1999, 100). Esta unidad, sin
embargo, se debe al sofisticado
desarrollo cientfico-tecnolgico actual
y no a la funcin intrnseca de cada una.
En este ltimo sentido, considera que el
cometido de la ciencia es puramente
cognitivo: "la representacin y
racionalizacin 'desinteresadas' del
hecho objetivo" (1999, 108). Mientras
el conocimiento cientfico es slo
descriptivo, el conocimiento tecnolgico
presenta una vertiente cognitiva y otra
normativa: la primera corresponde al
"saber cmo hacer" y la segunda al
"saber si" hacer algo concreto es
razonable dadas las circunstancias. El

saber hacer es especficamente tcnico,


pero generalmente est enraizado en el
conocimiento cientfico. Por ser la
ciencia descripcin (y explicacin) de
lo que es, considerarla "antitica"
respecto de los valores e intereses
humanos es "irracional e inapropiado".
"Las cuestiones normativas de valor,
importancia, legitimidad y similares,
quedan simplemente 'fuera del tema' en
este proyecto" (1999, 111).
Otros autores encuentran ms compleja
la cuestin de la distincin entre ciencia
y tecnologa y la consiguiente atribucin
de responsabilidad moral. Evandro
Agazzi (1934), filsofo italiano,
considera que ciencia y tecnologa
conforman un fenmeno de la

modernidad que no tiene precedentes en


pocas anteriores. La constitucin de la
tecnologa, afirma, es una consecuencia
de la ciencia moderna; es esta ciencia la
que proporciona los fundamentos
tericos de la accin eficaz. Por eso la
tecnologa no es la forma moderna de la
tcnica sino una rama de la tcnica, la
que se puede entender como "ciencia
aplicada". La tcnica actual sigue el
camino de la tcnica antigua: la
fabricacin de dispositivos tiles
descubiertos empricamente, sin usar, ni
necesitar, conocimiento cientfico. Una
diferencia entre tcnica y tecnologa que
tiene relacin directa con el tema de la
responsabilidad moral es el vnculo que
resulta establecido, por parte de cada

una, entre la humanidad y el mundo


artificial. El surgimiento de la tcnica de
algn modo produjo una bifurcacin en
el mundo humano al producir lo
artificial, pero esto signific una
domesticacin del mundo natural para
satisfacer las necesidades de la
naturaleza humana; adems, el
crecimiento de ese mundo artificial fue
lento y fragmentario, permitiendo as la
integracin paulatina de los artefactos al
ambiente natural y al contexto de las
condiciones humanas existentes. En el
caso de la tecnologa, la naturaleza no se
domestica sino ms bien es sustituida
por los artefactos, y el mundo artificial
crece con una rapidez, una amplitud y
una complejidad tal que le confiere las

caractersticas de una autonoma


creciente. Aunque la produccin de cada
tecnologa es local, su impacto y
consecuencias, que son imprevisibles,
inmediatamente se globalizan.
El modo en que cada uno de estos
procesos, el tcnico y el tecnolgico, de
acuerdo con Agazzi, se relaciona con el
tema de la tica, se da a travs de la
necesidad de regulacin requerida por
el mundo artificial que generan. En el
caso de la tcnica, se supona que sta
se pona al servicio del hombre
adaptando la naturaleza a sus
necesidades, pero se lo haca
"obedeciendo a la naturaleza", motivo
por el cual la necesidad de regulacin
estaba implcita en las "reglas del

oficio". Pero en el caso de la tecnologa,


el mundo artificial es creado por los
seres humanos en forma independiente
de la naturaleza, por eso necesita ser
regulado por los propios seres humanos
en forma de normas que aseguren la
armona y sabidura que antes se
confiaba a la naturaleza.
La caracterstica de la ciencia y la
tecnologa consistente en conllevar la
posibilidad de su uso incontrolado hace
que las mismas no contengan las
directivas de su propio uso. El campo
de la tica constituye el campo
apropiado desde donde se fijen las
directivas para su ejercicio positivo. Es
preciso tener en cuenta que para Agazzi
la posibilidad de distincin entre

ciencia y tecnologa es a lo sumo de


carcter conceptual, pero no existe entre
ellas ninguna distincin real, de ah la
pertinencia del trmino "tecnociencia"
que refiere a un fenmeno unitario.
No obstante, para el autor, la cuestin de
la responsabilidad exigida a la
tecnociencia no debe entenderse como la
de un asunto externo a la propia ciencia
o como la posible existencia de un
tribunal moral que juzgue sus
producciones. Es al cientfico
individual, en tanto investigador, a quien
compete el control responsable de la
tecnociencia, pues guindose por los
principios de la moralidad, puede
evaluar cada etapa de la investigacin
de modo que el resultado final pueda ser

visto por la humanidad como un


beneficio y no como una amenaza.
Sin embargo, el enfoque de Agazzi
puede ser cuestionado. La formacin
tica de los cientficos es necesaria, y
tambin lo es seguir las normas
establecidas para la regulacin de la
actividad cientfica. Pero de ninguna
manera es suficiente. Los peligros
inherentes a la tecnociencia, como dice
el propio Agazzi, son incontrolables, y
los conflictos actuales y potenciales que
genera exceden los lmites personales de
responsabilidad y pide el control ticopoltico y social. ste sera el modo ms
adecuado para que la ciencia y la
tecnologa logren la confianza y la
cooperacin de la sociedad toda en el

cumplimiento de los objetivos de la


ciencia, que no seran otros, de modo
mediato, que los de la propia sociedad.
Un ejemplo de armonizacin de los
intereses sociales y los cientficos se ha
visto en el presente documento en el
tema de la evaluacin de riesgo de las
tecnologas.
Len Oliv (n. 1950), filsofo
mexicano, rechaza la idea de que la
neutralidad valorativa sea una
caracterstica de la ciencia, porque tanto
ella como la tecnologa estn presentes
en el desarrollo de una tecnologa dada.
Tanto en una como en otra hay que
aplicar lo que cientficos y tecnlogos,
as como instituciones ecologistas, han
llamado el "principio de precaucin",

que exhorta a suspender la


implementacin de determinadas
tecnologas si existen bases razonables,
"aunque no se tenga evidencia
conclusiva de que existen relaciones
causales" (Oliv, 2000. 66), de que una
innovacin puede provocar daos en el
medioambiente.
A pesar de que es difcil establecer
cules son las bases razonables cuando
intervienen actores de cuo muy distinto
como empresarios y funcionarios de
Estado en la puesta en marcha de una
determinada tecnologa, la nica
solucin justa es la discusin y
participacin de todos los afectados por
esa tecnologa, incluso la comunidad
ms amplia.

Si parece difcil la aplicacin del


principio de precaucin en el campo de
la tecnologa por la diversidad de
actores intervinientes en las decisiones
y/o evaluaciones, ms difcil resulta
pensar que un cientfico, antes de
obtener la evidencia conclusiva, sobre
bases slo "razonables", deba, por
razones ticas solamente, suspender un
determinado desarrollo. Sin embargo,
como documenta Oliv, ese caso existi:
Mario Molina, Premio Nobel de
Qumica (1995) junto a su colega
Sherwood Rowland, enfrentaron lo que
Molina llam "un problema de tica
superior", a principios de la dcada del
70, cuando sospecharon que los
clorofluorocarburos (CFC), presentes en

muchos productos industriales


(aerosoles, aire acondicionado, etc.),
provocaban la destruccin de la capa de
ozono de la atmsfera terrestre. No es
necesario reproducir aqu la historia de
lo sucedido; lo importante es el nfasis
de Oliv sobre el hecho de que las
consecuencias dainas de la sustancia
podan ser detectadas en el "contenido"
mismo de la creencia cientfica. Por eso
afirma tajantemente: "No es cierto que
los nicos problemas morales de la
ciencia y la tecnologa los constituya el
uso posterior (bueno o malo) que se
haga de los conocimientos" (Oliv,
2000. 75).
Reflexiones generales sobre el tema
Los beneficios que la ciencia y la

tecnologa proporcionan a la sociedad


son cuantiosos, pero tambin estn
rodeados de riesgos. Aqu se ha
defendido la idea de que la
responsabilidad por los riesgos actuales
y potenciales de la empresa
tecnocientfica abarca diversos aspectos
de las actividades de investigacin y
tambin los resultados de la misma.
Cientficos y tecnlogos comparten esa
responsabilidad en la medida en que
ciencia y tecnologa no constituyen
mbitos separados con su propia lgica,
sino son extremos de un continuo que los
incluye.
Antes de la explosin tecnolgica del
siglo XX, la ciencia guardaba cierta
distancia del resto de la sociedad, pero

con el surgimiento de tecnologas de uso


cotidiano, la gente comienza a tomar
conciencia de la participacin en los
riesgos que tiene la ciencia. El
cuestionamiento por los
"descubrimientos" cientficos se fue
extendiendo y llev a que muchos
sectores sociales tomaran cartas en el
asunto. Los comits de tica en
hospitales y centros de salud, los
acuerdos internacionales sobre el
cuidado de los recursos disponibles en
el planeta, como el protocolo de Kyoto
de 1997 y la recientemente finalizada
Cumbre de Copenhague sobre la
reduccin de gases que causan el efecto
invernadero, y otros acuerdos regionales
sobre el uso cauteloso de los bienes

naturales compartidos, son ejemplos de


la bsqueda de lmites a las actividades
de la ciencia y la tecnologa.
Aun as los controles polticos y
sociales son muy resistidos, y lo son en
nombre de una ciencia que muchas veces
se autocalifica como autnoma y
reclama independencia completa de los
dems sectores sociales. Ya se vio que
algunas posiciones filosficas sobre la
ciencia respaldan tal pretensin
argumentando que la ciencia slo
produce conocimiento objetivo
universal-mente vlido, que por eso
mismo es neutralmente valorativo, no
pudindosele imputar ningn tipo de
responsabilidad moral. Pero esto no es
otra cosa que el enfoque que hemos

llamado "cientificista" que es cerrado y


determinante respecto del papel
jerrquico y hegemnico que otorga a la
ciencia y los valores que porta sobre
otros mbitos de inters social.
Se ha tratado de mostrar aqu la
insuficiencia de esta perspectiva, ya sea
desde el punto de vista epistemolgico,
tico o social. El conocimiento
cientfico no tiene la precisin ni la
objetividad que se crea al comienzo,
tampoco est libre de las influencias
externas en la eleccin de problemas y
la justificacin de las soluciones.
Adems, como hija de su tiempo, puede
estar sujeta a la direccin que le marcan
las fuentes de financiacin o los fines
polticos. En el terreno de los efectos y

resultados de las investigaciones, la


falta de responsabilidad ha sido y es
frecuente: la sofisticacin de los
armamentos es cada vez mayor, los
experimentos con personas no
informadas en lugares del Tercer Mundo
an ocurren, siempre aparecen
sustancias nuevas que son
contaminantes, etc.
La solucin propuesta consiste en que
los proyectos de investigacin y de
desarrollo tecnolgico sean puestos a
consideracin de los interesados o
afectados por ellos. A muchos puede
parecerles al menos utpica este tipo de
sugerencia, pero no hay soluciones
indiscutibles, salvo en lo que hacen al
mantenimiento del respeto y dignidad de

las personas. Adems, una ciencia no


contaminada por los intereses
econmicos, polticos y sociales no deja
tampoco de ser una idea utpica. La
dimensin tica la atraviesa, como lo
hace con todo quehacer humano.
Es preciso aclarar que al tratar el tema
de tica de la ciencia y sus aplicaciones
derivadas, que es objeto de enfoques
controvertidos e interpretaciones
contrapuestas, se incurre en
simplificaciones. Algunas veces ocurre
involuntariamente, pero la mayora de
las veces es necesario realizar recortes
con el fin de lograr concretar algunas
ideas, aunque con ello se cercene la
profundidad que tiene este tema. Queda
pues, como tarea siempre pendiente,

rescatar la riqueza y complejidad que le


es inherente al estudio de un fenmeno
tpicamente humano como es el de la
ciencia y su relacin con la tica.
Bibliografa
Agazzi, E. (1997). Equivalence or
Separation Betwen Science and
Technology?
En From Technique to Technology: The
Role of Modern Science. Society for
Philosophy & Technology. Vol. 4, n 2.
vila, H. (2002). Ciencia y tica. En
Temas de pensamiento cientfico.
Buenos Aires: Eudeba.
Bonilla, A. (2003). La tica de la
responsabilidad de Hans Jonas en su
discusin contempornea. (Conferencia
sin editar). Pontificia Universidade

Catolica Do Rio Grande Do Sul. Brasil.


Bunge, M. (1996). tica, Ciencia y
tcnica. Buenos Aires: Sudamericana.
Bunge, M. (1997). Ciencia, tcnica y
Desarrollo. Buenos Aires:
Sudamericana.
Ciapuscio, H. (1994). El fuego de
Prometeo. Buenos Aires: Eudeba.
Echeverra, J. (1998). Filosofa de la
ciencia. Madrid: Akal.
Funtowicz, S. y Ravetz, R. J. (1997).
Problemas ambientales, ciencia
postnormal y comunidades de
evaluadores extendidas. En Gonzlez
Garca, M. I., Lpez Cerezo, J. A. y
Lujn Lpez, J. L. Ciencia, tecnologa y
sociedad. Buenos Aires: Ariel.
Gonzlez Garca, M. I., Lpez Cerezo, J.

A. y Lujn Lpez, J. L. (1996). Ciencia,


tecnologa y sociedad. Madrid: Tecnos.
Guariglia, O. (1996). Moralidad. tica
universalista y sujeto moral. Buenos
Aires: Fondo de Cultura Econmica.
Laudan, L. (1984). Science and Values.
Berkeley: Univ. of California Press.
Mulkay, M. (1994). La ciencia y el
contexto social. En La explicacin
social del conocimiento. Mxico:
UNAM.
National Academy of Sciences. (1995).
On Being a Scientist: Responsible
Conduct in Research. Washington DC.
Oliv, L. (2000). Ciencia y tecnologa.
En Los linderos de la tica. Madrid:
Siglo Veintiuno.
Rescher, N. (1999). Razn y valores en

la Era cientfico-tecnolgica.
Barcelona: Paids.
Richards, S. (1987). Filosofa y
sociologa de la ciencia. Mxico: Siglo
Veintiuno. Shiva, V. (1993).
Colonialism and the Evolution of
Masculinist Forestry. En Harding, S.
(ed.). The "Racial" Economy of
Science. Indiana University Press.

Notas
[1]1

Samaja, J. (1993). Epistemologa y


Metodologa (329). Buenos Aires:
Eudeba.
[2]2. Recomendamos para el
conocimiento de la obra de Lewis
Carroll la edicin de: Eduardo Stilman
(comp. y trad.). (1998). Lewis Carroll.
Los libros de Alicia. Buenos Aires:
Ediciones de la Flor y Best Ediciones,
con prlogo de Jorge Luis Borges. En la
misma, se encuentra la edicin crtica de
las Aventuras de Alicia en el pas de las
maravillas, A travs del espejo y lo que
Alicia encontr all, La avispa con
peluca, La caza del Snark y una

cantidad importante de cartas y


fotografas.
[3]3. Las citas de la obra de Lewis
Carroll, Alicia en el pas de las
maravillas, corresponden a la edicin
de Alianza, Buenos Aires, 1990.
[4]4. En 1856, a pedido del editor de sus
primeros poemas, Charles Dodgson
debe elegir un seudnimo por lo que
propone varios: 1) Edgar Cuthwellis,
por transposicin de Charles Lutwidge,
2) Edgar U. C. Westhill, 3) Louis
Carroll, derivado de Lutwidge=Ludovic
y Charles=Carolus, 4) Lewis Carroll. El
editor escogi la ltima opcin. En
adelante har una tajante separacin
entre las obras publicadas por Dodgson,
las publicadas por Carroll y otras

totalmente annimas.
[5]1 Eco, U. (1993). El nombre de la
rosa. Barcelona: RBA Editores.
[6]2 Bertelloni, F. (1997). Para leer El nombre de la rosa de
Umberto Eco. Sus temas histricos, filosficos y polticos.

Buenos Aires: Oficina de publicaciones


del C.B.C.
[7]3 Eco, U. (1993, 198).
[8]4 Borges, J. L. (1969). El Golem. En El
otro, el mismo. Buenos Aires: Emec.
[9]5 Morris, C. (1985). Fundamentos de
la teora de los signos. Barcelona:
Paids.
[10]1 Pitcher, G. (1964). The Philosophy
of Wittgenstein. Englewood Cliffs:
Prentice-Hall.
[11]2 Devereux, G. y Weiner, F. (1950).
The Ocupation Status of Nurse. En

American Sociological Review. N 5


(vol. 15).
[12]3 Weininger, O. Sexo y Carcter
(286). Buenos Aires: Losada.
[13]4 Tapia, A. El rbol de la retrica.
[en lnea]. [consulta: 26 de julio de
2010].
<http://elarboldelaretorica.blogspot.com/2
argumentativos.html>
[14]1 Mostern, J. (2000). Conceptos y
teoras en la ciencia (208-209).
Madrid: Alianza.
[15]2 El acertijo "MU" extrado de
Hofstadter, D. R. (1979). Gdel,
Escher, Bach: an Eternal Goleen Braid
(34-41 y 260-261). N.Y.C.: Basic
Books.
[16]3 Borges, J. L. (1979). El libro de

arena. En El libro de arena. Madrid:


Alianza-Emec.
[17]4 Martnez, G. (2001). Borges y la
matemtica. Buenos Aires: Eudeba.
[18]5 Martnez, G. y Pieiro, G. (2009).
Gdel para todos. Buenos Aires: Seix
Barral.
[19]1 Peri Rossi, C. (1999). El amor es
una droga dura. Barcelona: Seix
Barral.
[20]2 "La Esfericidad de la Tierra 2: De
la Antigedad a Coln". (2009, enero
6). Odisea csmica. Blog de
actualizacin astronmica y espacial.
[en lnea]. [consulta: 10 de junio de
2010]
<http://www.odiseacosmica.com>
[21]1 Elster, J. (2000). El cambio

tecnolgico. Barcelona: Gedisa.


[22]2 Elster, J. (1991). Tuercas y
tornillos (67-68). Barcelona: Gedisa.
[23]1 Se entiende por monismo
metodolgico a la corriente
epistemolgica que sostiene que existe
un nico mtodo aplicable a las ciencias
tanto naturales como sociales, afirmando
de esta manera el monopolio del mtodo
hipottico-deductivo, o del dialctico.
En este sentido, toda posicin monista
es reduccionista, ya sea que reduzca la
cuestin del mtodo al hipotticodeductivo, o ya sea que la reduzca al
dialctico. En este sentido, a ambos
tipos de ciencias fcticas, o bien se
aplica el mtodo hipottico-deductivo, o
bien se aplica el dialctico, reduciendo

ulteriormente toda otra posibilidad.


(Asti Vera y Ambrosini, 2009.189)
[24]2 Se entiende por pluralismo
metodolgico a aquella corriente
epistemolgica que sostiene que para
cada tipo de ciencias fcticas, es decir,
para las naturales y para las sociales,
existe un mtodo propio de justificacin
de hiptesis o teoras para cada una de
ellas; en el primer caso, podr ser el
inductivista o el hipottico-deductivo;
en el segundo, ser el intencional, el
gentico, etc. (Ibdem)
[25]3 Tradicionalmente se denominan
como "filsofos analticos" a aquellos
cuya actividad principal consiste en
argumentar a favor o en contra de
distintas posiciones filosficas, incluso

pensando a la argumentacin como la


principal actividad de los filsofos. Este
tipo de actitud filosfica estaba
encarnada en los filsofos sociales como Dewey-, los matemticos o los
cientficos naturales -como Carnal y
Reichenbach. Segn Rorty, actualmente
el panorama ha cambiado, y esta actitud
la detentan los abogados, y lo que
definira entonces a la filosofa analtica
hoy sera un "estilo", una "aptitud
filosfica" consistente en ser capaz de
discernir en cualquier argumento
"lagunas lgicas", y de producir un buen
argumento en favor de cualquier
posicin. Frente a esta corriente
filosfica encontramos a los llamados
"filsofos continentales", como

Heidegger, Nietzsche, Derrida,


Foucault, Gadamer, etc., cuyo objetivo,
sostiene Rorty, consiste en la bsqueda
de una reconstruccin histrica
exhaustiva, de una redescripcin del
pasado adecuada para distintos fines y,
en este sentido, Rorty los llama "crticos
de la cultura". (Comesaa, 1998. 114115)
[26]4 Asti Vera, C. y Ambrosini, C.
(2009. 218-220).
[27]5 Comprese con el mismo
concepto y el ejemplo ya citado en
este Documento.
[28]6 Phillip Melanchthon,
nacido como Phillip
Schwartzerd, fue un reformista
germano, colaborador de

Martn Lutero, y principal jefe


del luteranismo tras la muerte
de su fundador. Fue uno de los
lderes de la Reforma
Luterana, aunque su intencin
era lograr un entendimiento
entre protestantes y catlicos.
Su obra y lecciones han tenido
una gran influencia en la
educacin, tal es as que se
convirti en el autor ms ledo
de su tiempo, y algunas de sus
obras fueron textos de estudio
en las escuelas hasta 1800.
[29]1 Biblioteca Escptica. [en
lnea]. [consulta: 26 de julio
de 2010].
<http://bibliotecaesceptica.word

son-laspseudocienciasmario-bunge/#more-1743>
[30]2 Kuhn, T. (1986). La estructura de
las revoluciones cientficas. Mxico:
Fondo de Cultura Econmica.
[31]3 Edmonds, D. J. y
Eidinow, J. A. (2001). El
atizador de Wittgenstein. Una
jugada incompleta (222).
Barcelona: Pennsula.
[32]4 Semmelweis, I. (1988).
Etiologa, concepto y
profilaxis de la fiebre
puerperal. En El desafo de la
epidemiologa, problemas y
lecturas seleccionadas.
Organizacin Panamericana
de la Salud.

[33]1

Para deslindar los distintos


aspectos de la planificacin de la
poltica cientfica, nos basaremos sobre
todo en el captulo mencionado de E. P.
Yfera.
[34]2 Florentino Ameghino (1854-1911),
cientfico argentino fundador de la
paleontologa sudamericana a fines del
siglo XIX.
[35]3 El adjetivo "malthusiana" alude a la
prediccin de Malthus, T. R. (17661834), economista y demgrafo
britnico. Segn esa prediccin, puesto
que la poblacin tiende a crecer en
progresin geomtrica y los alimentos
en progresin aritmtica, llegar un da
en que la poblacin superar los medios
de subsistencia de no mediar obstculos

preventivos y represivos. Si se entiende


el factor "poblacin" como poblacin de
investigadores y el factor "medios de
subsistencia" en calidad de recursos
financieros, la prediccin resultante
sera que si el aumento de
investigadores crece geomtricamente y
los recursos financieros lo hacen en
proporcin aritmtica, la cantidad de
investigadores superar el volumen de
los recursos. Pero el caso ya
mencionado de Venezuela contara como
un contraejemplo de esa prediccin.
[36]4 El destacado es del autor.
[37]1 Bacon, F. (1961). Novum
Organum, Buenos Aires: Losada.
[38]2 Esta referencia a las fuentes
diversas de las cuales surge la teora y

la tecnologa de la informacin puede


servir como ejemplo de lo que se
sostendr ms adelante sobre la
construccin del conocimiento, tanto el
cientfico como el tcnico.
[39]3 Los distintos aspectos del
significado del concepto de
responsabilidad sealados en este
artculo son tratados por Bonilla, A.
(2003).
[40]4 El concepto de responsabilidad
colectiva genera muchos
cuestionamientos, por ejemplo: cmo
se puede adjudicar responsabilidad a un
grupo por daos que provocaron unos
pocos, sin violar los principios de la
libertad individual?, se puede atribuir
intencionalidad al grupo, considerado

distinto de sus miembros, para atribuirle


responsabilidad?, etc.
[41]5 La explosin ocurri en un reactor
de la planta nuclear de Chernobyl, ex
Unin Sovitica, el 26 de abril de 1986.
[42]6 Larry Laudan, por ejemplo, afirma
que la filosofa de la ciencia slo se
puede ocupar de valores epistmicos o
cognitivos, pero no debe preguntarse por
el deber ser de los cientficos.
[43]7 Washington DC: National Academy
of Science, 1995.
[44]8 Generalmente se cree que la
recompensa de la que gozan los
cientficos es de naturaleza intelectual u
honorfica, en tanto la del tecnlogo es
ms bien econmica, pues los inventos
tienen un valor comercial del que carece

la produccin cientfica.
[45]9 El origen del Positivismo lgico se
remonta a la constitucin del Crculo de
Viena en la Universidad de Viena en
1922. Sus representantes principales son
L. Wittgenstein y M. Schlick. Sostienen
una forma de empirismo estricto: las
proposiciones cientficas pueden ser
verificadas en forma completa por la
experiencia. El Empirismo lgico que
contina esta corriente, representado
principalmente por R. Carnap, C.
Hempel y H. Reichembach, encarna un
empirismo ms moderado: la
experiencia slo puede mostrar que las
proposiciones cientficas son probables.
[46]10 A partir de la obra de Kuhn
(1962) surge una profusin de trabajos

sobre la ciencia sealando


especialmente su relacin con la
sociedad actual; constituyen un conjunto
heterogneo, pero suelen agruparse bajo
la sigla CTS (Ciencia, Tecnologa y
Sociedad). Se ha distinguido entre la
tradicin europea de Science and
Technology Studies y la tradicin
americana: STS (Science, Technology
and Society). Aunque comparten
presupuestos generales sobre la
dimensin social de la ciencia y la
tecnologa, la primera enfatiza el origen
de las teoras cientficas, es decir, en la
ciencia como proceso. La segunda se
centra en la descripcin de los lazos
polticos, econmicos y culturales y las
teoras cientficas. Ver Gonzlez Garca,

M., Lpez Cerezo, J. y Lujn Lpez, J.


(1996). 11 La expresin es de Vandana
Shiva (1993).
[47]12 La tesis de la incomensurabilidad
entre paradigmas y el relativismo que
implica fue radical en La estructura de
las revoluciones cientficas (Kuhn,
1962), pero luego, ya en la Posdata de
La estructura... y en las obras
posteriores, fue debilitando ese
relativismo haciendo hincapi en la
inconmensurabilidad lingstica.
[48]13 Ciapuscio (1994) desarrolla un
panorama completo de las teoras sobre
las diferencias entre ciencia y
tecnologa.
[49]14 Es preciso aclarar que para
Bunge, el cientfico s es moralmente

responsable pues se puede prestar a


cualquier tipo de conducta corrupta o a
alterar el correcto proceso de
investigacin cediendo a presiones
externas. Ver Bunge (1996), captulo V.

También podría gustarte